Sei sulla pagina 1di 200

6-yr-old child with meningococcal meningitis develops a swollen left knee on the

fifth day of antibiotic treatment. Which of the following is the most likely etiology of
this finding?
- Hemorrhage into the joint occurring as a result of disseminated intravascular
coagulation
- Progression of septic arthritis that was unrecognized at the onset of illness
- Nonspecific edema from progressive sepsis-related capillary leak
- Immune complex deposition resulting from production of anti-meningococcal
antibodies
- Secondary bacterial infection from the immunosuppressive effects of meningococcal
infection

RESPIRATION

1) All of the following concerning the differentiation of viral and bacterial


pneumonias are correct EXCEPT :-
a. the white blood cell count in a viral pneumonia is lower than in bacterial pneumonia
b. viral pneumonia of the infants is always of a mild severity
c. the progression of a viral pneumonia is slower
d. a lobar infiltration is usually indicative of a bacterial origin

2) Symptoms of brochiectasis include all of the following EXCEPT :-


a. the recurrence of pneumonia affecting the same area
b. voluminous secretion of the airways
c. a loss of appetite , growth retardation & clubbing
d. cyanosis due to the circulatory abnormality

3) Recognized causes of pleural effusion include all EXCEPT :-


a. heart failure
b. pneumonia
c. nephrotic syndrome
d. none of the above

4)Recognized causes of chronic cough include :-


a. acute bronchitis
b. pneumonia
c. immunodefeciency
d. none of the above

5)All of the following is involved in the pathogenesis of asthma EXCEPT :-


a. hyperinflation
b. bronchospasm
c. excess mucous secretion
d. edema of the submucosa

6) Concerning bronchiolitis :-
a. most cases are in infants below 15 months
b. 25% of cases are due to respiratory syncytial virus
c. steroids are usually helpful
d. antiviral drugs are indicated in some cases

CARDIOLOGY

1) Which of the following would differentiate most clearly between a venous hum &
the murmur caused by a patent ductus :-
a. tha site of auscultation
b. auscultation during systole and diastole
c. a venous murmur is always soft
d. altering the paient's head position diminishes or accentuates the intensity of the
murmur

2) All of the following conditions are associated with a systolic murmur EXCEPT:-
a. anemia
b. aortic reguge
c. mitral incompetence
d. ventricular septal defect

3) Which of the following congenital valvular dis is associated with sever cyanosis
manifested during the first few days of life :-
a. PDA
b. the post ductal form of coarctation of the aorta
c. transposition of great arteries
d. tetralogy of Fallot

4) all of the following statements concerning isolated ventricular septal defect are
correct EXCEPT :-
a. a left to right shunt become more sever with time
b. decompansation might occur
c. signs of pulmonary congestion might occur
d. prophylaxis against endocarditis is not needed if asymptomatic and small lesion

5) Which is the most common valvular abnormality with rheumatic fever :-


a. mitral insufficiency
b. mitral stenosis
c. aortic insufficiency
d. tricuspid insufficiency

6)All of the following statements concerning rheumatic poly arthritis are correct
EXCEPT :-
a. it usually affect the great joints
b. joint deformaties develop if the appropriate ttt is not introduced in time
c. it is associated with a high fever
d. the affected joints are swollen, warm , and movements are painful

7) Symptoms of chorea minor include all of the following EXCEPT :-


a. impaired coordination
b. muscular hypotonia
c. increased reflexes
d . emotional lability

8) All of the following statements concerning the therapy of rheumatic fever is


correct EXCEPT :-
a. penicillin therapy is indicated
b. in case of carditis , prednisone is indicated for at least 6 weeks
c. ibuprofen is sufficient if the patient only exhibits polyartheritis symptoms
d. bed rest is indicated until complete remission

9) symptoms of subacute bacterial endocarditis include all of the following EXCEPT


:-
a. the initial symptoms include fatigue , pallor and loos of appetite
b. initially a moderate then a high grade fever is detected
c. microembolization of the skin and kidneys
d. painful erythematous subcutaneous nodules about the tips of the digits

10) The effects of digitalis administration in a patient with cardiac decompansation


include all of the following EXCEPT :-
a. a declining elevated central venous pressure
b. hepatomegaly become less pronounced
c. the heart rate decreases
d. a short PR segment is observed on the ECG

NEUROLOGY
1)Which of the following is the most common cause of meningitis in infants and
children :-
a. meningococcus
b. enteroviruses
c. tuberculosis
d. haemophilus influenzae

2) All of the following statements concerning the Guillian Barre' syndrome are
correct EXCEPT :-
a. a symmetrical , flaccid pralysis and abnormality of sensation are characteristic for the
disease
b. the disease improves spontaneously despite its long clinical course
c. corticosteroids are the mainstay of ttt of acute cases
d. differentiation of the disease from poliomyelitis is difficult in some cases

3) Which of the following statements concerning Duchenne muscular dystrophy is


correct :-
a. it develops during the 3rd – 5th years and involves the lower extremities and the pelvis
b. it affects boys and girls
c. the prognosis for survival is good
d. the condition is associated with decreased activities of creatine kinase and aldolase in
serum

4) All of the following statements about the epilepsy characterized by grand mal
(tonic- clonic ) seizures are correct EXCEPT :-
a. the seizures are characterized by flexion or extension positions of the lower extremities
b. an interictal EEG is not sufficient for the diagnosis
c. phenytoin , Phenobarbital and valproate are the most important therapeutic drugs
d. despite appropriate therapy , dementia develops in the majority of cases

5) Muscular hypotonia, without muscular weakness is characteristic for :


a. Guillain-Barre' syndrome
b. Rickets
c. Down's syndrome
d. Werdnig-Hoffmann syndrome

6) Which of the following statements about absence epilepsy in childhood are false :-
a. it is accompanied by a loss of consciousness for a few seconds
b. spike discharges with 3\s frequency in the EGG are typical
c. it responds well to drug therapy
d. it does not usually affect school performance

7) Which of the following condition is correctly paired with expected CSF findings :-
a. tuberculous meningitis and high glucose with raised protein
b. Guillain-Barre' syndrome and normal cell count with reduced protein
c. viral meningitis and CSF lymphocytosis with normal glucose
d. bacterial meningitis and low glucose and protein

8) Spasticity may be associated with all of the following EXCEPT:-


a. clonus
b. clasp knife rigidity
c. lower motor neuron lesion
d. quadriplegia

9) Which of the following is true about cerebral palsy :-


a. it is a progressive motor deficit
b. the cause is usually a perinatal insult
c. mental retardation is a constant feature
d. should be considered if neonatal reflexes are absent beyond 6 months of age

10) What is the most common cause of obstructive hydrocephalus in an infant :-


a. Dandy-Walker syndrome
b. meningitis
c. Arnold-Chiari malformation
d. Aqueductal stenosis.

NUTRITION

1) Which of the following tests is not suitable to confirm the diagnosis of rickets :-
a. the serum alkaline phosphatase activity >> high “earliest manifestation”
b. an x-ray picture of the wrist
c. the serum calcium level >> normal
d. the serum 25-OH-D3 level

2) All of the following abnormalities of body constitution and function can be


detected in nutritional marasmus EXCEPT :-
a. the subcutaneous adipose tissue of the face is lost first
b. a tendency for hypothermia is observed
c. hunger is usually observed
d. a more frequent occurrence of pneumonia

3) Which of the following is an absolute contraindication of breast feeding :-


a. smoking
b. insulin therapy
c. mastitis
d. phenylketonuria

4) Which of the following is correct about weaning :-


a. it is usually started as early as 8 months of age
b. cow's milk is better avoided during 1st year of life
c. egg white can be given at about 7 months of age
d. weaning practice mean decreasing the amount of milk given

5) Symptoms of hypervitaminosis D include all of the following EXCEPT :-


a. polydypsia and polyuria
b. anorexia and constipation
c. hypocalcemia
d. calcification of tissues( kidney, vessel)

6)Possible causes of the development of rickets in a child regularly supplemented


with oral vit. D include all of the following EXCEPT :-
a. an abnormality in the metabolism of vitamin D in the skin
b. an abnormality in the metabolism of vitamin D in the kidney
c. lipid malabsorption
d. hyperphosphaturia

7) All of the following steps in the therapy of marasmic infants are correct
EXCEPT :-
a. feeding during night is important
b. large volumes of parenteral infusions should be avoided to prevent overload on the
atrophic myocardium
c. vitamins, K+ and folic acid should be supplemented
d. the marasmic infant should be given large amounts of food, primarily milk

8) What is the protein requirement of a 2 month old infant :-


a. 1 g/kg/day
b. 2.5 g/kg/day
c. 10 g/kg/day
d. 4.5 g/kg/day

GENETICS

1)Characteristic features of autosomal recessive inheritance include


a)50%risk of recurrence
b)both parents can be normal
c)more males are affected
d)consanguineous marriage is not a feature

2)One of the following anomaly is not associated with Down syndrome


a)mental retardation
b)congenital valvular heart disease
c)simian crease
d)webbing of neck

3)In case of autosomal dominant inheritance the inherited feature would be


manifested in one of the parents and in
a)50%of daughters and 75% of sons
b)25% of sons and 75% of daughters
c)50% of sons and 50% of daughters
d)the daughters only

4)All of the following statements concerning sex linked recessive inheritance pattern
are correct except
a)the disease only affects boys
b)the father of the diseased sons might be affected
c)the diseased boys might have diseased grandfathers
d)not all of the daughters of a carrier mother will be carriers

5)The parents of a child who is suffering from a metabolic disease characterized by


an autosomal recessive inheritance pattern ask for your advice they intend to have a
second baby which of the following consideration is incorrect
a)the phenotype of 75% of the potential offspring will be normal
b)the genotype of 25% of the potential offspring will be normal
c)the genotype of 50% of the potential offspring will be normal
d)50% of the offspring will be heterozygous

Infection & Growth

1-Which of the following vaccines would be contraindicated in a 4 year old boy


receving immunosuppressive therapy for autoimmune hepatitis
a)Hepatitis A vaccine
b)Hepatitis B vaccine
c)Acellular pertussis vaccine
d)Varicella vaccine

2-Possible causes of fever of unknown origin include all of the following except
a)Pylonephritis
b)Subacute bacterial endocarditis
c)Hypothyroidism
d)Salmonella enteritis

3-How much is the head circumferance of a mature well-developed 6 month infant


a)43 cm
b)38cm
c)46 cm
d)50 cm

4-Characteristic milestones in a 9 month old infant include


a)Can wave "bye-bye 12mo
b)Walks holding furniture 12mo
c)Sits unsupported 7 mo
d)Can say "mama'dada 9 mo

5-A 4-month-old baby boy born at term with 3,200 g birth weight,currently weighs
6,000g What is your opinion
a)The infant is overweight
b)The body weight of the infant is normal
c)The infant's nourishment is deficient
d)The infant has a chronic illness

6-Possible complications of scarlet fever include all of the following EXCEPT


a)Submandibular lymphadenitis
b)Otitis media
c)Subacute sclerosing panencephalitis >> measles complication
d)Acute glomerulonephritis

7-Which of the following statements concerning Koplik's spots detectable in measles


is correct
a)They develop synchronously with the eruption on the skin
b)They are usually seen in the late phase of the catarrhal incubation period
c)They are most common during the convalescent phase
d)The cause is bacterial superinfection

8-Which of the following statements concerning varicella is correct


a)Desquamated crusts containing the virus can transmit the disease
b)School age children are routinely vaccinated
c)The patient should be bathed regularly , neutral powder is applied to relieve itching
d)Oral acyclovir (Zovirax) therapy is indicated

9-Possible manifestations of mumps virus infection include all of the following


EXCEPT
a)Pancreatitis
b)Submandibular lymphadenitis
c)Orchitis
d)Meningoencephalitis

10-Possible causes of fever of non -infectious origin include all of the following
EXCEPT
a)Hyperthyroidism
b)Epilepsy
c)Systemic lupus erythematosus
d)A regulatory imbalance of the autonomic nervous system

Neonatology

1-WHICH OF THE FOLLOWING FACTORS LEADS TO NEONATAL


HYPERBILIRUBINEMIA:-
A-shortened neonatal red cell life span.
B-impaired excretion of unconjugated billirubin
C-limited conjugation of billirubin in liver
D-increased enterohepatic circulation
E-all of the above

2-WHICH OF THE FOLLOWING IS TRUE ABOUT RESPIRATORY DISTRESS


SYNDROME?
A-ground glass appearance on chest x-ray is common
b-the disease usually worsens 2-3 days after birth
C-steroids reduce severity if given early to premature
D-surfactants therapy is rarely useful

3-CHARACTERISTICS OF CEPHALOHEMATOMA INCLUDE ALL EXCEPT:-


A-subperiosteal
B-limited by margin of bones
C-resolves by 48 hours
D-soft tissue swelling
4-PERSISTENT MORO REFLEX AFTER 16 WEEKS INDICATES:-
A-normal infant
B-brain damage
C-hungry infant
d-none of the above

5-A NEONATAL CAUSE OF SEIZURES WHICH HAS THE BEST PROGNOSIS:-


A-birth asphyxia
B-bacterial meningitis
c- Hypoglycemia
D-hypocalcaemia

6-PHYSIOLOGICAL JAUNDICE USUALLY APPEARS ON …..DAY AND


REGREE ON …….DAY:-
A-first day and third day
B-fifth day and ninth day
C-third day and seventh day
D-seventh day and eleventh day

7-ALL OF THE FOLLOWING ARE EXPECTED IN INFANT OF DIABETIC


MOTHERS EXCEPT:-
a- large baby
B-hyperglycemia
C-hypocalcaemia
D-respiratory distress syndrome

8-AT BIRTH NORMAL HEART RATE:-


a-60-80/m
b-70-120/m
c-80-110/m
d-110-150/m

9-ALL OF THE FOLLOWING ARE SEEN DURING HYPOTHERMIA EXCEPT:-


A-bradycardia
B-excessive shivering
C-pallor
D-metabolic acidosis

10-WHICH OF THE FOLLOWING ARE CORRECT ABOUT NEONATAL


INFECTIONS:-
A-it needs a high index of suspicion for diagnosis
B-umbilical infection is usually a minor problem
C-positive cultures are essential for diagnosis
D-respiratory distress is not an expected feature

HAEMATOLOGY

1) Which of the following is a typical symptom of Henoch-Scholein purpura :-


a. polyarthralgia
b. jaundice
c. a purpuric skin rush invoving the face
d. a prolonged bleeding time

2) During which childhood age is the onset of acute lymphoblastic leukemia most
frequent :-
a. 10-12 years
b. 6 months
c. aldoscence
d. 3-5 years

3) Bleeding due to thrombocytopenia occurs if the platelet count is less than :-


a. 150,000 / ul
b. 100,000 / ul
c. 80,000 / ul
d. 50,000 / ul

4) All of the following results are indicative for iron deficiency , EXCEPT :-
a. anemia
b. large, hypochromic RBCs observed in the peripheral blood smear
c. a low serum ferritin
d. an elevated total iron binding capacity (TIBC

5) All of the following diseases are associated with a decreased production of


RBCs ,EXCEPT :-
a. hepatitis
b. leukemia
c. hypothyroidism
d. ankylostoma >> increase RBCs loss

6) Which of the following interventions is not suitable for the therapy of ITP :-
a. glucocorticoid therapy
b. intravenus immunogloblin therapy
c. anabolic hormone therapy
d. plasmapheresis

7) All of the following laboratory findings are characteristic for ITP , EXCEPT :-
a. the platelet count is low
b. the prothrombin time (PT) , the partial thromboplastin time ( PTT) and the thrombin
time (TT) are all normal
c. the number of megakayocytes in the bone marrow is low
d. the WBC count is normal

8) Which of the following disorders may be associated with thrombocytopenic


purpura :-
a. meningitis
b. hypersplenism
c. Henoch-Scholein purpura
d. thrombasthenia

9) Which of the following statements is FALSE about anemias :-


a. a sever anemia causes dyspnea and tachycardia
b. cyanosis develops in sever anemia
c. a cardiac murmur may become audible in anemia
d. reticulocytosis ensures a hemolytic origin of anemia

10) Which of the following statements about Schonlein-Henoch purpura is FALSE :-


a. swelling of the joints lasts for few days or weeks
b. the occurrence of a microscopic hematuria suggests renal involvement
c. it may be associated with abdominal pain due to edema or hemorrhage of the intestinal
d. the disease lasts for years even in uncomplicated cases

GIT
1- the minimal fluid requirement  of a 6-months infant with mild diarrhea is :
a. 50   ml/kg
b. 75   ml/kg
c- 150 ml/kg
d. 300 ml/kg

2-what  is the optimal initial therapy in case of moderately severe dehydration :
a. Transfusion of whole blood
b. infusion of half-isotonic sodium chloride –dextrose solution
c. infusion of 10 % dextrose solution
d. infusion of 0.9% sodium chloride solution

 3- Disease in which a specific diet is indicated include all the following except :
a. post-enteric malabsorbtion
b. congenital sucrase-isomalatase deficiency
c. galactosemia
d. congenital hepatic fibrosis
 
4- Manifestations indicative of intussusception  include all the following except :
a. A suddenly developing and recurring abdominal pain
b. the passage of non-feculent ,bloody mucus
c.  during painful episodes , the infant looks flaccid & limp
d. the detection of fluid levels in abdominal X-ray

 5- possible causes of hepatic cirrhosis includes all the following except :
a. hepatitis A virus infection
b. chronic hepatitis
c. biliary atresia
d. hepatitis B  virus infection

6- The therapy of hepatic cirrhosis include all the following except :


a. an energy-rich, fat free diet & vitamin substitution are necessary
b. the protein intake must be restricted & the intestinal ammonia production should be
inhibited
c. in case of esophageal bleeding an immediate surgical shunt operation is needed
d. ascites formation may be decreased by a low salt intake & administration of
spironolactone  

7- Which of the following statements about recurrent abdominal pain in childhood


is false :
a. the peak of incidence is at the age of 9-10 Years
b. the most common localization is the peri-umbilical area
c. the pain is usually vague & lasts few minutes
d. it's usually associated with rigidity or abdominal tenderness

 8- Possible causes of diffuse enlargement of the abdomen include all the following
except:
a. Enteral infections
b. malabsorption syndrome
c. pyloric stenosis
d. hypokalemia
 
9- Which of the following is correct about stomatitis
a. herpangina is caused by herpes simplex infection
b. oral thrush causes small sized erythrmatous  ulcers
c. herpetic stomatitis is usually afebrile
d. is a common problem in malnourished  infants

10- features of hepatic cirrhosis include all the following except


a. cirrhotic nodules are usually palpable
b. bleeding from esophageal varices is not uncommon
c. the serum albumin concentration is decreased  
d. ascites  develops in the advanced stages

Nephrology
1- All the following concerning acute post-streptococcal glomerulonephritis are
correct,Except:
a. it can be prevented by the early antibiotic therapy of streptococcal infections
b. steroids treatment  Is usually helpful >> Ab for 10 days “penicillin”    
c. the prognosis of the disease in childhood is usually good >> 95% recover
d. a hypertensive encephalopathy may occur

 2- Possible complications of acute post-streptococcal glomerulonephritis include all


the following, Ecxcept:
a. hypermagnesemia
b. hyperkalemia
c. anuria
d. pulmonary edema

3- typical laboratory alterations in acute post-streptococcal glomerulonephritis


include all the following, Ecxcept:
a. the antistreptolysin titer in the serum is usually elevated
b. elevated serum creatinine & blood urea nitrogen levels
c. hematuria, proteinuria & granular casts in the urine
d. A normal increased C3 complement level in the serum

 4- All the following statements concerning idiopathic nephritic syndrome are
correct, Except:
a. the onset is usually at 1-6 years of age
b. the history reveals a " minimal change" process
c. the early development of renal failure is characteristic
d. patients usually do not exhibit hypertension

5- findings usually detectable in acute renal failure include all the following, Except:
a. the serum creatinine in concentration is elevated
b. hyperkalemia
c. hyponatremia
d. hypophosphatemia

6- The average insensible water loss per day is :


a. 500 ml/kg  
b. 500 ml/m2
c. 300 ml/kg
d. 300 ml/m2

ENDOCRINOLOGY

1)Which of the following laboratory data is possible to differentiate between


hypothalamic-pituitary secondary hypothyroidism & primary hypothyroidism?
A) A low serum T4 level & normal TSH levels
B) An elevated serum T4 level & low TSH levels
C) A low serum T4 level & elevated TSH levels
D) An elevated serum T4 level & elevated TSH levels

2)All of the following diseases cause short stature EXCEPT


A)Turner's syndrome
B)Hypothyroidism
C)Achondroplasia
D)Klinefelter's syndrome

3)All of the following statements concerning delayed puberty are correct EXCEPT
A)Diagnosis is made if signs of puberty are not detectable after the age of 13 in girls or
14 in boys
B)Majority of cases is constitutionally delayed puberty
C)An elevated testosterone level is usually found
D)A pituitary or hypothalamic tumor is a possible cause

4)All of the following statements concerning congenital hypothyroidism are correct


EXCEPT
A)None or very few physical symptoms are observed at birth
B)The serum TSH level is low
C)The appetite is decreased but the infant does not seem to be thin
D)The early introduction of therapeutic measures gives good results

5)Which of the following is the first step in the therapy of a diabetic ketoacidotic
coma?
A)Correction of the acidosis with NaHCO3
B)The subcutaneous administration of insulin
C)Fluid replacement with the infusion of a solution containing no glucose & the
administration of a rapidly acting insulin preparation intravenously
D)Fluid replacement with the infusion of a glucose –containing solution

6)Manifestations of diabetic ketoacidosis include all of the following EXCEPT


A)Sw
B)Vomiting
C)Respiratory distress
D)Abdominal pain

EMERGENCIES
1)Symptoms of hypovolemic shock include all of the following EXCEPT
A)Pallor & cold exteremities
B)Tachycardia
C)A low central venous pressure
D)Cardiac enlargement
E)Acidosis

2)Which of the following conditions associated with airway obstruction ,is NOT
life-threatening?
A)Acute rhinopharyngitis
B)Acute epiglottitis
C)Laryngeal foreign body
D)Laryngeal edema
E)None of the above

3)In which of the following intoxication is gastric lavage CONTRAINDICATED?


A)Narcotics
B)Hydrocarbons (gasoline , petroleum)
C)Barbiturates
D)Acetaminophen
E)Salicylates
4)All of the following are guidelines for the therapy of circulatory failure in a
child EXCEPT
A)The administration of oxygen
B)The administration of digitalis in all cases
C)Discontinuation of oral feeding , infusion of volume expander
D)Mechanical ventilation for respiratory insufficiency
E)The elimination of the negative inotropic factors (acidosis, hypoglycemia,etc.)

5)Possible causes of the development of a coma include all of the following ,


EXCEPT
A)Diabetic ketoacidosis
B)Cerebral edema
C)Phenobarbital intoxication
D)Encephalitis
E)Chorea minor
___________________________________________

Neonatology

1. Respiratory distress in newborn babies is recognized complication of all of the


following Except:
a- Group B streptococcal infection.
b- Congenital heart disease.
c- Sickle cell anaemia.
d- Cerebral disorders.
e- Diaphragmatic hernia.

2. An infant born at 32 weeks gestation is at increased risk from all of the following
Except:
a- Meconium aspiration.
b- Respiratory distress syndrome.
c- Hypoglycemia.
d- Hypocalcemia.
e- Intraventricular heamorrhage.

3. Infants of diabetic mothers are at increased risk from all of the following except:
a-Hypocalcemia.
b-Anaemia.
c-Jaundice.
d-Shoulder dystocia.
e-Congenital abnormalities.

4. Infants of diabetic mothers are at increased risk from all of the following except:
a- Hypoglycemia.
b-Hypocalcemia.
c-Intrauterine growth retardation.
d-Hypomagnesemia.
e-Hyperbilirubinemia.

5. The following features are not typical of physiological jaundice except:


a- Recognizable jaundice in the first 48 hours.
b- Peak plasma bilirubin at four to five days.
c- Persistence beyond first week.
d- Irritability.
e- Pale stools.

6. Jaundice is most likely to be physiologic in a term infant, in which of the


following situations.
a- Jaundice at 12 hours of age.
b- Serum bilirubin level increasing less than 5mg\dl\day in the first 2-4 days
c- Direct serum bilirubin greater than 1 mg\dl.
d- Jaundice at 12 days of age.
e- Usually associated with anaemia.

7- Jaundice appearing on day 1 of life suggested all of the following except


a- Torch infection.
b- Erythroblastosis fetalis.
c- ABO incompatability.
d- Sepsis.
e- Intrahepatic biliary obstruction.

8- Hyperbilirubinemia at 2 weeks of age suggests all of the following except:


a- Physiologic jaundice.
b- Hypothyrodism.
c- Pyloric stenosis.
d- Biliary atresia.
e- Breast milk jaundice.

9- Which of the following is not a cause of apnea in infants..


a- Prematurely.
b- Hyperglycemia.
c- Severe hypoxemia.
d- Intraventricular hemorrhage.
e- Sepsis.

10- All of the following are included in evaluating ABGAR score except:
a- Heart rate.
b- Respiratory effort.
c- Muscle tone.
d- Maturity of the newborn.
e- Response to catheter in nostrils.

11- Which on of the following is not utilized in determining an APGAR score.


a- Colour.
b- Heart rate.
c- Deep tendon reflexes.
d- Grimace.
e- Respiration.

12-Factors that leads to higher incidence of hyaline membrane disease includes of


the following except:
a- Multiple pregnancy.
b- Cesarean section.
c- Intrauterine hypoxia.
d- Postmaturity.
e- Maternal diabetes.

13-Which of the non pulmonary causes of respiratory distress in the newborn.


a- Choanal atresia.
b- Neonatal pneumonia.
c- Congestive heart failure.
d- Lung collapse.
e- Hyaline membrane disease.

14- Regarding meconium,all of the following are true except:


a- More than90% of newborns pass meconium in the first 24h.
b- Meconium plug may cause intestinal obstruction.
c- It is composed of epithelial cells, bile and mucus.
d- Meconium stained amniotic fluid is a sign of intrauterine distress.
e- Meconium aspiration can lead to hyaline membrane disease.

15- in hyaline membrane disease all of the following is correct except.


a- is more common in babies of diabetic mothers.
b- Is due to surfactant deficiency.
c- It is more common in premature neonates .
d- Occurs most commonly at first few hours post delivery.
e- Always requires ventilation

Fever & rash


1-Compications of measles may include.
a- Encephalomyelitis.
d- Laryngitis.
c- Giant cell pneumonia.
d- Thrombocytopenic purpura.
e-All of the above.

2-All of the following are associated with meningococcal meningitis except:


a- Abrupt onset with malaise, headache and irritability.
b- Projectile vomiting and prostration.
c- Chills and convulsions.
d- Cherry-red spot of macula.
e- Petechial or purpuric skin lesion.

3-Incubation period of meningococcal meningitits is


a-7-10 days.
b-2-3 days.
c-8-15 days.
d- 3 weeks.
e-10-20 days.

4-Infection with which one of the following organisms does not require isolation
measures:
a- Diphtheria.
b-Chicken pox.
c- Pertussis.
d- Mumps.
e- Brucella.

5-Infections with or complication of streptococcal infection include all of the


following except: .
a- Scartet fever.
b- Pneumonia.
c- Endocarditits
d- Erysipelase.
e-Scalded skin syndrome.

6-All of the following diseases are water borne except:


a- Cholera.
b- Bacillary dysentery.
c- Amoebic dysentery.
d- Tetanus.
e- Hepatitis A.

7-Common manifestations of congenital rubella syndrome includes all of the


following except:
a- Intra uterine growth retardation.
b- Hydrocephalus.
c- Congenital heart lesion.
d- Cataract.
e-Mental retardation.

8- the rash of chicken pox is characterized by all of the following except:


a- Incubation period is 10-16 days.
b- Rapidly progresses from papules to vesicles.
c- Lesions appear in crops.
d- Greatest concentration of lesion is on extremities. >>generalized except soles&palms
e- Vesicles develop on mucus membrane of mouth and genitalia.

9-In measles infection the following are characteristic except:


a- Incubation period is 10 days.
b-Rash appears on the second day.
c-The disease is infectious before the appearance of the rash.
d-Koplik spot is common.
e- The infant is immune against measles for the first 4-6 ms of life.
10-The rash in measles is .
a- Papulo vesicular.
b- Urticarial.
c- Maculo popular.
d- Appears on the first day of illness.
e-Last 24 to 48 hours.

11-which of the following is least characteristic of varicella infection.


a-an incubation period of 14-16 days.
b- A rash progressing from papular to vesicular.
c- A tendency for the rash to be heaviest on the arms and legs.
d- Temperature that is usually less than 38,9 c.
e- Itiching.

12- maternal infection and viremia with rubella virus during the early weeks of
pregnancy may result in any of the following except:
a- Congenital malformation.
b- Stillbirth.
c- Abortion.
d- Macrosomia and large for gestational age in infant.
e- Chronic infection of the fetus.

13-which of the following is not characteristic of rubella infection in young children.


a- Mild leucopenia.
b- Post auricular and occipital lymph nodes.
c- Four to five days of fever before the rash.
d- A pink red maculopapular rash.
e- Mild thrombocytopenia.

14-clinical manifestations of scarlet fever includes all of the following except:


a- Exudative tonsillitis.
b- White strawberry tongue.
c- Hepatosplenomegally.
d- Erythemato punctiform lesions on the palate.
e- Red strawberry tongue.

15-the rash of scarlet fever is characterized by all of the following except:


a- A rash sand paper- like texture.
b-A tendency to desequmate.
c-A tendency to become generalized with in 24 hours.
d-A tendency to spare area of skin folds (axilla,groin).spare circumoral pallor
e-A red, erythematous appearance.
16-the three most common bacterial causes of meningitis in childhood are.
a-Neisseria meningitides,hemophylus influenza,pneumococcus.
b-Neisseria meningitides, pneumococcus, E.coli.
c-Neisseria meningitides, group A streptococcus,staphylococcus.
d-Neisseria meningitides, hemophylus influenza, L.monocytogens.
e-Hemophylus influenza, group A streptococci, E.coli.

17- which of the following is seen most commonly as a complication of gastro


intestinal shigella infection.
a- A generalized erythematous rash.
b- Arthritis.
c- Encephalopathy.
d- endocarditis.
e- Pneumonia.

18-All of the following are common complications of measles except:


a- Otitis media.
b- Pneumonia.
c- Arthritis.
d- Laryngeotracheitis.
E- Encephalitis.

CVS & RH. Fever


1- all of the following could be manifestation of large VSD Except:
a- feeding difficulties.
b- Poor growth.
c- Cardiomegally.
d- Recurrent chest infection.
e- Absent femoral pulse.

2- Teratology fallot consists of all of the following Except:


a- pulmonary stenosis.
b- Ventricular septal defect.
c- Aortic coarctation
d- Rt. Ventricular hypertrophy.
e- over-riding aorta.

3- Which of the following is not characteristic of cyanotic heart disease:


a- Elevated blood pressure in arms.
b- Polycythemia.
c- Digital clubbing.
d- Dyspnea on exertion.
e- Limited exercise tolerance.

4- Clinical manifestations of rheumatic fever include all of the following Except:


a- fever, usually present in early stage.
b- Arthritis characteristically involves small joints and painless.
c- Carditis occurs in about 40% of patients.
d- Erythemia morginatum in the associated skin rash.
e- Chorea.

5- The white strawberry tongue is associated with:


a- Scarlet fever.
b- Varicella.
c- Rubella.
d- Rubeolla
e- Diphteria.

6- Each of the following may be a common manifestations of mumps Except:


a- orchilis.
b- Pancreatitis.
c- Parotitis.
d- Bilateral arthritis.
e- Asception meningitis.

7- Teratology of fallot is characterized by all Except:

a- obstruction of RH. Out flow.


b- VSD.
c- Dextroposition of Aorta.
d- Rt. Ventricular hypertrophy.
e- Osticm secundum defect.

8- All of the following are regarded as major manifestations of rheumatic fever


Except of:
a- Carditis.
b- Prolonged P-R interval.
c- Erythema marginatum.
d- Subcutaneous nodules.
e- Chorea.
9- Which are of the following finding is commonly seen in patient with Rt. to left
shunt:
a- cyanosis.
b- Clubbing of the fingers and toes.
c- Secondary polycythemia.
d- All of the above.
e- Non of the above.

10- Which of the following is not a clinical feature or recognized complication of


Tetralogy of fallot:
a- Anoxic Spells.
b- Brain abscess.
c- Congestive heart failure.
d- Cyanosis.
e- Poor growth.

11- A child who skips, name four color and dresses and undresses is how old:
a- 15 months.
b- 24 months.
c- 30 months.
d- 18 months.
e- 60 months.(3y 3 color-4y 8color)

12- The ability to manipulate a small objects with the pincer grasp is usually
noted at what age?
a- 0-2 months.
b- 3-5 months.
c- 6-7 months.‫األقرب إلى الصحيح‬
d- 8-9 months.
e- 10-12 months.

13- A developmentally normal child who is just able to sit without support,
transfer objects from hand to hand, and speak in monosyllabic babble, is
probably what age:
a- 3 months.
b- 4 months.
c- 9 months.
d- 6 months.‫األقرب إلى الصحيح‬
e- 11 months.

14- Neurological, intellectual, and physical development in infants and children


occur in an orderly and sequential manner. Al the following are integers of
developmental milestones Except:
a- gross motor.
b- Fine motor.
c- Bone age.
d- Language.
e- Social.

15- Child at 12 months of age can do all of the following Except:


a- Say baba and mama.
b- Can walk with or without support.
c- Copy circle. 3y
d- Move bye-bye.
e- Play simple ball game.

16- A 4 month old infant can:


a- hold her head reasonably steady in sitting position.
b- Site a lone without support. 6m
c- Say mama and dada but in discriminately.9m
d- Say mama and dada appropriately.
e- Point to object that she wants.

17- A6 month old infant differs from a 3 months old in regard to his or her ability
to:
a- control bowel and bladder.
b- Crawl
c- Sit or almost sit, without support.
d- Simile socially.
e- Walk holding an to furniture.

18- Most infant loss weight immediately after birth, normal term infants
generally regain their birth, weight by:
a- 24 hours.
b- 48 hours.
c- 72 hours.
d- 3-5 days. (3-4 days loss weight )
e- 7-10 days. (1 week – 10 days regain their birth, weight )

19- At birth, the skull "Cranium":


a- Is larger than the face.
b- Is smaller than the face.
c- Cranium and face are equal in size.
d- May be larger or smaller than the face.
e- Is usually flattened anteriorly.
20- A weight gain of about 1.2kg during the first 3 months of life is:
a- About average.
b- Above average but within normal limits.
c- Below average but within normal limits.
d- Below average and probably abnormal.
e- Above average and probably abnormal.

21- The anterior fontanel usually feels closed on physical examination "palpation":
a- by 3 months.
b- Between 3 and 9 months.
c- Between 9 and 18 months.
d- Between 18 and 24 months.
e- Between 24 and 36 months.

22- During the first years of life on infant who weighted (3.4kg) at birth ordinarily
would gain about:
a- 2.3kg.
b- 4.5kg.
c- 6.8kg.
d- 9kg.
e- 11.4kg.

23- During the second year of life, the average weight gain is about:
a- 1.5kg.
b- 3kg. (2kg)
c- 5kg.
d- 8kg.
e- 12kg.

24- During the first year of life, the average gain in body length is about:
a- 12.7cm.
b- 25cm
c- 38cm
d- 51cm
e- 64cm.

25- The normal average of hemoglobin concentration of I year of age is about:


a- 17mg/dl.
b- 15mg/dl
c- 12mg/dl (12.5mg/dl)
d- 10mg/dl
e- 8mg/dl
26- The average head circumference of a term infant at birth is about:
a- 25cm
b- 30cm
c- 35cm
d- 40cm
e- 50cm

27- During the first month of life, head circumference grows about:
a- ½ cm
b- 1.2cm
c- 2.5cm
d- 5cm
e- 7.5cm

28- The average growth in head circumference during the first year of life is about:
a- 4cm
b- 12cm
c- 25cm
d- 37cm
e- 50cm

29- During the second year of life the average gain in body length is about:
a- 12 to 15cm.
b- 20 to 25cm.
c- 30 to 40cm.
d- 40 to 50cm.
e- Over 50cm.

30- One year old child would be able to do all of the following Except:
a- build a tower of three cubes.
b- Grasp a pellet
c- Reach for an object
d- Stand a lone
e- Transferee an object from hand to hand.

31- By age of two years the child can speak:

a- 3 word sentences.
b- Four word sentences.
c- Five word sentences.
d- Pronoun I and you.
e- Six word sentences.
32- A child can hop-on-and foot by age of:
a- one year.
b- Two year.
c- One year and half.
d- Two years and half.
e- Four years.

33- A child triple their birth weight by age of:


a- one year.
b- Two year.
c- Sex months.
d- Three months.
e- Nine months.

Rickets
1- Craniotabes could be seen in all of the following condition Except:
a- Rickets.
b- Vitamin E deficiency.
c- Hydrocephalus.
d- Syphilis.
e- Premature baby.

2- Physical features of vitamin D-deficient rickets included all of the following


Except:
a- Bitot Spots.
b- Craniotabes.
c- Enlagement of the costochondral junction.
d- Thickening of the ankles and wrists.
e- Large anterin fontranell.
f- Bowed legs.

3- All of the following are true about the clinical manifestations of kwashiorkor
Except:
a- the presence of edema.
b- Rash in sun exposed area.
c- Hair changes.
d- Weak muscles.
e- An increased susceptibility to infection.

4- The recommended daily dietary allowance of vitamin D per young infant is:
a- 100 I.U
b- 400 I.U
c- 800 I.U
d- 1600 I.U.
e- 4000 I.U.

5- Which of the following Vitamin are Fat soluble:


a- A, D and C.
b- D, K and B1
c- A, D, K and E.
d- C, D, K and E
e- B1, K and E.

6- Supplementation with which of the following Vitamins is most important for an


exclusively breast feed infant:
a- Vitamin A
b- Vitamin E
c- Vitamin C
d- Vitamin B1
e- Vitamin D

7- All of the following are clinical signs of rickets Except:


a- Craniotabes.
b- Enlargement of the costochordral junction.
c- Thickening of wrists and ankles.
d- Poor growth.
e- Conjunctivitis.

8- Which of the following statement concerning Vitamin D metabolism is false:


a- it is Activated to cholecalciferol in the skin.
b- It requires bile for absorption.
c- The kidney is active in its metabolism.
d- It circulates in plasma as 25OH cholecalciferol.
e- It is stored in the liver but not metabolized there.

9- In the absence of Vitamin D, Serum calcium may be maintained by:


a- Parathyroid hormone secretion.
b- Decreased renal excretion of phosphate.
c- Small dietary increases.
d- Decreased renal excretion of alkali.
e- Increased amounts of Vitamin A in diet.

10- Clinical disorders associated with increased incidence of Vitamin D deficiency


include all of the following Except:
a- cystic fibrosis.
b- Hepatic disease.
c- Celiac disease.
d- Chronic anticonvulsant therapy.
e- Obesity.

11- Rickets may be treated by all of the following Except:


a- 1500 to 5000 I.U of Vitamin D daily for 2 to 4 weeks.
b- Sun light plus 1500-5000 I.U of Vitamin D daily until heading
demonstrated on x-ray.
c- Sun lights.
d- 6000 I.U of Vitamin D as single dose.
e- ↑Ca+2 in diet and ↓phosphate.

12- In infant rickets all of the following is correct Except:


a- craniotabes is an early finding.
b- Serum calcium is usually normal.
c- Muscle are a atrophied.
d- Low serum phosphoric.
e- Rosary.

13- All of the following are regarding Vitamin D Except:


a- the required prophylactic dose is 400I.U one day.
b- Vitamin D is metabolized in liver and kidney.
c- Causes muscle weakness when deficient.
d- Causes limb pain when deficient.
e- Excessive dose is safe.

14- a 4 month old with Vitamin D-deficient rickets. Would be expected to show all of
the following Except:
a- Craniotabes.
b- Bow Legs. (‫ شهور‬4 ‫)عمره‬
c- Rosary.
d- Low Serum phosphate level.
e- High Alkaline phosphatase level

Nephrology
1- All of the following are consistent with the diagnosis of idiopathic nephrotic
syndrome of child hood Except:
a- Onset is usually between 2 and 6 years of age.
b- Pathologic renal changes are minimal by light microscopy.
c- C3 level is depressed.
d- Hypertension is unusual.
e- Hyperlipidemea.

2- In nephrotic syndrome all of the following is correct Except:


a- Edema is massive.
b- Ascites may be present.
c- Total serum globulin is diminished.
d- massive proteinuria.
e- Steroid are the drug of choice for treated.

3- In acute post streptococcal nephritis all of the following is correct Except:


a- Smoky urine.
b- Usually present with massive edema.
c- Specific gravity of urine is high.
d- Hypertension.
e- Penicillin may be of value in patient management.

4- Concerning urinary tract infection in children all of the following is correct


Except:
a- Usually diagnosed if you find baeilliuria of 10000/mm2 or more.
b- Initial symptoms may be systemic (fever, abdominal pain).
c- Treatment should be continued for at least 2 week.
d- Found smelling is not a common presenting complaint.
e- Common causative organism are gran –ve organism.

5- All of the following statement are correct about routine urine examination,
Except:
a- Specific gravity of 1015 is normal.
b- Red blood cell cast is normal finding.
c- One red blood cell is normal finding.
d- One white blood cell is normal finding.
e- Negative sugar is normal finding.

6- Of the following, the most reliable for the diagnosis of urinary tract infection:
a- Fever and loin pain.
b- A numerous WBCs in the urine analysis.
c- Bacteria seen in the urine analysis.
d- The presence of a single isolated of < 105 /ml colony in urine.
e- Dysuria and frequency.
7- A child with chronic renal failure is expected to develop all of the following
Except:
a- Abnormal linear growth.
b- Hypophosphatemia.
c- Hypertension.
d- Hyperkalemia.
e- Rickets.
8- In minimal lesion nephrotic syndrome all are true Except:
a- There is proteinuria without gross hematuria.
b- Serum BUN and Creatinine are normal.
c- Ascites could be present.(pu etre)
d- Hypertension is constant finding.
e- Hepatomegally may be present.

9- Nephrotic syndrome is characterized by:


a- Proteinuria.
b- Hypoproteinemia.
c- Edema.
d- Hyperlipidemia.
e- All of the above.

10- Minimal change nephrotic syndrome is characterized by the following Except:


a- Edema.
b- Albuminurea.
c- Hypercholestrolemia.
d- Hypoproteinemia.
e- Hypertension.

11- Classical acute nephritis in children, characterized by all of following Except:


a- Hypertension.
b- Macroscopic hematuria.
c- Follows streptococcal infection.
d- Necessitates fluid restriction.
e- Corticosteroid are the first line of therapy.

12- Proteinuria is usually found in all of the following Except:


a- Nephrotic syndrome.
b- Iron deficiency anemia.
c- Acute glomonal nephritis.
d- Renal tumor.
e- Orthostatic.

13- The commonest cause of nephritis syndrome in child (3) years old is:
a- Idiopathic.
b- B-streptococcal group A infection.
c- System lupus erythematous.
d- Insulin dependent diabetic mellitus.
e- Urinary tract infection.

14- which are of the following statements regarding hematuria is not


true
a- If casts are present , the source of hematuria must be the kidney.
b- Bright red urine that clots usually suggests renal, or upper
urinary tract source of bleeding.
c- The addition finding of proteinuria usually suggests a renal
source.
d- The most common neoplasm associated with hematuria is
wilm's tumor.
e- It can be a cause of anemia.

15- Which of the following statement is true of post streptococcal glomerulonephortic:


a- More than 10% of children develop chronic renal failure.
b- Hypertensive encephalopathy is recognized complication
c- ASO Titer is not useful marker of streptococcal infection.
d- Life long penicillin prophylactic is recommended.
e- Abnormalities of serum complement usually persist for more than 3
months.

16- Post streptococcal acute glamerulonephritis is associated with all of the following
Except:
a- oliguriea..
b- fall in C3 level.
c- Granular and red cell cast.
d- Smoky urine.
e- Polyurea.

Respiratory System

2- Infections with mycoplasma pneumonia can produce each of the following


EXCEPT:
a- Guillain – barré syndrome .
b- Lobar and bronchopneumonia.
c- Tracheobronclitis.
d- Otitis media .
e- Glomerlonephritis.
3- The drug of choice for treatment of mycoplasma pneumonia infection in 4 years old
child is :
a- Cefuroxime .
b- Chlorayphenicol.
c- Erythromycin .
d- Penicillin.
e- Tetracycline .

4- All of the following are not associated with acute Bronchiolitis EXCEPT: .
a- Usually associated with High fever and rashes . low grade
b- Usually associated with bilateral infiltrates on chest tray.
c- Commonly associated with retractions, tachypena and wheezing .
d- Characterized by the absence of cough despite respiratory distress
e- Most common between 2 and 5 years of age . ( < 2 yrs )

5- Acute epiglottises is not associated with all of the following EXCEPT:


a- Gradual onset of cough , fever and stridor over several days. Acute onset
b- Infection with Para influenza virus . Hib
c- Hemophilia influenza type B septicemia .
d- Easy in swallowing .
e- High probability of recurrence .

6- All of the following statements are true of acute laryngeo tracheobronclitis


(group)EXCEPT:
a- Barking cough and hoarse voice are early symptoms .
b- Boys are more likely to be admitted to hospitals than girls
c- The symptoms frequently respond to salbutamol .
d- The use of nebulized adrenaline is indicated.
e- Para influenza virus are the predominant etiological agents .

7- Empyema is associated with all of the following Pathogens Except:


a- Staphylococcus aureus.
b- H. Influenza type B.
c- Group A streptococcus.
d- Pneumococcus.
e- Mycoplasma.

8- Viral Bronchiolitis is characterized by all of the Following except:


a- Decreased in functional residual capacity.
b- Air trapping on the chest X- RAY.
c- Hypoxia.
d- Inflammation and partial occlusion of bronchioles.
e- Wheezing and tachypnea.

9- Characteristic features of pneumococcal pneumonia includes All of the following


Except:
a- Meningism.
b- Delirium.
c- Petecheal rash.
d- Hypochondral pain.
e- Fever.

10- Persistent stridor is caused by all of the following Except:


a- Vocal cord paralysis.
b- Laryngeomalacia.
c- Tracheal hemangioma.
d- Tracheomalacia.
e- Vitamine D deficiency.

11- The following are related to asthma Except:


a- IS more prevalent in boys.
b- Is more strongly associated with house dust mite than any Any other identified
allergen.
c- Nocturnal cough.
d- Excercised induced wheeze.
e- Pectus exacavatum ( funnel chest ).

12- The following infectious agents are generally recognized as causing pneumonia
Except:
a- Influenza viurus.
b- EcHo virus.
c- Mycoplasma pneumonia.
d- Streptococcus pneumonia.
e- Chlamydia trachomitis.

13- The following steps are appropriate in the management of Status asthmaticus
Except:
a- Humidified oxygen.
b- Prescription of corticosteroid therapy.
c- Prescription of inhaled beta adrenergic agonist.
d- Prescription of aminophylline intravenious.
e- Prescription of sedative to restless child.
14- In contrast to acute epiglottitis, acute laryngeo tracheo-Bronchitis has all of the
following features Except: .
a- A more insidious onset.
b- The etiologic agent is almost always viral.
c- A slower course.
d- The etiologic agent is almost always bacterial.
e- The patient is less likely to require intubation.

15- All of the following drugs could be used in controlling Acute attacks of bronchial
asthma Except:
a- Adrenaline.
b- Ketotifen (zaditin).
c- Ephidren sulfate.
d- Aminophylline.
e- Salbutamol inhaler.

16- The virus most commonly associated with the clinical Picture of acute Bronchiolitis
in infants and children is
a- Adeno virus.
b- Respiratory syncytial virus.
c- Rhino virus.
d- Coxsackie virus.
e- Herpes virus type 2.

16- In pneumonia consolidation all of the following is correct except:


a- there is Impaired percussion note.
b- Vocal resonanace is increased.
c- Type of breathing is harsh vesicular.
d- Mediastinum is central.
e- Chest pain may be a presenting symptom.

17- All of the following are not related to acute bronchiolitits except
a- Commonly caused by bacterial infections.
b- Associated with retractions, tachypnea and wheezing.
c- Physical examination reveals extensive signs of consolidation.
d- X-Ray reveals pneumatoceles.
e- Best treatment is broncho-dilator.

18- In staphylococcal pneumonia all of the following is correct except


a- Occurs more frequently in infant.
b- Empyema and pneumothorax are commonly seen .
c- No respond well to crystalline penicullive .
d- Therapy is usually required for one week only.
e- X-Ray reveals pneumatoceles.

19- In pleural effusion the following signs are elicited except.


a- Impaired percussion note.
b- Shifted Mediastinum .
c- Diminished breathing on the affected side.
d- Respiratory movements on the affected side are increase.
e- Respiratory movements on the affected side are decreased.

20- Physical signs of pulmonary collapse include all of the following except
a- Impaired percussion note.
b- Diminished breath sounds.
c- Shifted mediastinum.
d- Increased tactile vocal fremitus.
e- Decreased tactile vocal fremitus.

21- Stridor in the newborn may be caused by all of the following except .
a- Congenital Goiter .
b- Vescular anomalies are rings.
c- Larynogo malacia .
d- Aspiration pneumonia .
e- Hypocalcemia.

22- All of the following are characteristic of Jaundice associated with breast feeding
except.
a- Significant elevations of unconjugated bilirubin .
b- A rapid fall in serum bilirubin after discontinuation of nursing.
c- Nursing can be resumed after several days without return of hyperbilirubinemia.
d- Significant elevations of conjugated bilirubin.
e- Kernicterus has never been reported to occur as result of breast milk jaundice.

34- Which is most likely to occur as an isolated manifestation of acute Rheumatic fever:
a- Arthritis.
b- Carditis
c- Chorea.
d- Erythemia Marginatum
e- Fever.

35- The most common roentgeno-graphic abnormality in a child with asthma is:
a- Bronchiectasis.
b- Generalized hyperinflation.
c- Lower lobe Infiltration.
d- Pneumomediastinum.
e- Right middle lobe atcleclosis.

36- Congenital Hypothyrodism should be included in the differential diagnosis of


newborn with:
a- Coma.
b- Prolonged Jaundice.
c- Pulmonary edema.
d- Renal failure.
e- Sever anemia.

37- Ordinarily, the first dose of live, attenuated measles vaccine (as MMR) should be
administered:
a- At about 3 months of age.
b- At 6 to 9 months of age.
c- At 15 months of age. (Palestine >> 12 m)
d- At about 24 months of age.
e- At the time of school entry.

38- A side effect of tetracycline that is limited to children is:


a- pseudotumor cerebri.
b- Hyperglycemia.
c- Tremors and convulsions.
d- Permanent discoloration of teeth. (les dents)
e- Hematuria and proteinuria.

39- The arthritis of acute rheumatic fever usually:


a- Is monoarticular.
b- Heals without deformity.
c- Appears after the fever subsides.
d- Is seen only in patients with concurrent Carditis.
e- Involves large and small joint equally.

40- The most common presentation of pinworm infection is:


a- Appendicitis.
b- Diarrhea.
c- Intussusceptions.
d- Perianal pruritis.
e- Vaginitis.

41- All of the following are features of trisomy 21 (down):expt


a- hypotonia.
b- Microcephaly.
c- Simian crease on palms.
d- Recurrent chest infection.
e- Normal development.

42- Causes of failure to thrive include all of the following Except:


a- Emotional deprivation.
b- Intestinal mal-absorption.
c- Renal tubular disorders.
d- Chronic heart failure.
e- Acute chest infection.

43- Pneumococcal infection are more frequent and more sever in all of the following
condition Except:
a- Immunodeficiency.
b- Post splenectomy.
c- sickle cell disease.
d- Congenital heart disease.
e- Formula fed infants.

44- All of the following are signs of lower motor neuron disease Except::
a- Fibrillation.
b- Fasciculation.
c- Spasticity.
d- Muscle wasting.
e- Hyporeflexea.

45- All of the following are sings of congenital hypothyrodism Except:


a- Prolonged jaundice.
b- Hypertonic muscle tone.
c- Constipation.
d- Umbilical hernia.
e- Dry skin and hoarse cry.

46- Convulsions in the newborn could be caused by all of the following Except:
a- Hypokalemia.
b- Hypoglycemia.
c- Meningitis.
d- intracranial hemorrhage.
e- Hypoxia.

47- Acute glomerulonephrities may be associated with which are of the following:
a- Glucosuria.
b- Elevated serum cholesterol.
c- Normal or elevated C3 complement.
d- Encephalopathy.
e- Hypotension.

48- Pallor, jaundice, hemosiderosis and extrmedallary hematopoiesis are most


chareteristic of which of the following:
a- Sickle cell
b- Thalasemia Major.
c- Iron deficiency anemia.
d- Hemoglobin- H disease.
e- Hemoglobin- C disease.

49- Kwashiorkor is characterized by each of the following Except:


a- A history of inadequate protein intake with relatively adequate calories.
b- A history of prolonged breast feeding.
c- Edema.
d- Hypoalbuminemia.
e- Marked vulnerability to death from inter-current enteric infection.

50- Manifestation of Cretinism include all of the following Except:


a- Puffiness of face.
b- Mental retardalion.
c- Diarrhea.
d- Prolonged physiological jaundice.
e- Dry skin.

51- Dehydration is classified into isotonic, hypotonia, hypertonic according to are of the
following:
a- Serum potassium.
b- Serum sodium level.
c- PH of Blood.
d- Blood Glucose.
e- Serum Calcium.

52- All of the following are common in bronchial asthma Except:


a- Generalized wheezing.
b- Increased eosinophil’s count.
c- Paroxysmal nature of the attacks paroxysmal.
d- Gross hemoptysis.
e- Dyspnea.

53- A child which "chicken pox" is liable to all Except:


a- Lymphadenitis.
b- Encephalitis.
c- Orchitis.
d- Bronchitis.
e- Gastroenteritis.

54- Kwashiorkor is characterized by all of the following Except:


a- Mental apathy.
b- Hypervitaminosis A.
c- Flag sign.
d- Skin manifestation.
e- Edema.

55- An infant of diabetic mother is liable to all of the following Except:


a- Hyperglycemia.
b- Hypocalemia.
c- Respiratory distress.
d- Birth trauma.
e- Hyperbilirubinemia.

56- Arthritis in childhood occurs in all of the following Except:


a- Acute Rheumatic fever.
b- Sickle cell disease.
c- Acute leukemia.
d- Hemolytic Uremia syndrome.
e- Rheumatoid arthritis.

57- In Rheumatic fever all of the following is correct Except:


a- Is associated with painful arthritis.
b- Is associated with prolonged P-R interval.
c- Dose not recur.
d- Is frequently the result of streptococcal throat infection.
e- Has a peak incidence between 5-15 years.

58- The following statement regarding mumps are correct except:


a- Infection is spread by droplet infection.
b- Incubation period is 16-18 days.
c- The Virus can be isolated from Urine.
d- Epididymo-orthitis is commoner in young boys.
e- Parotid duct is red and pointed.

59- In Rickets all of the following is correct except:


a- Craniotabes is a late sign.
b- Delay closure of fontanel occurs.
c- There may be normal calcium level.
d- There is characteristic cupping of the distal ends of the radus and ulna.
e- High level of Alkaline Phosphates enzyme.

60- Characteristics of henoch-schonlein purpura include all of the following except:


a- A low platelet count.
b- Skin lesions which initially blanch on pressure.
c- Normal complement level.
d- Normal thrombin time and partial thromboplastin time.
e- Skin biopsy help in diagnosis.

61- In Patent ductus arteriosus all of the following is correct except:


a- Usually closes functionally with in 10-15 hours of birth.
b- That is larg may result in Eisenmenger syndrome.
c- Closes following a prostaglandin Infusion.
d- Persists in pulmonary atresia due to arterial hypoxemia.
e- That is small produces a continuous murmur in the pulmonary area below
the left clavicle.

62- Bloody stools are a recognized clinical feature in children affected by all of the
following except:
a- Vero toxin producing strains of Escherichocoli.
b- Giardia lamblea.
c- Cow's milk protein intolerance.
d- Ascaris lumbriocoides.
e- Shigelliosis.

63- The following is live attenuated vaccine except:


a- Measles, Mumps and rubella. (MMR)
b- Poliomyelitis "Sabin".
c- Rubella.
d- B.C.G.
e- Pertussuss.

64- The most common ophthalmologic complication in children with juvenile arthritis is:
a- Cataract.
b- Ptosis.
c- Glaucoma.
d- Corneal ulceration.
e- Iridocyclitis.

65- Concerning hypovitaminosis, all of the following statement are correct except:
a- Vitamin A deficiency lead to blindness.
b- Vitamin B "Thiamin" deficiency leads to peripheral neuropathy.
c- Vitamin C deficiency leads to bleeding.
d- Vitamin K deficiency leads to bleeding.
e- Vitamin B12 deficiency leads to Microcytic anemia.

66- Bone age is best evaluated by doing X-ray of the following:


a- Elbow Joint.
b- Wrist.
c- Ankle.
d- Knee Joint.
e- Hip Joint.

67- All of the following are advantages of breast feeding except:


a- Reduced incidence of allergy.
b- Reduced incidence of otitis media.
c- Reduced incidence of colic.
d- Increased psychologic comfort.
e- Vitamin K content.

68- Clinical features of Vitamin E deficiency include all of the following except:
a- Cereballer ataxia.
b- Muscle weakness.
c- Peripheral neuropathy.
d- Hemolysis.
e- Hepatosplenomegally.

69- Teething is associated with all of the following except:


a- drooling of saliva.
b- Gingival swelling.
c- Irritability.
d- Rash.
e- Relief with acetominophin.

70- Manifestation of hyperkalemia include all of the following except:


a- Paraesthesia.
b- Weakness.
c- Paralysis.
d- Wide QRS complex.
e- Tetany.

71- Congenital infections may be associated with all of the following Except:
a- Growth retardation in utero.
b- Polycythemia.
c- Thrombocytopenia.
d- Hepatosplenomegaly.
e- Skin Rash.

72- At what age meningitis cannot be excluded with confidence in children presenting
with febrile convulsion:
a- Fiver years.
b- Four years.
c- Three years.
d- Nine months.
e- all age.

73- Patients with sever cerebral palsy usually die because of:
a- Urinary tract infections.
b- Chest infections.
c- Meningitis.
d- Heart failure.
e- Hepatic failure.

74- All of the following are complication of chronic renal failure except:
a- Hyperkalemia.
b- Alkalosis.
c- Anemia.
d- Hypertension.
e- Renal Osteodystrophy.

75- Which of the following types of vaccines is not efficient in administered below the
age of two years:
a- Injectable polio vaccine (Salk).
b- Measles vaccine.
c- Hepatitis B vaccine.
d- Pneumo vax.
e- Hemophyllus Type B conjugated vaccine.

76- In general all of the following vaccine must avoided in child with immunodeficiency
except:
a- Measles vaccine.
b- Mumps vaccine.
c- Oral polio vaccine (Sabin).
d- Influenza Vaccine.
e- Pertasiss vaccine.

77- All of the following are true about the laboratory manifestation of kwashiorkor
except:
a- Persistent ketonuria.
b- Hypoalbuminemia.
c- Hypoglycemia.
d- Potassium deficiency.
e- Low serum amylase levels.

78- Which of the following infection typically has an incubation period of less than 2
weeks:
a- Mumps. 12-28
b- Varicella. 14-16
c- Rubella. 12-21
d- Measles . 7-14
e- Rabies.

79- When the mother contact german Measles "Rubella" early in pregnancy, effects on
the fetus may include all of the following except:
a- Thrombocytosis.
b- Cataract.
c- Glaucoma.
d- PDA.
e- Hepatosplenomegally.

80- Which of the following is contraindication to L.P. in a 9 months old infant with
suspected meningitis:
a- Uncontrolled bleeding diathesis.
b- Bulging fontanel.
c- L.P. 2 days before.
d- Marked in cooperativeness on the part of patient.
e- Significantly elevated WBCs consistent with bacteremia.

81- Hypokalemia is likely to occur in all of the following except:


a- Diarrhea.
b- Renal tubular acidosis.
c- Infantile Pyloric stenosis.
d- Diuretic therapy.
e- Infants of diabetic mother.

82- Which of the following is characteristic in Guillan-Barre Syndrome:


a- Female are affected more often than male.
b- Sensory symptoms are more striking than paralysis.
c- Muscles wasting “unusual”.
d- No cranial nerve involvement.
e- Sinus tachycardia is common in acute phase.
83- Signs of dehydration in acute gastro-enteritis include all of the following except:
a- Sunken eyes.
b- Oliguria.
c- Dry tongue and Mucus Membrane.
d- Bulging anterior fontanel.
e- Loss of skin elasticity.

Pediatric M.C.Q’s Review


DR/NABIL M.D.AL BARQOUNI
Assistant professor in IUG College of Medicine
A TRUE statement about the epidemiology of measles is that
A. In countries with no immunization programs, the peak age of infection is infancy
B. In GAZA immunization programs have reduced the incidence of infection by 80%
C. Usually spread by direct contact with infectious droplets
D. Patients become contagious when the rash appears
E. Incubation period is 4-5 days from exposure to onset of symptoms

Recognised complications of measles include all except :


A. Encephalitis.
B. Fébrile convulsions.
C. Diarrhoea.
D. Infertility in males.
E. Corneal ulceration.

All the following are recognized complications of chickenpox EXCEPT :


(A) Reyes syndrome
(B) Encephalitis
(C) Pneumonia
(D) Hemorrhagic vesicles
(E) Orchitis

Toxic shock syndrome (TSS) is associated with all of the following findings
EXCEPT
(A) Vomiting and diarrhea
(B) Disorientation
(C) Increased platelet count
(D) Elevated liver function tests
(El Elevated blood urea nitrogen

A previously healthy 2 year old is brought to ED because her mom is unable to


awaken her for 45 minutes. She has not been ill. PE reveals an afebrile, hypotonic
child who withdraws her hand from painful stimuli but does not spontaneously open
her eyes. RR is 36/min, BP is 92/64. What is the next best thing to obtain?
A. BUN
B. CXR
C. EKG
D. EEG
E. Toxicology screen

The MOST common site of Pseudomonas colonization in ICU is:


A. Central venous catheters
B. Foley catheters
C. Peripheral intravenous catheters
D. Surgical wounds
E. Tracheostomy tubes

A 6 year old boy with severe factor VIII deficiency hemophilia develops increased
swelling of the right distal forearm. There is not history of trauma. The peripheral
circulation is normal and there is no joint involvement. The MOST important
complication of bleeding in this location is:
A. Blood loss
B. Muscular Damage
C. Neurologic Impairment
D. Tendon Shortening
E. Vascular Damage

The MOST common cause of acute laryngotracheobronchitis in children that also


causes many cases of bronchiolitis and pneumonia among young infants is:
A. Adenovirus
B. Influenza virus
C. Parainfluenza virus
D. Respiratory syncytial virus
E. Rhinovirus

Which of the following laboratory findings is unusual in patients with simple


( nutritional ) rickets ?
(A) Aminoaciduria
(B) Hyperphosphaturia
(C) Elevated levels of serum alkaline phosphatase
(D) Hypercalciuria
(E) Hypophosphatemia

An ambulance team brings a 6 year old girl to the ED. She had been unrestrained in
the back seat. The girl is unresponsive on arrival and is bleeding profusely from a
scalp wound. Her Glasgow Coma Scale is 3.
What is the BEST initial step in evaluation and management?
A. Control profuse scalp bleeding
B. Establish IV access
C. Order portable cervical spine radiographs
D. Remove all clothing
E. Secure an adequate airway

Which of the following sets of blood gas values is most compatible with acute
aspirine poisoning in a 16 month old child:
a-PH 7.60, pco2 40 mmHg, Hco3 40 mmol/L
b-PH 7.50, pco2 40 mmHg, Hco3 30 mmol/L
c-PH 7.25, pco2 20 mmHg, Hco3 8 mmol/L
d-PH 7.20, pco2 45 mmHg, Hco3 20 mmol/L
e-PH 7.00, pco2 35 mmHg, Hco3 8 mmol/L

Hirschsprung's disease:
A. Is confined to the rectum and sigmoid colon
B. Often presents with large hard stool
C. Abdominal distension and vomiting relieved by rectal stimulation and
explosive release of watery stool
D. Usually associated with soiling
E. Barium study is the most reliable for diagnosis

Accepted indication for the removal of adenoids in childhood is:


a. Mouth breathing
b. Serious post nasal obstruction
c. Nasal escape of air in speech
d. Inspiratory snoring
e. Recurrent otitis media

A 2 week old presents with tachypnea, poor perfusion, gallop rhythm, diminished
pulses, and hepatomegaly. ABG shows metabolic acidosis. Echocardiography
reveals critical Aortic Stenosis.
What intervention is most likely to stabilize the infant’s condition?
A. Dobutamine
B. Epinephrine
C. Nitric Oxide
D. 100% oxygen
E. Prostaglandin E-1

A male infant born at 36 weeks gestation had a left testicle palpable in the inguinal
canal. At 12 months of age, the left testicle has failed to descend into the scrotum.
What is most appropriate care for this infant?
A. Observation until 2 years of age
B. Orchiopexy
C. Radionuclide scan of left testicle
D. Treatment with human chorionic gonadotropin
E. Treatment with testosterone

Intussuception in childhood:
A. Has as the earliest sign the passage of red current jelly stools
B. Has a peak incidence in the first 3 months of life
C. Requires operative reduction in the majority of cases
D. May be initiated by meckel's diverticulum
E. It has association with umbilical hernia

Necrotizing enterocolitis is associated with all of the following except:


A. Congenital heart disease
B. Prematurity with respiratory distress syndrome
C. Maternal diabetes mellitus
D. Breast feeding
E. Polycythaemia

Medical indication for circumcision in children include:


A. Hypospadias
B. 3 month infantile colic
C. Recurrent ballanitis
D. Ambiguous genitalla
E. Cryptorchidism

In congenital diaphragmatic hernia:


A. Hernias most commonly occur on the left
B. Persistent fetal circulation occurs uncommonly
C. Associated congenital anomalies are common
D. Most present between 12 and 24 hours of age
E. Pulmonary hypoplasia is rarely the cause of death

The average time of closure of the anterior fontanel is:


A. 12 months
B. 14 months
C. 16 months
D. 18 months
E. 22 months

A newborn female has an open neural tube defect, low set ears, VSD, and rib and
vertebral column malformations.
Which of the following MATERNAL conditions was most likely present during
pregnancy?
A. Alcoholism
B. Diabetes mellitus
C. Hypothyroidism
D. Iodine deficiency
E. Syphilis

The arthritis of acute rheumatic fever usually:


A. Is monoarticular
B. Heals without deformity
C. Appears after the fever subsides
D. Is seen only in patients with concurrent carditis
E. Involves large and small joints equally

Which of the following support the diagnosis innocent murmur:


A. Present only in diastole
B. Fixed splitting of second heart sound
C. Is loud in high output states
D. Heard allover the praecordium
E. The venous hum is best heard at the apex

A 4 month old with vitamin D deficiency rickets would be expected to show all of the
following except:
A. Craniotabes
B. Bow legs
C. Rosary
D. Low serum phosphate level
E. High alkaline phosphate levels

The following are causes of jaundice EXCEPT:


A. Congenital hypothyroidism.
B. Wilson's disease.
C. Pityriasis rosea.
D. Alagille Syndrome.
E. Alpha-1-antitrypsin deficiency.

All of the following are true about kwashiorkor except:


A. The presence of edema
B. Rash in sun exposed areas
C. Hypoalbuminemia
D. Weak muscles
E. An increased susceptibility to infection

The following are single gene disorders Except:


A. Cystic fibrosis.
B. Galactosemia.
C. Duchenne muscular dystrophy.
D. Marfan syndrome.
E. Turner syndrome.

The risk of neonatal jaundice is increase by all except:


A. Prematurity.
B. Trisomy 21.
C. Elective caesarean section.
D. Congenital hypothyroidism.
E. Cephalohaematoma.

All of the following are recognized causes of hypertension in childhood except:


A. Chronic glomerulonephritis
B. Renal vein thrombosis
C. Neuroblastoma
D. Peripheral pulmonary artery stenosis
E. Congenital adrenal hyperplasia

A 5 year old male is hospitalized in January with fever and seizures. LP reveals
clear CSF with 47 WBCs/mm3 all of which are lymphocytes. On PE he appears
obtunded but arouses with painful stimuli. Neurologic exam reveals no focal
findings. Which diagnostic test is most likely to reveal this child’s illness?
A. Bacterial culture of CSF
B. PCR test of CSF for HSV
C. Strepto.pneumoniae bacterial antigen test of CSF
D. Viral culture of CSF
E. Viral culture of nasopharyngeal and rectal swabs

A 11 year old girl complains of dysuria and abdominal pain for 2 days. She denies
nausea, vomiting, flank pain and vaginal discharge, mild suprapubic tenderness,
and otherwise normal findings.
What is the most likely diagnosis?
A. Bacterial vaginosis
B. Candidal vulvovaginitis
C. Chlamydia urethritis
D. Pelvic inflammatory disease
E. Urinary tract infection

A 3 year old girl comes to the ER with temperature of 40.0o C and acute onset
diarrhea. Stool is guaiac positive with leukocytes. There is no history of foreign
travel and the child has not received antibiotics recently.
What is the most likely organism?
A. Clostridium difficile >> not received antibiotics
B. Giardia lamblia >> leukocytes
C. Rotavirus >> high fever + leukocytes + blood stool
D. Salmonella enteritidis
E. Vibrio cholerae >> not travel

You are evaluating a 4 week old boy for tearing of the right eye that has worsened
over the past week. Physical exam reveals slight tearing but no evidence of purulent
exudate or conjunctival erythema. All other findings are normal.
The MOST appropriate initial management is…
A. Administration of amoxicillin
B. Endoscopic dacrocystorhinostomy
C. Instillation of silver nitrate in the eyes
D. Observation with intermittent massage of the duct
E. Surgical dilation of the nasolacrimal duct

The following can be the cause of a non-blanching rash except:


A. Idiopathic thrombocytopenic purpura.
B. Roseola infantum.
C. Henoch-Schönlein purpura.
D. Haemolytic uraemic syndrome.
E. Meningococcal sepsis.

Regarding nocturnal enuresis all true except:


Is more common in girls.
Can be a presenting feature of D.M.
Can be a presenting feature of D.I.
Emotional disturbance can be a cause.
Desmopressin is used in the treatment.

In acute bronchiolitis:
A. Ribavirin is the treatment of choice for hospitalized cases
B. Lung volume is usually decreased
C. Bronchodilators are usually effective
D. Feeding difficulties are common
E. Upper air way obstruction is a common feature

The following congenital heart lesions are "Acyanotic" except:


A. Ventricular septal defect.
B. Atrial septal defect.
C. Patent ductus arteriosus.
D. Aortic stenosis
E. Transposition of the great arteries

Hemolytic uremic syndrome is characterized by all of the following except:


A. Microangiopathic hemolytic anaemia
B. Familial cases
C. Thrombocytopenia
D. E-coli (0157:H7) infection
E. All of the above

A previously health 1-year-old infant who weighs 10 kg presents to your office with
a fever of 39° C (102.2° F). Her mother is very concerned about the child’s intake
and asks for guidance regarding caloric requirements during this illness.
Of the following, the best estimate of the child’s caloric requirements at this time is
A. 500 kcal/d plus 500 kcal due to the fever
B. 1,000 kcal/d plus 250 kcal due to the fever
C. 1,500 kcal/d
D. 1,500 kcal/d plus 250 kcal due to the fever
E. 2,000 kcal/d

Skin peeling occurs in all of the following conditions except:


A. Burns.
B. Kawasaki disease.
C. Scarlet fever.
D. Toxic shock syndrome.
E. Erythema infectiosum (slapped cheek disease).

In neonatal RDS (respiratory distress syndrome) all are true except:


A. Surfactant is useful in the treatment.
B. It is common in infants below 28 weeks gestation.
C. Antenatal steroids are beneficial.
D. Maternal opiate abuse increases the risk.
E. Maternal diabetes increases the risk.

In children with cerebral palsy all are true except:


A. Associated with a degree of learning impairment.
B. Birth asphyxia is the most common cause. Most >> antenatal
C. Hand preference before 12 months can be a useful sign.
D. The spastic form is the most common clinical type.
E. Management should adopt a multidisciplinary approach.

Which of the following condition is one type of haemoglobinopathies:


A. G6PD deficiency.
B. von Willebrand disease.
C. Spherocytosis.
D. Pyruvate kinas deficiency .
E. Sickle cell disease.

All of theses neuromuscular disorders are "genetic diseases" except:


A. Duchenne muscular dystrophy.
B. Myotonic dystrophy.
C. Spinal muscular atrophy.
D. Guillain Barre syndrome.
E. Frierich's Ataxia

A 5-year-old boy is brought to the emergency department after having been struck
by an automobile. Physical examination reveals facial abrasions, abdominal
tenderness, and gross blood at the urethral meatus. Pelvic radiography reveals a
left-sided fracture of the superior pubic rami.
Of the following, the best procedure for INITIAL evaluation of the urinary tract in
this patient is
A. Bladder catheterization via the urethra
B. Computed tomography of the abdomen
C. Intravenous pyelography
D. Renal untrasonography
E. Retrograde urethrography

Potter phenotype may be due to:


A. Renal agenesis
B. Renal dysplasia
C. Obstructive uropathy
D. Severe amniotic fluid leak
E. All of the above

In pyloric stenosis all are true except:


A. Is more common in boys.
B. Surgery is the usual treatment.
C. Infants often have a metabolic alkalosis.
D. Constipation can be a presenting feature.
E. A barium meal is usually performed to confirm the diagnosis.
The following can cause failure to thrive except:
A. Celiac disease.
B. Cystic fibrosis.
C. Emotional neglect.
D. Gastro-oesophageal reflux.
E. Otitis media.

The following are causes of polyhydramnios except:


A. Maternal diabetes mellitus.
B. Potter syndrome.
C. Anencephaly.
D. Oesophageal atresia.
E. Spina bifida.

You are attending the emergency delivery by cesarean section of a primiparous


woman. The gestation was complicated by pregnancy-induced hypertension. Deep
variable fetal heart rate decelerations were noted during labor. At delivery, the
infant is acrocyanotic with poor tone; spontaneous movement and minimal
respiratory effort are present.
Of the following, your INITIAL management is to
A. Ascertain the heart rate and assign a 1-minute Apgar score
B. Begin tactile stimulation and provide blow-blow oxygen supplementation
C. Dry all skin surfaces and clear the oropharynx
D. Initiate bag-mask ventilation
E. Insert an umbilical catheter and administer naloxone

The following are inherited in an X-linked manner all are true except:
A. Duchenne muscular dystrophy.
B. Haemophilia A.
C. Cystic fibrosis.
D. G6PD deficiency.
E. Hunter syndrome.

Concerning chickenpox all are false except:


A. The incubation period is 7 days.
B. Aspirin is a useful anti-pyretic.
C. Transmission is usually by contact or airborne.
D. Encephalitis not a recognized complication.
E. There is no effective vaccine.

Regarding Kawasaki's disease:


A. Purulent Conjunctivitis is a recognized feature.>>not purulent
B. It is caused by a spirochaete organism. >>medium vasculitis
C. It is a cause of coronary artery aneurysms.
D. Fever is not a typical feature. >> fever typical
E. Is associated with profound thrombocytopenia >> anemia-leukopenia-thrombocytosis

A 1-year-old boy has been treated with a low-phenylalanine diet for the past year
after having been identified in infancy as having phenylketonuria. Despite
appropriate dietary restriction of phenylalanine, he has developed neurologic
symptoms. Of the following, this child is MOST likely to be deficient in
A. Biotin
B. Cobalamin
C. Carnitine
D. Tetrahydrobiopterin
E. Thiamine

A 22-month-old girls is nonverbal. She sat alone at 7 months and walked by 13


months, but now exhibits a wide-based stance, no longer ambulates, and will not
pick up or manipulate toys. Findings include: height and weight at the 50th
percentile; head circumference below the 5th percentile, with no increase over the
past 8 months; normal fundi, and no organomegaly.
Of the following, the MOST likely diagnosis is
A. Adrenoleukodystrophy
B. Cerebral palsy
C. GM2 gangliosidosis (Tay-Sachs disease)
D. Hypothroidism
E. Rett syndrome

Of the following, the MOST likely etiology of this decrease in hemoglobin is


A. Aplastic crisis
B. Folic acid deficiency
C. Hyperhemolytic crisis
D. Iron-deficiency anemia
E. Splenic sequestration

An 18-day-old term infant develops fever, lethargy, and focal seizures. Findings
include: an ill-appearing infant without exanthem, hepatomegaly, or jaundice.
Analysis of cerebrospinal fluid reveals white blood cells, 115/mm3; 45%
neutrophils; 55% lymphocytes; red blood cells, 40/mm3; glucose, 45 mg/dL; protien
200 mg/dL; and negative Gram stain.
In addition to ampicillin and cefotaxime, the MOST appropriate treatment to begin
at this time is
A. Acyclovir
B. Amphotericin B
C. Dexamethasone
D. Metronidazole
E. vancomycin

A 10-year-old girl has complained of intermittent left lower abdominal pain for 2
days. Previous evaluations, including a thorough physical examination, urinalysis,
and complete blood count, have not revealed the cause. Tonight she is complaining
of pain in the lower left abdomen that radiates into her left leg. There is no history
of fever, vomiting, or diarrhea.
The MOST likely cause of this girl’s pain is
A. Appendicitis
B. Intussusception
C. Malrotation of the intestine
D. Nephrolithiasis
E. Ovarian torsion

A 13-year-old boy has a congested, itchy, and runny nose accompanied by itchy
eyes. These symptoms usually occur in the fall, are unresponsive to over-the-
counter decongestants, and his father has the same problems. Physical examination
reveals pale, boggy nasal turbinates; clear nasal discharge; and dark circles under
his eyes. These findings are MOST consistent with
A. Allergic rhinitis
B. Infectious rhinitis
C. Nonallergic rhinitis with eosinophilia
D. Rhinitis medicamentosa
E. Vasomotor rhinitis

A 5-month-old child is brought to the emergency department by her mother because


she has been “crying a lot” for the past 24 hours. The mother denies any history of
fever, trauma, or illness. Physical examination reveals a lethargic toddler who is
very irritable when examined and who has mild tachycardia, scattered bruises over
the chest, and ecchymosis behind the left ear.
The MOST appropriate management is to
A. Administer intravenous naloxone
B. Administer a 20 mL/kg bolus of intravenous normal saline
C. Obtain an abdominal radiograph
D. Obtain a complete blood count and blood culture and administer intravenous
ceftriaxone
E. Obtain a computed tomographic scan of the head and a skeletal survey.

A 14-year-old boy who has allergic rhinitis reports that he frequently develops
coughing and wheezing after about minutes of playing soccer. These symptoms
improve after resting for 30 minutes. Of the following, the drug that will give the
BEST response in this patient if administered just prior to exercise is
A. Inhaled beta2-agonist
B. Inhaled corticosteroid
C. Oral beta2-agonist
D. Oral corticosteroid
E. Oral theophylline

A term infant is delivered vaginally to a healthy 24-year-old primigravida.


Immediately after birth, the infant’s respiratory effort is vigorous, but subcostal
retractions and cyanosis persist. The abdomen is scaphoid in appearance. Bag and
mask ventilation is initiated. Auscultation reveals decreased breath sounds on the
left and heart tones that are louder on the right.
The MOST likely explanation for these findings is
A. Congenital cystic adenomatoid malformation of the lung
B. Dextrocardia with situs solitus
C. Diaphragmatic hernia
D. Esophageal atresia with tracheoesophageal fistula
E. Pneumothorax

_______________________________________________________

‫البرقوني‬
1. A 16-month-old child has a diet that consists almost exclusively of breast milk. Which
of the following findings would be consistent with this dietary history?
(A) High serum blood urea nitrogen level
(B) Low weight for age
(C) High blood pressure
(D) Low serum iron level
(E) High serum calcium level

2. The most serious acute medical outcome of drowning is :


(A) pulmonary edema
(B) acute renal failure
(C) hypoxic brain injury
(D) blood loss
(E) cardiac arrhythmia
3. Which of the following injuries accounts for the greatest number of emergency
department visits?
(A) An injury caused by a motor vehicle accident
(B) A burn
(C) An injury caused by a fall
(D) Poisoning
(E) Drowning

4. Contraindications to induction of emesis in childhood poisoning include all of the


following EXCEPT:
(A) ingestion of turpentine
(B) rapidly increasing drowsiness
(C) ingestion of drain cleaner
(D) ingestion of acetaminophen
(E) ingestion by a child younger than 6 months

5. All of the following conditions may cause sensorineural hearing loss EXCEPT :
(A) rubella
(B) meningitis
(C) perinatal asphyxia
(D) otitis media
(E) aminoglycoside administration

6. A newborn is brought to the delivery room. After assessing the infant's heart rate,
color, and respiratory effort, a decision is made that the infant should receive
ventilatory assistance. After 30 seconds of this therapy, cardiac compressions are
initiated. The most likely Apgar
score at this point in time is:
(A) 9
(B) 7
(C) 6
(D) less than 5
(E) 8

7. A 5-year-old child has coarse facial


features, hepatosplenomegaly, and
progressive loss of developmental
milestones. In considering this clinical
picture, the most likely cause is :

(A) mucopolysaccharidoses
(B) carbohydrate metabolism disorder
(C) aminoaciduria
(D) urea cycle enzyme deficiency
(E) hereditary fructose intolerance

8. A 12-year-old girl is initially seen with all the classic clinical features of allergic
rhinitis, which include all of the following EXCEPT which?

(A) Dark shadows under the eyes


(B) Erythematous nasal mucosa
(C) Dental malocclusion
(D) Thin, watery, nasal secretions
(E) A transverse crease across the nose

9. A previously healthy 13-year-old boy has


a mild pneumonia characterized by
a nonproductive cough.

The most appropriate therapy is:


(A) cephalexin
(B) amoxicillin
(C) erythromycin
(D) penicillin
(E) trimethoprim-sulfamethoxazole

10. A 2-year-old child is noted to have an erythematous, bulging right tympanic


membrane. The two most likely bacterial
causes of this illness are :
(A) Streptococcus pyogenes and Staphylococcus aureus
(B) Haemophilus influenzae and Staphylococcus aureus
(C) Haemophilus influenzae and Streptococcus pneumnae.
(D) Streptococcus pneumoniae and Staphylococcus aureus
(E) Moraxella catarrhalis and Streptococcus Pyogenes

11. A 6-week-old infant has a history of noisy breathing. The noise was first noted
shortly after birth, is inspiratory in nature, is worse now that the infant has a
viral respiratory illness, and remits almost completely when the child is asleep.
What is the most likely etiology of this child's noisy breathing?

(A) Asthma
(B) Bronchopulmonary dysplasia
(C) Cystic fibrosis
(D) Laryngomalacia
(E) Tuberculosis . Meningococcal sepsis
.
12. A 4-year-old child has a history of chronic left lower lobe pneumonitis. On
contrast bronchography, the area involved with the pneumonitis does not fill,
whereas the area around it does fill. What is the most likely diagnosis?
(A) Asthma
(B) Pulmonary sequestration
(C) Cystic fibrosis
(D) Bronchopulmonary dysplasia
(E) Bronchogenic cyst

13. A 5-year-old, apparently healthy child has had three episodes of painless, gross
hematuria over the past day. All of the following laboratory procedures are
indicated in the evaluation of this child EXCEPT which?
(A) Examination of the urine for red blood cell morphology and casts
(B) Renal and bladder ultrasonography
(C) Prothrombin time, partial thromboplastin time, and platelet count
(D) Urinary screening for hypercalciuria
(E) 24-hour urine collection for total protein and creatinine clearance

14. Which one of the following statements regarding the anemia of chronic disease is
true?

(A) Mean corpuscular volume is elevated


(B) Serum iron level is elevated
(C) Serum iron-binding capacity is elevated
(D) Marrow iron stores are increased
(E) Iron therapy is required to raise hemoglobin level

15. All of the following conditions are characterized by hypochromic, microcytic red
blood cells EXCEPT :
(A) iron deficiency anemia
(B) thalassemia major
(C) thalassemia minor
(D) glucose-6-P-D deficiency
(E) anemia of chronic disease

16. All of the following disorders are associated with prolonged bleeding time
EXCEPT :
(A) hemophilia A
(B) von Willebrand's disease
(C) aspirin-induced thrombocytopathia
(D) Bernard-Soulier syndrome
(E) idiopathic thrombocytopenic purpura

17. A full-term male infant is noted to have circumoral cyanosis and twitching of his
left hand at 12 hours of age. On physical examination, he is found to have absent
pupillary response to light and a small penis.

The most likely diagnosis is :


(A) hypocalcemia
(B) hypoglycemia
(C) congenital hypothyroidism
(D) congenital heart disease
(E) idiopathic epilepsy cyst

18. Factors most likely to contribute to the development of diabetic ketoacidosis


include all of the following EXCEPT :
(A) overeating
(B) vomiting
(C) omission of insulin doses
(D) infection
(E) lack of patient education

19. All of the following are problems commonly associated with cerebral palsy
EXCEPT :
(A) epilepsy
(B) mental retardation
(C) blindness
(D) behavioral problems
(E) strabismus

20. A 10-year-old girl is initially seen with a 2-day history of fever and a 4-cm warm,
tender, and fluctuant left anterior
cervical lymph node. What is the most likely diagnosis?
(A) Hodgkin's disease
(b) Acute lymphoblastic leukemia (ALL)
(C) Histiocytosis X
(D) Acute bacterial lymphadenitis
(E) Metastatic neuroblastoma

21. Which of the following symptoms would suggest a diagnosis of recurrent


abdominal pain?

(A) Pain awakening a child from sleep


(B) Pain associated with vomiting
(C) Pain located periumbilically
(D) Pain radiating to the back
(E) Continuous pain

22. Which of the following is the best initial treatment for scald burns?
(A) Debride the wound
(B) Apply cool water
(C) Apply butter or margarine
(D) Cover the wound with a bandage
(E) Apply pressure to the site of the burn

23. Which of the following is the best indicator of cystic fibrosis?


(A) A positive family history of cystic fibrosis
(B) The presence of digital clubbing
(C) A sweat test with a chloride concentration of 70 mEq/L
(D) Bronchiectasis on a chest radiograph
(E) A history of meconium ileus

24. A healthy 3-year-old girl presents with the acute onset of petechiae, purpura, and
epistaxis. Her complete blood count is as follows: hemoglobin 12 g/dL, white blood
cell count 5550/mm 3, differential normal, platelet count
2000/mm 3. What is the most likely diagnosis?
(A) Idiopathic thrombocytopenic purpura (ITP)
(B) Acute lymphocytic leukemia
(C) Aplastic anemia
(D) Disseminated intravascular coagulation
(E) Glanzmann's thrombasthenia

25. A 6-year-old girl experiences staring spells once or twice a day. They last only 15-
30 seconds. During these spells, she will stare, breaking off in midsentence at times.
Eyeblinking and lipsmacking are sometimes seen. After the spell, she will either
continue talking or she may look momentarily puzzled. She has no other neurologic
symptoms, and her schoolwork has not deteriorated. The episodes described are
most consistent with which of the following?

(A) Major motor seizures


(B) Partial complex seizures
(C) Absence (petit mal) seizures
(D) Daydreaming
(E) Tics

26. The following skills are present by the age of 9 months except:
A. Crawling.
B. Social smile.
C. Bowel control.
D. Sitting unsupported.
E. Head raising in the prone position.

27. The sex chromosomal complex XXY is diagnostic of:


A. Turner’s syndrome
B. Gonadal dysgenesis
C. Klinefelter’s syndrome
D. all of the above
E. None of the above

28. BCG Vaccine is given as follows :


A. O.1 ml injected intradermally
B. Two drops orally
C. O.5 ml injected subcutaneously
D. O.5 ml injected intramuscularly
E. 0.5 ml injected intravenously.

29. Seizures in the neonatal period are caused by:


A. Hypoglycemia
B. Brain anoxia
C. Hypocalcemia
D. Maternal heroin addiction
E. All of the above.

30. In febrile convulsions the following statements are true except:


A. occur in 2-5 % of children
B. genetic predisposition
C. high fever
D. in children aged between 6 months to 5 years
E. occur in newborns

31. Among the complications of whooping cough:


A. Pneumothorax.
B. Hemoptysis.
C. Conjunctival hemorrhage.
D. all of the above
E. none of the above.

32. All of the following are common in bronchial asthma EXCEPT:


A. generalized wheezing
B. increased eosinophils count
C. paroxysmal nature of the attacks
D. gross hemoptysis
E. dyspnea.

33) Congenital rubella syndrome is characterized by all the following except:


A. Is preventable.
B. Causes cataract.
C. Causes microcephaly.
D. Most serious when infection is acquired in the last trimester of pregnancy.
E. May cause congenital heart disease.

34) The following statements about a normal full term neonate are true:
A. Weighs about 3250 gm
B. It’s length is about 50 cm
C. Has a head circumference of around 35 cm
D. Has a hemoglobin of nearly l0gm/dl
E. Extension of upper and lower limbs

35) Physiologic hvperbilirubjnemia of the newborn is characterized by:


A. High level of serum conjugated bilirubin
B. Elevated serum alkaline phosphatase
C. Appears on the 3rd day of life
D. Positive Coombs’ test
E. All of the above

36) A patient with Down syndrome may suffer from all the following except:
A. VSD
B. Hypertonia
C. Mental retardation
D. Leukemia
E. Special facial features

37) Obligatory vaccination during the first year of life in Palestine includes all except:
A. BCG
B. DPT
C. Polio-vaccine
D. Meningococcal vaccine
E. Hemophilus influenzae vaccine

38) In cephalhematoma one of the following statements is correct:


A. The hematoma may cross over one cranial bone to the other.
B. Aspiration is the best treatment.
C. Hyperbilirubinemia may complicate the picture.
D. All of the above.
E. None of the above

39) Breast milk is contraindicated in:


A. Preterm babies
B. Phenylketonuria
C. Gastroenteritis
D. All of the above
E. None of the above

40) The daily requirement of calories in a two months old infant is:
A. 40 Cal/kg
B. 60 CalIkg
C. 80 Cal/kg
D. 100 Cal/kg
E. 200 Cal/kg

41) Daily vitamin D requirement in a 4 months old infant is:


A. 60.000 IU
B. 400 IU
C. 1500 IU
D. 100 IU
E. 1000 IU

42) Kwashiorkor has all the following except:


A. Edema
B. Anorexia
C. Color changes of hair
D. Spasticity
E. Skin changes

43) Hypertonic dehydration is characterized clinically by all EXCEPT:


A. high fever
B. moist tongue
C. hot dry skin
D. Irritability
E. oliguria

44) Seizures in the neonatal period are caused by:


A. Hypoglycemia
B. Brain anoxia
C. Hypocalcemia
D. Maternal heroin addiction
E. All of the above

45) Two years old infant with noisy breathing, barking cough and inspiratory distress
is suffering from:
A. Pneumothorax
B. viral croup
C. Heart failure
D. Bronchial asthma
E. Pleural effusion

46) Which of the following is a risk factor for asthma:


A. A family history of asthma
B. Chronic exposure to tobacco smoke.
C. Infantile eczema.
D. Air pollution.
E. All of the above.

47) The most common congenital heart disease in children is.

A. PDA
B. Tetralogy of Fallot
C. VSD
D. Coarctation of the aorta.
E. ASD

48) A continuous murmur- is most often due to:


A. Aortic incompetence
B. Coarctation of the aorta
C. Pulmonary artery stenosis
D. Aortic stenosis
E. Patent ductus arteriosus

49) Rheumatic arthritis is characterized by all except:


A. Fleeting in nature.
B. Tendency to affect big joints.
C. Normal ESR.
D. Dramatic response to salicylates.
E. May be associated with carditis.

50) Minimal change nephrotic syndrome is characterized by the following except:


A. Edema
B. Albuminuria
C. Hypercholesterolemia
D. Hypoproteinemia
E. Hypertension

51) A 3 year-old boy has had a temperature of 39°C and a stiff back for the last 3 days.
Examination shows a red throat, resistance of the neck to flexion. THE MOST
SPECIFIC DIAGNOSTIC TEST IS:
A. lumbar puncture
B. random blood sugar
C. throat culture
D. WBC, and re examination in 24 h
E. X - ray chest.

52) The most common congenital obstructive lesion of the left side of the heart is:
A. aortic stenosis
B. coarctation of the aorta
C. mitral stenosis
D. pre-aortic septal hypertrophy
E. accessory chordae tendinae

53) Compared with human milk, cow milk formula is MORE likely to contain a(n)
A. greater concentration of essential fatty acids
B. higher protein concentration
C. increased lactose content
D. lower calcium-phosphorus
E.ratio lower iron concentration

54) A 6-year-old girl can write her name and can count 10 objects.
Of the following, the MOST likely additional activity of which she is capable is:
A. hitting a baseball
B. knowing her right hand from her left
C. making a simple meal
D. printing neatly in small letters
E. sounding out words while reading

55) All of the following diseases are transmitted by X-linked recessive inheritance

Except:
A. Hemophilia B.
B. Hemophilia A.
C. Vitamin D-resistant rickets. >> X linked dominant
D. Bruton hypogammaglobulinemia.
E. Color blindness.

56) Which of the following statements regarding hepatitis A infection in children is


true?
(A) Most children with hepatitis A will develop clinically apparent jaundice.
(B) The most rapid test for diagnosis is viral culture of nasopharyngeal secretions.
(C) All household and sexual contacts should receive immune globulin.
(D) The incubation period is 10 to 21 days.
(E) Immunity to hepatitis A virus is short lived.

57) In which of the following children would the use of live oral poliovirus vaccine
(OPV) be contraindicated?
(A) one who developed a fever of 39°C following the previous administration of OPV
(B) one with diarrhea
(C) one with an upper respiratory infection and low-grade fever
(D) one whose sibling has acquired immune deficiency syndrome (AIDS)
(E) one with Hirschsprung’s disease

58) An increased incidence of inguinal hernia is associated with:


(A) female gender
(B) congenital heart disease
(C) post-term delivery
(D) cryptorchidism
(E) celiac disease

59) Which of the following statements about congenital toxoplasmosis is true?


(A) Microcephaly and hydrocephalus may occur.
(B) Subsequent infants are frequently affected.
(C) The full impact of the disease can be predicted at birth
(D) Abdominal wall defects may occur.
(E) Infected mothers are usually ill, with fever, headache, and malaise.

60) Which of the following, when given in excess, would most likely cause
pseudotumor cerebri?
(A) vitamin A
(B) vitamin B
(C) vitamin C
(D) vitamin D
(E) zinc

‫ طب األطفال في طب الطواريء‬f‫امتحان‬

1) A 4-year-old girl developed a fever to (40°C) and a diffuse, raised purpuric


rash this morning. Laboratory studies reveal: WBC 5,100/cu mm;
hemoglobin, 11.8 g/dL; platelets, 189 x 103/cu mm; prothrombin time, 11.8
sec; partial thromboplastin time, 28 sec; and fibrinogen, 250 mg/dL Of the
following, the MOST likely etiology of the purpura is:

A. disseminated intravascular coagulation


B. Henoch-Schِnlein purpura
C. hereditary telangiectasia
D. inherited platelet dysfunction
E. meningococcemia

2) A medical student asks you the difference between prothrombin time (PT)
and partial thromboplastin time (PTT). Which of the following statements is
TRUE?

A. Only the PTT is prolonged by heparin


B. Only the PTT is prolonged in hemophilia and von Willebrand disease
C. The International Normalized Ratio is derived from the PTT
D. The PTT is preferred for monitoring coumarin dosage
E. The PTT measures all the components of the PT except factor V

3) A 1-week-old term female infant suddenly develops melena. The mother is


healthy, and the pregnancy was normal. Delivery was attended by her family
at home, but it was uncomplicated. The infant is breastfeeding. Hemoglobin is
6.7 g/dL, white blood cell count is 13,000/cu mm, and platelets are 599 x
103/cu mm . Of the following, the MOST likely cause of the bleeding is:

A. factor VIII deficiency hemophilia


B. factor IX deficiency hemophilia
C. necrotizing enterocolitis
D. rectal fissure
E. vitamin K deficiency

4) A 10-year-old boy develops severe headache, photophobia, and emesis. He is


afebrile, and his blood pressure is 180/100 mm Hg. Results of cranial nerve
and motor examination are nonfocal. He has nuchal rigidity and extensor
plantar reflexes bilaterally (positive Babinski sign). Of the following, the
MOST appropriate study to obtain at this time is

A. cerebrospinal fluid analysis


B. computed tomography
C. electroencephalography
D. magnetic resonance imaging
E. radiography of the skull

5) A 4-year-old child is brought by ambulance to the emergency department.


He is unconscious and has deep, regular, and rapid respirations. A clinical
finding that BEST distinguishes this respiratory pattern as central
hyperventilation rather than tachypnea is:

A. arterial carbon dioxide pressure (Paco2) of 26 mm Hg


B. decreased mental status
C. nonreactive pupils
D. oxygen saturation of 95% by pulse oximetry
E. sternal retractions

The most likely finding in the initial stages of septic shock is:
A. abnormally low blood pressure
B. absent femoral pulses
C. delayed capillary refill
D. hyperpnea
E. tachypnea

6) The mother of a 3-year-old girl finds the girl submerged in a backyard pool.
Of the following, the finding that is MOST compatible with a favorable
neurologic prognosis for this child is:

A. apnea at the time of admission to the emergency department


B. coma at the time of admission to the emergency department
C. first spontaneous respirations within 10 minutes of resuscitation
D. initial arterial pH <7.0
E. submersion of more than 25 minutes

7) The mother of a 3-year-old reports that her son has begun to cough and have
difficulty breathing. She was keeping him home from preschool today
because he had a cough and upper respiratory tract infection without fever.
The MOST likely cause of this child's symptoms is:

A. aspiration pneumonia
B. bacterial pneumonia
C. foreign body aspiration
D. reactive airway disease
E. vascular ring

8) A 6 year old girl had a mild URTI that has worsened. She now has a high
fever, inspiratory stridor and barking cough, anteroposterior and lateral
neck radiographs demonstrates a normal supraglottic shadow, subglottic
narrowing and clouding of the tracheal air column. Of the following the most
likely cause of these findings is :

A. Haemophilus influenzae type b >> epiglottitis


B. Moraxella cataralis
C. Pseudomonas aeroginosa
D. Staphylococcus aureus
E. Streptococcus pneumonia

9) In acute bronchiolitis :

A. Ribavirin is the treatment of choice for hospitalized cases


B. Lung volume is usually decreased
C. Bronchodilators are usually effective
D. Feeding difficulties are common
E. Upper air way obstruction is a common feature

10)All of the following associated with Glucose-6-phosphate dehydrogenase


deficiency except :

A Is inherited as a sex-linked condition


B is clinically manifest in girls
C Causes drug-induced haemolysis
D Is an indication for splenectomy
E Is more pronounced in mature red blood cells
11)The following are commonly associated with Diabetes mellitus in children
except :

A always presents with keto acidosis


B Can often be managed with subcutaneous insulin
C May be associated with prior mumps infection
D Often remits for a variable period of time
E Has a peak incidence at 10-12 years of age

12)The following suggest bacterial rather than viral gastroenteritis except :

A Visible blood mixed with faeces


B Febrile seizure
C Diarrhoeal frequency of >5 stools per day
D Full blood count showing: Hb 5.2 g/l. WCC 14.5 platelets 102 x109/l
E Pulse rate of 80/ min in a child of 2 years

13)Which one of the following is a reliable method of assessing dehydration in


infants: Useful indices of dehydration in infants include:

A ) Assessment of intra-ocular tension by palpation


B ) Skin turgor over the dorsum of the hand
C ) Palpation of the anterior fontanelle
D ) Blood pressure
E ) Serum sodium concentration

14)The following are recognised causes of acute renal failure:

A Burns
B Dermatomyocytis
C Duchenne muscular dystrophy
D Penicillin therapy
E Alport's Syndrome

15)Which of the following statements applies to infants with gastroenteritis:

A ) In most instances require treatment with an antibiotic which is not absorbed from
the gut
B ) May have blood in their stools
C ) Should be admitted to hospital if they are unable to tolerate fluid orally
D ) Always develop lactose intolerance
E ) Should have a barium meal if the initial refeeding with milk is unsuccessful
16)Which of the following statements is true concerning: Intussesception in
children:

A ) Is more common between the age of 3 months to 8 years


B ) Bile stain vomiting is an early feature
C ) Bleeding per rectum is the most common presenting symptom
D ) Is usually caused by a polyp or pedunculated tumour
E ) Hydrostatic reduction plays an important role in the management

17)Minimal change nephrotic syndrome is characterized by the following except:

A. Edema
B. Albuminuria
C. Hypercholesterolemia
D. Hypoproteinemia
E. Hypertension

18)Beta thalassemia major is characterized by all except:

A. Hemosiderosis
B. Hepatosplenomegaly
C. Bone marrow hyperplasia
D. Positive Coombs test
E. Jaundice

19)You are asked to help prepare an oral rehydration solution. You recall that,
in addition to water, the MOST important components of such a solution are:

A. bicarbonate and potassium


B. chloride and potassium
C. lactate and potassium
D. sodium and fructose
E. sodium and glucose

20)An option for management of atypical pneumonia:

A. Penicilline
B. Erythromycine
C. Cephalosporines
D. All of the above
E. None of the above
21)Atropine poisoning causes :

A. Hallucination.
B. Bradycardia.
C. Profuse sweating.
D. Pin –point pupils.
E. Hypothermia.

22)Oxyuriasis is caused by:

A. Enterobius vermicularis
B. Toxocara canis
C. Trichuris trichura
D. Ankylostoma duodenale
E. none of the above

23)Cause of microcytic anemia, all are true except:

A. Iron deficiency anemia


B. Thalassemia
C. Anemia of chronic disease
D. Vit. B 12 deficiency
E. lead poisoning

24)The most likely pathogen responsible for croup (Acute laryngo-tracheo-


bronchitis) in children is:

A-Haemophillus influenzae type b


B-Staph.aureus
C-Respiratory syncetiat virus
D-Para influenza virus
E-Enterovirus

GOOD LUCK

1. Dental caries (cavities) in children are due to:


A. Metabolism of Streptococcus mutans
B. Enamel defects in teeth
C. Hereditary factors
D. Failure to brush teeth
E. All of the above

2. The most common cause of pancreatic insufficiency in childhood is


A. Chronic pancreatitis
B. Congenital lipase deficiency
C. Cystic fibrosis
D. Shwachman-Diamond syndrome
E. Pearson syndrome

3. A 5-mo-old boy was clinically diagnosed with gastroesophageal reflux at 3 mo of


age. Treatment with cimetidine resolved his constant crying and even seemed to
improve his spitting. His weight continues in the 50th percentile. On a follow-up
visit, however, his parents report that for the past 2 wk he once again has begun
frequent regurgitation, hiccups, and irritability. He is breast-fed and is given
occasional supplements of cow’s milk-based formula. His examination is normal.
Which of the following is the most appropriate intervention?
A. Reassurance
B. Fundoplication
C. Upper endoscopy with biopsies
D. Adjustment of the dose of cimetidine
E. Daily use of antacids

4. A 6-yr-old girl with documented ?l-antitrypsin deficiency presents to the


emergency department for assessment of large-volume hematemesis. Physical
examination is remarkable for clear lung fields, pallor, and splenomegaly.
Hepatomegaly and petechiae are absent. Which of the following is the most likely
source of the hematemesis?
A. Peptic ulcer disease
B. Thromboeytopenia secondary to hypersplenism
C. Swallowed blood from pulmonary hemorrhage
D. Esophageal variees
E. Erosive reflux esophagitis.

5. A 6-wk-old infant has a thoracotomy for transposition of the great vessels.


Following surgery, the infant develops loose stools. Stool cultures are negative and
the stool is Hemoccult-negative. Fecal pH is 6.0. The infant fails to gain weight.
Chest x-ray shows a pleural effusion. Which of the following would be the most
appropriate dietary intervention?
A. Medium chain triglycerides-predominant formula
B. Soy formula
C. Lactose-free formula
D. Carbohydrate-free formula
E. Long chain fat-predominant formula.

6. A 10-yr-old girl is brought for evaluation because her mother was found to have
familial adenomatous polyposis coli (APC). Genetic screening identified identical
APC gene mutations in mother, aunt, and grandfather, but no mutations were
identified in the girl’s APC gene. The girl requires:
A. Colonoseopy
B. Proetoeolectomy
C. Annual physical examination
D. CT scan
E. Repeat genetic testing in 1 yr.

7. A 4-yr-old boy presents with diarrhea and headaches. He appears to have a limp,
which favors the left leg. On examination, he has a poorly defined mass in the left
flank and a blood pressure of 170/100 mm Hg. Abdominal CT scan demonstrates a
large paraspinous mass on the left. The next step in the care of this boy is to:
A. Start therapy with a long-acting somatostatin analogue
B. Refer for immediate surgical exploration
C. Start a-adrenergic blockade and obtain MRI scan
D. Start -adrenergic blockade and obtain spine MRI scan
E. Refer to physical therapy.

8. A chronic or recurrcnt course of abdominal pain, weight loss, and anemia in an


adolescent girl suggests
A. Henoeh-Sch nlein purpura
B. Inflammatory bowel disease
C. Anorexia nervosa
D. Pelvic inflammatory disease
E. Irritable bowel disease.

9. All of the following statements regarding cyclic vomiting are true Except
A. Onset occurs between ages 3 and 5 yr
B. Episodes last 2-3 days
C. There are four or more emesis episodes per hour
D. It is a migraine equivalent
E. It is associated with nystagmus.

10. Wilson disease is associated with all of the following Except


A. Cardiomyopathy
B. Hepatomegaly
C. Ascites and portal hypertension
D. Dystonia and tremor
E. Hemolysis
F. Fanconi syndrome.

11. A 2.9-kg girl is born at term after having carried a prenatal diagnosis of left
congenital diaphragmatic hernia since 19 wk of gestation. Delivery and the early
postnatal period are uneventful, with mild tachypnea and retractions developing at
24 hr of age. Chest film reveals a normal abdominal gas pattern and multiple lucent
areas in the left lower thorax. The most likely diagnosis is:
A. Congenital diaphragmatic hernia (Bochdalek
B. Congenital diaphragmatic hernia (Morgagni
C. Congential cystic adenomatoid malformation
D. Congenital diaphragmatic hernia (hiatal
E. Cystic fibrosis.

12 A 5-yr-old child is referred with a 6-wk history of morning headaches, often


associated with vomiting. His parents have noted that during this period he has
become irritable and moody. Which of the following would be most likely to be
identified during the physical examination?
A. Marked elevation of blood pressure
B. Tenderness on percussion of frontal sinuses
C. Papilledema
D. Loud orbital bruit
E. Significant refractive error

13. A 6-yr-old child with neurofibromatosis (NFl) is found to have an optic glioma
on a routine MRI study, confined to the right optic nerve. Findings on the
neurologic, physical, and retinal examinations are normal. The visual acuity is 20/20
bilaterally, uncorrected. Which of the following is the correct management?
A. Surgical removal of the tumor
B. MRI of the optic nerve every 3 mo
C. Chemotherapy
D. Annual examination by a pediatric ophthalmologist.
14. A 2-yr-old boy with the spastic diplegia form of cerebral palsy is being
evaluated. MRI of his brain is most likely to show:
A.Multicystic encephalomalacia
B.Periventricular leukomalacia
C.Normal anatomy
D.Basal ganglia abnormalities
E.Agenesis of the corpus callosum

15. Which of the following therapies has been found to be effective in long-term
follow-up for some patients with the X-linked neurodegenerative disease
adrenoleukodystrophy?
A.Vegetable oil as a dietary supplement
B.Hyperbaric oxygen therapy
C.Bone marrow transplantation
D.Total volume exchange blood transfusion
E.Plasmapheresis

16. A 5-yr-old child has a 6-mo history of increasing difficulty in walking associated
with urinary incontinence. Physical examination shows an alert child with increased
deep tendon reflexes, clonus and bilateral Babinski reflexes in the lower extremities,
and absent deep tendon reflexes in the upper extremities associated with grade 3/5
weakness symmetrically in all extremities. The most likely diagnosis is:
A.Myasthenia gravis
B.Spinal cord tumor
C.Guillain-Barré syndrome
D.Metachromatic leukodystrophy
E.Peripheral neuritis

17. A 15-yr-old boy has lost his ability to walk. On physical examination, his ankle
and knee deep tendon reflexes are noted to be diminished. The weakness is greatest
in peripheral muscles. Cranial nerves all are normal. One week before these
symptoms arose, he returned from a camping trip. The most likely diagnosis is:
A.Myasthenia gravis
B.Organophosphate poisoning
C.Spinal muscular atrophy
D.Botulism
E.Tick paralysis
18. All of the following statements regarding congenital neuromuscular
disorders are true except:
A.Most are hereditary
B.Most are nonprogressive conditions
C.The definitive diagnosis is best made by electromyography (EMG)
D.Diagnosis for some disorders may be confirmed by genetic analyses of lymphocytes
E.Hypotonia is a common feature.

19. All of the following statements regarding Duchenne muscular dystrophy are
true except:
A.It is the most common hereditary neuromuscular disease
B.It is inherited as an autosomal dominant trait
C.Symptoms are rarely present at birth or in early infancy
D.The serum creatine kinase is consistently greatly elevated
E.It is more common in males

______________________________________________________________

Questions
1. The most unlikely cause for neonatal seizures on Day 2, in an
infant is
a. Metabolic
b. Birth Asphyxia
c. Intraventricular Hemorrhage
d. Neonatal tetanus
1. (d) Neonatal tetanus
Neonatal tetanus is very rare nowadays. The clinical features usually
start on Day 5 to Day 10 and NEVER IN FIRST TWO DAYS, rarely
after 2 weeks. Common Metabolic causes are (1) hypoglycaemia
(2) hypocalcaemia. Birth Asphyxia is the most common cause of
Neonatal seizures.

2. A full term neonate is having episodes of cyanosis and apnea


which worsen on feeding and seems better when he is crying.
The next step to establish a quick immediate diagnosis is
a. Echocardiogram
b. Bronchoscopy
c. Upper GI
d. Nasal exploration with a catheter
2. (d) Nasal exploration with a catheter
The Diagnosis is Bilateral choanal atresia :
(1) Infants are mostly obligate Nasal breathers
(2) Immediate diagnosis . bedside-passing a narrow 8 Fr. Catheter.
(3) Confirmatory: Endoscopy and CT Scan
(4) Oral airway –– Lifesaving
(5) Treatment –– Surgical

3. All of the following are seen in infants with Laryngomalacia, except,


a. Stridor beginning at (or) shortly after birth
b. No evidence of significant Expiratory obstruction
c. Stridor is typically loudest during feeding
d. Usually increases during crying
3. (d) Usually increases during crying
(1) Stridor due to laryngomalacia, usually diminishes during sleep
(or) when the child is crying (.ed Muscle tone may hold the
supraglottic structures out of airstream)
(2) No treatment is needed usually

4. Which of the following childhood problems improves with increase


in age
a. Conduct disorder
b. Emotional problems
c. Temper tantrum
d. Sleep disorder
4. (c) Temper tantrum (or) Oppositionalism
(1) From the age of 18 months to 3 years child begins to develop
autonomy and starts separating from primary caregivers.
(2) They tend to do the opposite of what has been requested and get
frustated when they can’t express their autonomy.
(3) Some children show their frustration by physical aggression
(Biting, kicking, hitting) (or) resistance. This physical aggression
is known as Temper tantrum.
(4) Reaches a peak . 2nd to 3rd year of life and subsides at 3 to 6
years.

5. An Infant exhibits stranger reaction by


a. 4 months
b. 6 months
c. 10 months
d. 1 year
5. (b) 6 Months
(1) By 6-7 months, an infant can differentiate between strangers and
primary caregivers.
It develops fear of strangers and turns away or runs toward primary
caregiver in sight of strangers.
(2) It is a normal phenomenon, may last upto 13 months.

6. Vocabulary of a 3 year old child is


a. 100 words
b. 150 words
c. 250 words
d. 500 words
6. (c) 250 words
18 months – 6 – 20 words
3 years – 250 words
2 years – uses Pronouns
1 year – 2 words with meaning

7. “Handedness” is evident in a child by


a. 2 years
b. 3 years
c. 4 years
d. 5 years
7. (b) 3 years

8. The Hormone not essential for inutero fetal growth is


a. Insulin
b. Insulin like growth factor
c. Somatotropic hormone
d. Thyroxine
8. (c) Somatotropic Hormone
Insulin and Insulin like growth factor are most essential for fetal
growth. Thyroxine is also essential. Somatotropic Hormone (or)
Growth hormone is not essential for fetal growth in utero.

9. A child can walk up and down stairs, alternating his feet by


a. 3 years
b. 2 years
c. 4 years
d. None of the above
9. (a) 3 years
Climb upstairs – 2 years
Coordinated climbing – 3 years

10. External Genitalia can be clearly differentiated in a fetus by


a. 12 weeks
b. 20 weeks
c. 8 weeks
d. 10 weeks
10. (d) 10 weeks
8 weeks– Ovaries and Testis distinguishable
10 weeks – External genitalia distinguishable

11. Child changes rattle from one hand to another at the age of
a. 3 Months
b. 6 Months
c. 8 Months
d. 1 Year
11. (b) 6 months
3– months – Holds rattle
5–6 months – Transfers any object from hand to hand.

12. Visual acuity reaches Adult level by


a. 3 years
b. 4 years
c. 5 years
d. 7 years
12. (d) 7 years
Binocular vision develops by 3 – 6 months.
Adult acuity and depth perception by 6–7 years.

13. Pincer grasp is seen at


a. 3 months
b. 9 months
c. 5 months
d. 7 months
13. (b) 9 months
3 months– Grasp appears
5 months– Bidextrous Grasp with Reaching
7 months– Palmar grasp
9 months– Pincer grasp

14. A child starts to take self decisions by the age of


a. 7 years
b. 3 years
c. 10 years
d. 12 years
14. (a) 7 years
Ref : Page 51
A child starts to make self evaluations and decisions from 6 years
and this continues upto 12 years during which refinement occurs.
Self esteem becomes a central issue.

15. Deciduous Dentition is completed by the age of


a. 6 months
b. 6 weeks
c. 6 years
d. None of the above
15. (d) None of the above
Ref : Nelson Page 1204 (Table)
Deciduous dentition appears by 6–7 months. Lower central incisor
is usually the first tooth to erupt and completed by 20 – 30 months
(second molars) – last deciduous tooth to erupt. First permanent
dentition 6 - 7 years (3 tooth)
(1) Lower central incisor and first molar
(2) Upper First molar

16. Fatty acids necessary during 0 to 6 months of age for brain are
a. Arachidonic acid
b. Linoleic acid
c. Both of the above
d. None of the above
16. (c) Both of the above
Ref : Nelson page 154.

17. The triad of Autistic Spectrum Disorders (ASD) are all except.
a. Impaired social interaction
b. Impaired learning
c. Impaired Communication
d. Impaired imagination
17. (b) Impaired Learning
Ref : O.P.G. Page 65
.(a), (c), (d) are the classic triad of ASD
.Onset usually before 3 years
.Impaired learning is associated finding.

18. Breast Milk at room temperature can be stored for


a. 7 hours
b. 24 hours
c. 4 hours
d. 10 hours
18. (d) 10 hours
Ref : O.P.G. Page 151
Expressed Breast milk can be stored for
1) 10 hours at Room temperature
2) 24 hours in a Refrigerator
3) 3 months –– at –20º C in a freezer.

19. Nipple Confusion occurs when


a. A Baby is allowed to suckle on one side most of the times
b. A suckling baby is pulled half way and placed on the opposite side
during feeding
c. When the baby is positioned incorrectly
d. None of the above
19. (d) None of the above
Ref : O.P.G. Page 149
NIPPLE CONFUSION occurs in a baby who has been fed with bottle
feeds, when put on to suckle at the breast. It finds very difficult and
confusing to feed at the breast.

20. Order of development of 2º sexual characteristics in a male is


a. Testis development – Axillary hair – Pubic hair – beard
b. Testis development – Pubic hair – Axillary hair - beard
c. Beard – Axillary hair – Pubic hair – Testis development
d. Axillary hair – Pubic hair – beard – Testis development
20. (b) Testis development – Pubic hair – Axillary hair - Beard
Ref : Nelson (55)

21. Order of development of 2º sexual characteristics in female is


a. Thelarche – Menarche – Pubarche – Adrenarche (Axillary Hair)
b. Menarche – Thelarche – Pubarche – Adrenarche (Axillary Hair)
c. Adrenarche (Axillary hair) – Thelarche – Pubarche – Menarche
d. Menarche – Pubarche – Adrenarche – Thelarche
21. (a) Thelarche – Menarche – Pubarche – Adrenarche (Axillary Hair)
Ref : Nelson (55)

22. An Infant sits with minimal support, rolls over from supine to prone
position, reaches for a toy, does not have a pincer grasp – the
developmental age is
a. 3 Months
b. 4 Months
c. 6 Months
d. 8 Months
22. (b) 4 months
Ref : Nelson (55)
Pincer grasp – 9 months
Palmar grasp – 3 months – 4 months

23. Incorrect about Thumb sucking


a. Can lead to malocclusion
b. Is a source of pleasure
c. Is a sign of insecurity
d. Must be treated at 1 year to prevent stubborness
23. (d) Must be treated at 1 year to prevent stubborness.
Ref : O.P.G. (55)
Thumb sucking usually between 1–4 years. Intervention required
only if it persists beyond 4–6 years.

24. Two Infants are born at 36 weeks. Infant A weighs 2.6 Kg, Infant B
weighs 1.6 Kg. Infant B is more likely to have
a. Congenital malformations
b. Low Hematocrit
c. Hyperglycemia
d. None of the above
24. (a) Congenital malformations
Ref : Behrman 16/e. 475 – 477
Macmillan 3/e. PP 206–209.
. Small for Date babies have different complications than appropriate
for date preterm infants.
. Congenital malformations, Asphyxia and Meconuim aspiration
Syndrome are common. Pneumothorax, Pulmonary hemorrhage
are more common than Hyaline membrane disease in these
infants.
. Neonatal symptomatic hypoglycaemia and elevated (or) normal
Hematocrit are common.

25. Which of the following drugs given during the last 2 weeks of
pregnancy is most likely to have deleterious effects on the fetus
a. Propranolol
b. Penicillin
c. Heparin
d. Phenytoin
25. (a) Propranolol
. Pencillin and Heparin have not been shown to cause ill effects on
the fetus.
. Phenytoin – causes Fetal hydantoin Syndrome when given in first
trimester.
. Propranolol – causes decrease in Heart rate, low cardiac output at
times of . ed demand and may cause hypoglycaemia.
26. The most uncommon manifestation / sign in Neonatal meningitis
is
a. Lethargy
b. Neck stiffness
c. Vomiting
d. Hypothermia
26. (b) Neck stiffness
Ref : IAP Textbook 2nde / Page 299 Behrman 1b/e – 544 – 546.
(1) The signs and symptoms of Neonatal meningitis are subtle and
nonspecific
(2) Neck stiffness is usually seen in older children.
(3) Temperature instability, Irritability, Vomiting, bulging AF, Lethargy
are common.

27. The most common Intra cranial tumor in 2 – 12 years age group is
a. Cerebellar Astrocytoma
b. Medulloblastoma
c. Ependymoma
d. Choroid plexus papilloma
27. (a) Cerebellar Astrocytoma
Ref : I A P text book 2nd edition – Page 306, 307
(1) Commonest Intracranial tumors in 2-12 yrs . INFRA TENTORIAL
IN LOCATION
(2) Of the Infra tentorial tumors, cerebellar astrocytoma, medulloblastoma
and brain stem gliomas are in order most common.
(3) Less than 2 years and adolescence, Infratentorial = Supratentorial
tumors.
(4) *Commonest Intra Cranial Tumor in general is secondary
metastasis.

28. The Important Prognostic Anthropometric measurement in a


newborn is
a. Weight
b. Head circumference
c. Length
d. None of the above
28. (a) Weight
Ref : I A P : 76 page chapter (7.1)
. In any Newborn, be it Term / Preterm weight is the most important
measurement. Appropriate weight for date babies tend to do well
(prognostic point) than small / low birth weight babies.
. Among low birth weight babies, those with normal or near normal
Head circumference have a good prognosis.

29. In a standard growth chart; Head circumference is included for


the age up to
a. 1 Year
b. 3 Years
c. 2 Years
d. 5 Years
29. (b) 3 years
Ref : I A P text book Page 79

30. The brain size of a one year old Infant in relation to an adult brain
is
a. 67%
b. 75%
c. 90%
d. 60%
30. (c) 90%
Ref : I A P – Page (79),
(1) At Birth : 67% Adult Brain Size
(2) At One year : 90%
. Brain Growth is most rapid in 20 – 36 weeks of Intrauterine life and
then in the first year of life.

31. The average normal blood pressure of a 3 year old child is


a. 75/50 mmHg
b. 90/60 mmHg
c. 65/45 mmHg
d. None of the above
31. (b) 90/60 mm Hg
Ref : I A P 2nd edition Page 31
The average normal BP in children
. New born 65/45 mm Hg
. 1 Year 75/50 mm Hg
. 3 Years 90/60 mm Hg
. 10 years 100/70 mm Hg

32. All are early indications for Exchange blood transfusion in Infants
with Rh-Hemolytic Disease except
a. Cord Hb > 12g/dL
b. Cord Bilirubin 5 mg/dL (or) more
c. Unconjugated Bilirubin > 10 mg/dL – 24 hrs
d. Rise of Bilirubin more than 0.5 mg/dL/hour
32. (a) Cord Hb > 12 g/dl
Ref : I A P Page 73
Cord Hb < 10 mg/dl is an early indication.

33. Two carpal bones are radiologically seen in the wrist X-ray by the
end of
a. 1 Year
b. 2 years
c. 4 years
d. 12 years
33. (a) 1 year
Ref : Nelson – Page 37
Capitate and Hamate appear as early as 2 – 3 months

34. IQ between 50-70 indicates


a. Moderate Mental Retardation
b. Mild Mental Retardation
c. Severe Mental retardation
d. Profound MR
34. (b) Mild Mental Retardation
Ref : O.P.G. 543
Intelligence Quotient = (Mental age/Chronological age) × 100
. Mild MR – 51 – 70
. Moderate MR – 36 – 50 . Sub Normal Intelligence - 71 – 89
. Severe MR – 21 – 35 . Normal Intelligence 90 – 100
. Profound MR – 0 – 20

35. Weight gain during first 3 months of age


a. 250 gm/month
b. 500 gm/month
c. 350 gm/month
d. 750 gm/month
35. (d) 750gm / month
Ref : O.P.G. – 4th page
First 4 months – 30gm / day
Next 4 months – 20gm / day Wt. gain
Last 4 months – 10gm / day
. Average Birth Wt. : 3 Kg – (O.P.G – 4)
. By 10th day : Birth Wt. is regained (3) Kg
. 5 months : Doubles Birth weight (6) Kg
. 12 months : Tribles Birth Wt. (9) Kg
. 24 months : Quadruples Birth Wt. (12) Kg; ie (1/5) of adult weight
Height at 2 yrs = (1/2) Adult Height; (i.e) (85–90 cm)
36. Length of the babies is recorded up to
a. 1 Year
b. 2 Years
c. 2 1/2 years
d. 1 1/2 years
36. (b) 2 years
Ref : O.P.G. 4
. Length of a Neonate is 50 cm
3 months – 60 cm
9 months – 70 cm
1 year – 75 cm
2 years – 90 cm

37. Head circumference at 12 years of age is


a. 48 cm
b. 50 cm
c. 52 cm
d. 49 cm
37. (c) 52 cm
At Birth – 35 cm
3 months– 40 cm
12 months – 45 cm
2 years – 48 cm
12 years – 52 cm
Increase in Head circumference >1 cm/2 weeks . Hydrocephalous
Head circumference equals chest circumference at 1 year.

38. Anthropometric measure that does not show much change in 1–


4 years is
a. Abdominal circumference
b. Head circumference
c. Skinfold thickness
d. Mid arm circumference
38. (d) Mid arm circumference
Ref : Nelson 2131
Between 1 – 4 years the subcutaneous fat gets gradually replaced
by increase in muscle mass. Both occur equally.Hence the loss of
fat equals increase in muscle. So there is little change in Midarm
circumference between 1 – 4 years.

39. A 5 year old child is assessed to have developmental age of 1


year. His IQ is
a. 20
b. 40
c. 80
d. 100
39. (a) 20
IQ = ( Mental age/Chronological age) × 100
IQ = (1/5) × 100 = 20

40. Shakir tape is used to measure


a. Height
b. Length
c. Midarm circumference
d. Head circumference
e. Chest circumference
40. (c) Midarm Circumference

41. Skeletal age of Infants of 8 months age is assesed by


a. X ray of Pelvis
b. X ray of Radius&ulna
c. X ray of skull
d. X ray of shoulder
41. (d) X ray of shoulder
Ref : O.P.G – 6
. 3 – 6 months – X ray shoulder
. 1 – 13 yrs – Hands & wrist
. 12 – 14 yrs – Elbow and Hip

42. Capacity of stomach in a newborn is


a. 20 ml
b. 30 ml
c. 50 ml
d. 100 ml
42. (b) 30 ml
Ref : Chaurasia Vol II 4/e – Page 238

43. Vomiting on the first day in a newborn is due to


a. Esophageal atresia
b. Pyloric stenosis
c. Amniotic fluid gastritis
d. Congenital megacolon
43. (a) Esophageal atresia
Ref : Nelson 588
(1) Vomiting from esophageal atresia occurs with the first feed.
(2) The diagnosis is suspected when excess, unusual drooling of saliva is observed and
resistance is encountered on attempting to pass a catheter into stomach.
(3) Vomiting in pyloric obstruction can occur anytime but mostly occurs in 4th to 6th
week.

44. Air in the jejunum in an X ray of an Infant can be seen in


a. 2–3 Hrs
b. 15 Min.–60 Min.
c. 3 Hrs to 6 Hrs
d. 24 Hrs.
44. (b) 15 min. – 60 min.
Ref : Nelson – 588
Air can be demonstrated in the X ray of an infant in
(a) Jejunum – 15 – 60 min.
(b) Ileum – 2 – 3 hrs.
(c) Colon – 3 hrs.
Absence of rectal gas in 24 hr film . Abnormal.

45. Large doses of Vit K administered to a new born may result in


a. Hyperammonemia
b. Hypobilirubinaemia
c. Prolonged Hyperbilirubinaemia
d. Prolonged bleeding time
45. (c) Prolonged Hyperbilirubinaemia
Ref : Nelson 190
. Hypoprothrombinaemia – due to deficiency of Vit K. otherwise called Hemorrhagic
Disease of new born. Prevented by prophylactic administration of Vit K.
. Hyperbilirubinaemia is caused by administration of large doses of Synthetic Vit K
analogues. It does not occur with Synthetic Vit K analogues.

46. Foetal scalp Blood pH; Less than –– is abnormal


a. 7.25
b. 7.3
c. 7.35
d. 7.4
46. (a) 7.25
Ref : Nelson 538
Fetal scalp blood pH ; during normal labour, decreases from 7.33 to
7.25. Any value less than 7.25 indicates Fetal distress, and pH less
than 7.2 requires active intervention. Acidosis is mainly due to hypoxia
induced Lactic Acidosis and monitoring of fetal scalp blood lactate
levels also shows the condition of the fetus.
47. Neonatal seizures – cause of which carries the best prognosis;
a. Birth asphyxia
b. Hypoglycaemia
c. Meningitis
d. Late onset Hypocalcemia
47. (d) Late onset hypocalcaemia

48. The normal heart rate at Birth is


a. 60–80 bpm
b. 80–110 bpm
c. 70–120 bpm
d. 110–150 bpm
48. (d) 110 – 150 bpm
Ref : Nelson 280 – Table 57.1

49. In Neonatal necrotising enterocolitis all of following are found except


a. Blood in stools
b. Delayed gastric emptying
c. Temperature instability
d. Increased Bowel sounds
49. (c) Temperature Instability
Ref : Nelson 590.
NEC – Neonatal Necrotizing Enterocolitis
. NEC is the most common life threatening emergency of GIT in neonates.
. Common in LBW and PreMature babies
. Distal ileum and Proximal colon are the commonest sites to be involved.
. Less common in Infants fed on Breast milk.
. Rarely occurs before initiation of enteral feeds.
. DIC, Shock, Acidosis are sequelae.

50. Umbilical cord usually falls after


a. 5–10 days
b. 3 days
c. 14 days
d. None of the above
50. (a) 5 – 10 days
Ref : Nelson – 608

51. The bone most frequently fractured during delivery is the


a. Radius
b. Clavicle
c. Ulna
d. Femur
51. (b) Clavicle
Ref : Nelson 572

52. The Characteristics of caput succedaeneum, are all except


a. Crosses midline
b. Crosses the suture line
c. Does not disappear within 2–3 days
d. None of the above
52. (c) Does not disappear within 2–3 days
Ref : Nelson 562
Caput Succedaneum
It is a diffuse edematous sometimes ecchymotic swelling of the layers of scalp in the
presenting part of the baby during delivery.
It is seen during birth
. Usually disappears in 3–5 days
. Large caput indicates prolonged labour.
. Moulding of the head and overriding of parietal bones usually seen.
. Crosses Midline and Sutural lines.

53. All are true about Cephalohematoma except


a. Limited to sutural lines
b. Subperiosteal hemorrhage
c. Visible several hours after birth
d. Requires I and D “Incision & Drainage”
53. (d) Requires I and D
Ref : Nelson 562
Caphalohematoma is a subperiosteal hemorrhage limited to a cranial bone.
It is viable several hours after birth.
I & D contraindicated.
May cause Hyperbilirubinaemia and prolongation of physiological jaundice.

54. A child following delivery, incomplete moro’s reflex, grasp


preserved one UL; Abduction and supination difficulty. The lesion
is at the level of
a. C8 T1
b. C2 – C3
c. C5 C6
d. C6 C7
54. (c) C5 C6
Ref : Nelson 565
Erb’s paralysis due to pulling of the infant vigorously by the arm.
C8, T1 – Klumpkis paralysis
55. 25% of Infants void within
a. 72 hrs
b. 48 hrs
c. 24 hrs
d. 8 hrs
55. (c) 24 hrs.
. Urine is passed usually during birth (or) immediately after birth.
. 95% of all infants void within 24 hrs.
Meconium : 99% . Pass within 48 hrs.

56. Most common cause of convulsions on the first day of life in a neonate is
a. Anoxia
b. Head injury
c. Hypoglycaemia
d. Meningitis
56. (a) Anoxia
Ref : O.P.G. Page 511
Nelson P: 567
Hypoxic ischaemic encephalopathy is the most common cause of neonatal
convlusions (50%).
Sepsis, meningitis are the next common causes.
Metabolic disturbances, like hypoglycaemia, hypocalcaemia and dyselectrolytemia
account for 25%.

57. Most common cause of respiratory distress after birth in the first 24 hrs is
a. Neonatal sepsis
b. Meconium Aspiration
c. Pneumonia
d. Air embolism
57. (b) Meconium Aspiration
Ref : O.P.G. 167 Table.
Commonest cause of respiratory distress in a preterm neonate is Hyaline Membrane
disease.
Post term and SFD babies, with H/o Meconium aspiration may develop respiratory
distress in 15% of cases.
Thin meconium causes, chemical pneumonitis, thick meconium causes airway
obstruction causing areas of atelectasis and emphysema.

58. All of the following increase the risk of respiratory distress syndrome (RDS),
except:
a. Maternal diabetes
b. Alkalosis
c. Caesarean section
d. Maternal diabetes
58. (b) Alkalosis
Ref : O.P.G : 166
RDS: Respiratory Distress Syndrome almost always occurs in PRETERM (< 34 weeks)
babies.
Surfactant production starts after 34 weeks and optimum levels occur only after 36 – 37
weeks.
Surfactant is a lipoprotein containing phosphotydyl choline and phosphotydyl glycerol.
Secreted by Type II Alveolar cells.
It helps to reduce surface tension (soap like effect) and thus prevents the collapse of
alveoli during expiration.
Respiratory distress is evident within SIX hours of life.
Lecithin / Sphingomyelin ratio (Prenatal diagnosis) normal > 2 and indicates adequate
lung maturity.
Shake test: simple bed side test. Where the amniotic fluid is mixed with absolute alcohol
in a test tube and shook for 15 minutes. Copious bubbles indicates adequate surfactant
and thus lung maturity.
Treatment
. Intratracheal administration of surfactant is now recommended.
. Ventilatory support - Intermittent Mandatory ventilation –– severe disease (IMV)
. Mild disease – Continuous Positive Airway Pressure (CPAP).

59. All are true regarding use of prenatal corticosteroids in the prevention of HMD,
in cases of preterm labour except;
a. All mothers at risk of preterm delivery between 24–34 wks need the therapy
b. Maternal Hypertension is not a contraindication
c. Maternal diabetes is not a contraindication
d. The effect of one course lasts for 2 weeks
59. (d) The Effect of one course lasts for 2 weeks

The benefits of prenatal corticosteroids are


(1) 50% reduction in reduction of RDS
(2) 40% reduction in mortality in babies with RDS
Chorioamnionitis is a contraindication for the therapy.
Treatment schedule :
(1) Inj. Betamethasone 12 mg Intramuscular every 24 hours ; 2 doses -(preferred drug)
(2) Alternatively, Inj. dexamethasone 6mg i.m every 12 hours (4) doses.
Optimal effect occurs after 24 hrs of initiating treatment.
One course lasts for 7 days.
Hence if labour prolongs / continues after the expected preterm date (i.e. example 34 – 36
weeks) the course should be repeated on the 7th day to have the desired effect.

60. True regarding Transient Tachypnoea of newborn is


a. Benign self limiting disease
b. Usually occurs in Preterm neonates
c. Chest X ray shows absent vascular markings
d. Significant respiratory distress is present
60. (a) Benign self limiting disease
Ref : O.P.G. 168
TTN :
(1) Occurs in Term Neonates
(2) Due to delay in clearance of lung fluids
(3) No respiratory distress or very minimal
(4) Oxygen treatment in adequate
(5) X ray shows Prominent vascular markings and prominent interlobar fissure.
(6) Prognosis – good.

61. Low birth weight baby is defined as


a. Birth weight <1500 gms
b. Birth weight <1500–2000 gms
c. Birth weight <2500 gms
d. None of the above
61. (c) Birth wt < 2500 gms.
Ref : Nelson 550
Low Birth weight : < 2500 gms
Very low birth weight : < 1500 gms
Normal Birth weight : < 3.5 Kg

62. All are seen during hypothermia in early neonate except;


a. Bradycardia
b. Excess shivering
c. Fall in core temperature
d. Metabolic acidosis
62. (b) Excess shivering
Ref : Nelson 528
Shivering mechanism is not developed in a neonate. Non shivering
thermogenesis occuring in Brown fat (nape of neck) accounts for the
source of heat in an infant.
__________________________________________________________

1) Defibrillation ( non-sync. shock) is used to treat:


a- pulseless ventricular tachycardia
b- hemodynamically unstable patient with Atrial flutter
c- hemodynamically unstable patient with Atrial fibrillation
d- hemodynamically unstable patient with ventricular tachycardia
e- none of the above
2) When you see the following ECG in cardiac arrest , the most appropriate
management is :
A) Defibrillation
B) Cardioversion (synchronized)
C) Give Atropine
D) Give Adenosine
E) Give lidocaine

3)The following ECG shows:


A) Ventricular fibrillation
B) Atrial fibrillation
C) Pulseless ventricular tachycardia
D) Extracystole
E) First degree heart block

4) Adrenaline 1: 10000 means


A) 1mg adrenaline in 1 ml H2O
B) 1mg adrenaline in 100ml H2O
C) 1g adrenaline in 100 ml H2O
D) 1g adrenaline in 1000ml H2O
E) 1g adrenaline in 10000ml H2O‫اعتقد‬

5) The following drugs can be given via Endotracheal tube EXCEPT


a- Adrenalin
b- Atropine
c- Lidocaine
d- Sodium bicarbonate
e- Diazepam

6) Which of the following maneuver is contraindicated by removing foreign body


from an infant ?
a. abdominal thrusts
b. finger sweep
c. chest thrusts
d. back blows
e. All of the above

7) Which statements about cardiac arrest is FALSE


a. Cardiac arrest is a clinical diagnosis
b. Any patient who is unconscious and who does not have a major pulse ( carotid or
femoral ) is in cardiac arrest
c. Some respiratory efforts such as gasping may persists for several minutes
d. Asystole is the most frequent initial rhythm documented in witnessed cardiac arrest
e. The aetiology of cardiac arrest in infants and children is different from that in adults.

8) Which cardiac rhythm is shockable ( can be treated with defibrillation )


A) Ventricular fibrillation
B) Pulseless Ventricular tachycardia
C) Asystole
D) PEA
E) A+ B

9) The most appropriate crystalloid used for resuscitation of hypovolemic trauma


patient is :
A- Ringer lactate
B- DW5%
C-0,45% Normal saline
D-Gelatines
E-DW 10%

10) A 4-year-old child was struck by a car traveling approximately 30 MPH and
was thrown approximately 15 feet. She is complaining of abdominal pain. Physical
exam reveals a blood pressure of 68/40 mmHg, a heart rate of 200 beats per minute,
and a firm, tender abdomen. She weighs approximately 20 kg. As long as the systolic
blood pressure remains below 90 mmHg, what is the most appropriate fluid
management?
(A) 800 cc of normal saline followed by 400 cc of blood.
(B) 400 cc of normal saline followed by 200 cc of blood.
(C) 400 cc of normal saline repeated 3 times, then 100 cc of blood.
(D) 800 cc of normal saline repeated 3 times, then 400 cc of blood.
(E) 400 cc of normal saline repeated 3 times, then 200 cc of blood

11) A 2-year-old male is rushed in by his parents after being found unconscious
near open bottles of his grandfather's medications. The airway is secure, and
successful bag- valve ventilations are started. He is pulseless and unresponsive.
Intravenous access is being obtained. The cardiac monitor shows a wide complex
tachycardia at a rate of 260. What is the next step in management?
(A) Shock the patient with 200 J.
(B) Shock the patient with 0.5 J/kg.
(C) Shock the patient with 2 J/kg.
(D) Administer epinephrine 0.01 mg/kg IV.
(E) Administer atropine 0.01 mg/kg IV

12) Synchronized cardioversion shock is used for all of the following EXCEPT:
A- VF
B- VT
C- SVT
D- AT
E- AF

13) Which of the following actions helps deliver maximum current during
defibrillation?
A) Apply conductive paste to the paddles
B) Reduce the pressure used to push down on the defibrillator paddles
C) Decrease shock energy after the 2nd shock
D) Place alcohol pads between the paddles and skin
E) Use small paddles

14) The most appropriate method to open the airway in comatose patient with
suspected cervical injury is :
A) Jaw thrust
B) Head tilt
C) Head tilt ,chin lift
D) Extension of the neck
E) Flexion of the neck

15- Endotracheal intubation in advanced CPR :


A- airway management of choice .
B- best airway control .
C- once achieved ,allow continuous chest compression .
D- not essential as first step .
E- all of above are true .

16) Appropriate Endotracheal tube size for 8 y child is :


A) Size 3
B) Size 4
C) Size 5
D) Size 6
E) Size 7

17) Daily fluid maintenance for 30 kg child is approximately


A. 1000 ml
B. 1300ml
C. 1700ml
D. 2000ml
E. 2300 ml

18)- Advantages of the laryngeal airway mask compared with ETT include all of the
following EXCEPT
A- Less tramatic
B- Incidence of aspiration is less than the endotracheal tube
C- No laryngoscope needed
D- Quick
E- May be used in emergency situation

19) The cricoid cartilage in an infant is located at the level of which cervical
vertebra?
A. C1.
B. C2.
C. C3.
D. C4.
E. C5.

20)Which drug can be used to treat this ECG ?


A) Lidocaine
B) Atropine
C) Adenosine
D) Adrenaline
E) All of the above

21) Normal blood volume for 3Kg Neonate is approximately :


A ) 180cc
B) 260 cc
C) 360cc
D)480 cc
E) 540 cc

22) Which statement about iv cannulation is FALSE


A) External jugular IV is indicated for critically ill patients < 12y old in whom an
extremity vein is not obtainable.
B) Intraosseous IV is indicated for critically ill patients < 6y old in whom an extremity
vein is not obtainable
C) Umbilical veins is indicated for new born <1week
D) Upper extremity IV sites are preferable to lower extremity sites .
E) The acellular fluids have the highest flow rates

23) Lactated Ringer contains all of the following EXCEPT


A) Potassium
B) Calcium
C) Sodium
D) Magnesium
E) Lactate

24)Which of the following causes of PEA is most likely to respond to immediate


treatment?
a. massive pulmonary embolism
b. hypovolemia
c. massive acute myocardial infarction
d. myocardial rupture and stroke
e. all of the above have the same response

__________________________________________________________________

MCQ PAPER 1

1-Concerning bronchiolitis:

A. Epidemics often occur during winter.


B. A chest radiograph usually shows a poorly inflated chest. (X)
C. Poor feeding is an indication for treatment with ribavirin. (X)
D. The disease is more benign in infants born prematurely. (X)
E. Respiratory syncytial virus is the most common causative virus.

2-The following are single gene disorders:

A. Cystic fibrosis.
B. Type 1 diabetes mellitus. (X)
C. Duchenne muscular dystrophy.
D. Marfan syndrome.
E. Turner syndrome. (X)
3-The risk of neonatal jaundice is increase by:

A. Prematurity.
B. Trisomy 21.
C. Elective caesarean section. (X)
D. Congenital hypothyroidism.
E. Cephalahaematoma.

4-In the emergency treatment / rescusitation of infants:

A. Tension pneumothorax is diagnosed by chest x-ray. (X)


B. Intraosseous needles must not be used under 1 year old. (X)
C. Oxygen should be administered in anaphylaxis.
D. Cardiac arrest is usually secondary to hypoxia.
E. Cardiac compressions are contraindicated when the heart rate is 4(X)

5-Recognised complications of measles include:

A. Encephalitis.
B. Febrile convulsions.
C. Diarrhoea.
D. Infertility in males. (X)
E. Corneal ulceration.

6-Concerning asthma:

A. Inhaled steroids are useful in the treatment.


B. Wheeze on auscultation is pathognomonic. (X)
C. It is seldom a cause of hospital admission. (X)
D. Exacerbation of symptoms can occur with exercise.
E. Sufferers should be excluded from sport at school. (X)

7-The following can be the cause of a non-blanching rash:

A. Idiopathic thrombocytopenic purpura.


B. Systemic lupus erythematosis.
C. Henoch-Schönlein purpura.
D. Haemolytic uraemic syndrome.
E. Meningococcal sepsis.

8-Regarding nocturnal enuresis:


A. Is more common in girls. (X)
B. Can be a presenting feature of diabetes melitus.
C. Can be a presenting feature of diabetes insipidus.
D. Emotional disturbance can be a cause.
E. Desmopressin is used in the treatment.

9-Concerning childhood immunisations (in the UK):

A. BCG vaccine should be given to children with HIV. (X)


B. A history of febrile convulsions is a contraindication. (X)
C. Meningococcal C vaccine is routinely offered.
D. Polio vaccine is usually administered orally. (X)
E. A fever Hib vaccine means futher Hib vaccinations are contraindicated. (X)

10-The following congenital heart lesions are "cyanotic":

A. Ventricular septal defect. (X)


B. Atrial septal defect. (X)
C. Patent ductus arteriosus. (X)
D. Tetralogy of Fallot.
E. Transposition of the great arteries.

MCQ PAPER 2

1-Concerning gastroenteritis (in the UK):

A. It can be fatal.
B. The most common cause is Salmonella spp. infection. (X)
C. Broad-spectrum antibiotics are usually indicated. (X)
D. A degree of lactose intolerance may follow gastroenteritis.
E. Blood pressure is a sensitive indicator of dehydration. (X)

2-Skin peeling occurs in the following conditions:

A. Infectious mononucleosis. (X)


B. Kawasaki disease.
C. Scarlet fever.
D. Toxic shock syndrome.
E. Erythema infectiosum (slapped cheek disease). (X)

3-In neonatal RDS (respiratory distress syndrome):


A. Surfactant is useful in the treatment.
B. It is rare in infants below 28 weeks gestation. (X)
C. Antenatal steroids are beneficial.
D. Maternal opiate abuse increases the risk. (X)
E. Maternal diabetes increases the risk.

4-In children with cerebral palsy:

A. All have a degree of learning impairment. (X)


B. Birth asphyxia is the most common cause. (X)
C. Hand preference before 12 months can be a useful sign.
D. The spastic form is the most common clinical type.
E. Management should adopt a multidisciplinary approach.

5-The following conditions are haemaglobinopathies:

A. G6PD deficiency. (X)


B. von Willebrand disease. (X)
C. Spherocytosis. (X)
D. Thalassaemia.
E. Sickle cell disease.

6-Theses neuromuscular disorders are "genetic diseases":

A. Duchenne muscular dystrophy.


B. Myotonic dystrophy.
C. Spinal muscular atrophy.
D. Guillain-Barre syndrome. (X)
E. (Typical) Myasthenia gravis. (X)

7-Concerning oncological disease in children:

A. Acute myeloid leukaemia is the most common leukaemia. (X)


B. Most brain tumours are metastases from other primaries. (X)
C. Wilm's tumour can present with haematuria.
D. Neuroblastoma can present in the first month of life.
E. Squint can be the presenting feature of retinoblastoma.

8-In pyloric stenosis:

A. Is more common in boys.


B. Surgery is the usual treatment.
C. Infants often have a metabolic acidosis. (X)
D. Constipation can be a presenting feature.
E. A barium meal is usually performed to confirm the diagnosis. (X)

9-The following can cause failure to thrive:

A. Coeliac disease.
B. Cystic fibrosis.
C. Emotional neglect.
D. Gastro-oesophageal reflux.
E. Patau syndrome.

10-The following are causes of polyhydramnios:

A. Maternal diabetes mellitus.


B. Potter syndrome. (X)
C. Anencephaly.
D. Oesophageal atresia.
E. Polycystic kidneys. (X)

MCQ PAPER 3

1-The following congenital abnormalities are deformations:

A. Coarctation of the aorta. (X)


B. Haemangioma. (X)
C. Talipes.
D. Cleft lip. (X)
E. Imperforate anus. (X)

2-Regarding anorexia nervosa:

A. Is never fatal. (X)


B. Occurs exclusively in females. (X)
C. Amenorrhoea can be a feature.
D. Growth hormone is typically elevated.
E. Distorted body image is common.

3-Systemic lupus erythematosus (SLE):

A. Is more common in females.


B. Psychosis is a recognized manifestation.
C. There is usually an associated thrombocytosis. (X)
D. Is more common in Caucasians than Africans. (X)
E. Autoantibodies against double-stranded DNA are frequently found.

4-In children with eczema:

A. Topical steroids are the first-line treatment. (X)


B. IgE is raised in most cases.
C. Parental atopy is a positive risk factor.
D. In infants the extensor surfaces are mostly affected.
E. Herpes simplex can cause serious infection.

5-Concerning congenital pseudohypoparathyroidism:

A. Inheritance is autosomal dominant.


B. Short 4th metacarpals are characteristic.
C. Parathyroid hormone levels are usually increased.
D. Calcium levels are decreased.
E. Urine analysis is helpful in confirming the diagnosis.

6-The following are inherited in an X-linked manner:

A. Duchenne muscular dystrophy.


B. Haemophilia A.
C. Cystic fibrosis. (X)
D. Sickele cell disease. (X)
E. Hunter syndrome.

7-Concerning chickenpox:

A. The incubation period is 7 days. (X)


B. Aspirin is a useful anti-pyretic. (X)
C. Transmission is usually by contact or airborne.
D. Encephalitis is a recognized complication.
E. There is no effective vaccine. (X)

8-Regarding Kawasaki's disease:

A. Conjunctivitis is a recognized feature.


B. It is caused by a spirochaete organism. (X)
C. It is a cause of coronary artery aneurysms.
D. Fever is not a typical feature. (X)
E. Is associated with profound thrombocytopaenia(X)

9-The following cause a microcytic anaemia:

A. Ulcerative colitis.
B. Pernicious anaemia. (X)
C. Methotrexate therapy. (X)
D. Folate deficiency. (X)
E. ß-Thalassaemia.

10-A normal two year old child would be expected to:

A. Drink from a cup.


B. Build a tower of 5 blocks.
C. Copy a circle. (X)
D. Kick a ball.
E. Hop on one foot. (X)

MCQ PAPER 4

1-Concerning brain tumours in children:

A. A sixth cranial nerve palsy can be the presenting feature.


B. The majority are metastatic from other tumors. (X)
C. They are the most common solid organ tumor.
D. Brain stem glioma usually present with personality change. (X)
E. Medulloblastomas can metastasize to the spine.

2-In depression in childhood:

A. A family history is common.


B. Suicide attempts are likely to be successful. (X)
C. Sleep is typically impaired.
D. Is more common in pre-pubertal boys than pre-pubertal girls.
E. Antidepressant medication is ineffective. (X)

3-Risk of developmental dysplasia of the hips is increased by:

A. Male sex. (X)


B. Asian ethnic origin. (X)
C. Positive family history.
D. Oligohydramnios.
E. Breech presentation.

4-The following exhibit autosomal dominant inheritance:

A. Gilbert syndrome.
B. Becker muscular dystrophy. (X)
C. Turner syndrome. (X)
D. Tuberous sclerosis.
E. Cystic fibrosis. (X)

5-Compared with cow's milk, human breastmilk contains:

A. Less sodium.
B. Less calcium.
C. Less protein.
D. Less fat. (X)
E. Less carbohydrate. (X)

6-Regarding cystic fibrosis:

A. It is most common in sub-Saharan Africa. (X)


B. The most common mutation is called delta-F508.
C. The responsible gene is located on chromosome 14. (X)
D. Can present in neonates with intestinal obstruction.
E. Can be investigated by measuring sweat electrolytes.

7-An 8 month old child would be expected to:

A. Roll from back to front.


B. Sit unsupported.
C. Transfer objects between hands.
D. Build a tower of 3 blocks. (X)
E. Have full head control.

8-The following are recognised causes of clubbing:

A. Cystic fibrosis.
B. Familial.
C. Bacterial endocarditis.
D. Fallot's tetralogy.
E. Ulcerative colitis.

9-The following are causes of short stature:


A. Klinefelter syndrome. (X)
B. Marfan syndrome. (X)
C. Emotional deprivation.
D. Hypothyroidism.
E. Achondroplasia.

10-Concerning systemic lupus erythematosus (SLE):

A. It causes a characteristic "butterfly rash".


B. Is more common in males. (X)
C. Arthralgia is a rare feature. (X)
D. Maternal SLE can cause heart block in the neonate.
E. Anti-nuclear antibodies are usually present.

MCQ PAPER 5

1-Concerning epilepsy in children:

A. There is an association with cerbral palsy.


B. Most forms of childhood epilepsy require lifelong treatment. (X)
C. Infantile spasms may respond to corticosteroid treatment.
D. Consciousness is usually impaired in absence seizures.
E. A post-ictal hemi-paresis may occur.

2-The following are common findings in Trisomy 21:

A. Learning difficulties.
B. Protruding tongue.
C. Congenital heart defects.
D. Single transverse palmar crease.
E. Hypertonia. (X)

3-Regarding rheumatic fever:

A. It occurs following a staphylococcal infection. (X)


B. May present with chorea (a movement disorder).
C. Erythema marginatum is a "major" diagnostic criterion.
D. It requires treatment with immunoglobulin. (X)
E. Renal impairment is the most serious long-term risk. (X)

4-UK infants are routinely immunised against:


A. Meningococcus type B. (X)
B. Diphtheria.
C. Cholera. (X)
D. Polio.
E. Pertussis.

5-These neuromuscular disorders are inherited conditions:

A. Duchenne muscular dystrophy.


B. Becker muscular dystrophy.
C. Spinal muscular atrophy Type 1 (Werdnig-Hoffmann disease).
D. Guillain-Barre Syndrome. (X)
E. Dystrophia myotonica.

6-Concerning ADHD (attention deficit hyperactivity disorder):

A. Impulsive behaviour is a feature.


B. Is associated with a higher rate of expulsion from school.
C. Is associated with a higher rate of teenage pregnancy.
D. Is associated with a higher rate of tic disorders.
E. May respond to treatment with stimulants such as amphetamines.

7-The following are causes of jaundice:

A. Prune belly syndrome. (X)


B. Wilson's disease.
C. Pityriasis rosea. (X)
D. Alagille Syndrome.
E. Alpha-1-antitrypsin deficiency.

8-The following drugs used in asthma are beta-agonists:

A. Salmeterol.
B. Salbutamol.
C. Ipratropium bromide. (X)
D. Fluticasone. (X)
E. Terbutaline.

9-Neonates in the UK are routinely screened for the following:

A. Hirschprung's disease. (X)


B. Phenylketonuria.
C. Fragile X syndrome. (X)
D. Hypothyroidism.
E. Congenital rubella infection. (X)

10-Regarding the condition of patent ductus arteriosus (PDA):

A. Chest CT is the optimum radiological investigation. (X)


B. May cause a "collapsing pulse".
C. May be treated by open surgical ligation.
D. May be treated medically using indomethacin.
E. May be treated by catheter trans-venous umbrella occlusion.

__________________________________________________________________

Neonatology
1) Regarding ultrasound scans of the neonatal head:
A Periventricular haemorrhages occur in 25% of very low birth weight infants. (True)
B Bleeds into the germinal matrix are unlikely to be associated with long term sequelae.
(True)
C Most haemorrhages occur in the first 72 hours of life. (True)
D Grade 4 haemorrhages are unlikely to be symptomatic. (False)
E Ischaemic lesions are easily detected in the paraventricular area. (False)

Comments:
Bleeds occur in about 25% of very low birth weight infants and are relatively easy to
see. Ischaemic lesions are much more difficult to detect, but may be marked by a flare
in the periventricular area. Lesions are graded I-IV.
1. Grade I means a bleed into the germinal matrix.
2. Grade II is unilateral blood in the lateral ventricle.
3. Grade III means the changes are bilateral and associated with dilatation of the
lateral ventricles.
4. Grade IV means that there are intracerebral abnormalities associated.
Grade IV lesions are the most serious, and are associated with significant risk of
neurodevelopmental problems. The worst outcomes are associated with bilateral
periventricular leucomalacia. Dilatation of the ventricles is readily detected on
ultrasound scan, and the commonest cause is intraventricular bleed. This may
spontaneously regress, arrest or progress causing significant hydrocephalus with tense
fontanelle, suture separation and enlarging head circumference. Fits or other
neurological symptoms may occur at this stage, which is usually treated with a VP
shunt.
2) Which of the following is/are true of Hirschsprung's disease?
A Often presents with neonatal large bowel obstruction. (True)
B Is due to absence of ganglion cells in Auberbach's plexus. (True)
C A contrast-study will show dilatation of the aganglionic segment. (False)
D Increased acetylcholinesterase activity is a histological feature. (True)
E Early treatment may involve rectal irrigation or an emergency colostomy. (True)

Comments:
Hirschsprung's disease is a common cause of neonatal large bowel
obstruction. It results from failure of migration of ganglion cells to the affected
segment of bowel. This always involves the distal colon but the proximal extent
of the involvement is variable and in rare cases may involve the whole of the
large bowel. Histologically, the affected segment has absent ganglion cells in
the Meissner's and Auerbach's plexus but immunohistochemical evidence of
increased ACE activity. 80% of cases present in the neonatal period. Contrast
studies show the affected segment to be tonically contracted. Rectal irrigation
or an emergency colostomy may be required before a definitive 'pull-through'
procedure.

3) The following may present as haemolytic disease of the newborn:


A Hereditary spherocytosis (True)
B Glucose 6 phosphate dehydrogenase deficiency (True)
C Sickle cell trait (False)
D ABO incompatibility (True)
E Vitamin K1 deficiency (False)

Comments:
Immune: (Rh, ABO, other)
Membrane defects: Spherocytosis, elliptocytosis
Enzyme defects: G6PD, PK, hexokinase
Sepsis
Polycythaemia: IDM, fetal transfusion.
Sickle cell and thalassaemia do not present in the neonatal period (HbF present).

4) The following features are characteristic of William's Syndrome:


A Supravalvular aortic stenosis (True)
B Short stature (True)
C Transient neonatal hypocalcaemia (False)
D Normal facies (False)
E Mild to moderate learning difficulties (True)
Comments:
Recognized clinical features of William's Syndrome include:
1. Short stature
2. Characteristic facies "elfin" (full face with high rounded cheeks, broad
forehead, flattened bridge of the nose and long upper lip),
3. Idiopathic hypercalcaemia
4. Supravalvular aortic stenosis
5. Mild to moderate learning difficulties.

5) Concerning blood flow in the fetus:


A Blood flows from right to left through the foramen ovale. (True)
B Blood in the ascending aorta has a higher oxygen content than in the descending aorta.
(True)
C The ductus arteriosus is closed. (False)
D Pulmonary pressure equals systemic pressure. (False)
E The haemoglobin may be 20g/dl. (True)
Comments:
Persistence of the fetal circulatory pattern of right-to-left shunting through the patent
ductus arteriosus and foramen ovale after birth is due to an excessively high
pulmonary vascular resistance. Fetal pulmonary vascular resistance is usually elevated
relative to fetal systemic or postnatal pulmonary pressure. This fetal state permits
shunting of oxygenated umbilical venous blood to the left atrium (and brain) through
the foramen ovale and bypasses the lungs through the ductus arteriosus to the
descending aorta. After birth, pulmonary vascular resistance normally declines rapidly
as a consequence of vasodilatation due to gas filling the lungs, a rise in postnatal PaO2,
a reduction in PCO2, increased pH, and release of vasoactive substances. Normal
haemoglobin range in the first 1-3 days of life is between 14.5-22.5 g/dl.

6) The following are normal in the newborn infant:


A A systolic blood pressure of 70mm in mercury. (True)
B Penile erections. (True)
C A Persistently palpable bladder. (False)
D A soft diastolic heart murmur. (False)
E An inherent prepuce. (True)
Comments:
Murmurs if innocent would be systolic and of short duration. Persistently
palpable bladder indicates the presence of urethral obstruction.

7) The following are associated with neonatal hypoglycaemia:


A Septicaemia (True)
B Respiratory distress syndrome (True)
C Rhesus incompatibility (True)
D Maternal beta blockers (True)
E Intravenous hydrocortisone (False)
Comments:
Neonatal Transient: Inadequate substrate (SGA, permaturity)
Hyperinsulinism (IDM, erythroblastosis fetalis) Septicaemia
Neonatal Persistent: Hyperinsulinism (Beckwith, nesidioblastosis)
Hormone deficiency (panhypopit, ACTH, GH, cortisol, adrenaline)
Substrate limited (ketotic hypoglycaemia, MSUD) Glycogen storage
diseases Disorders of gluconeogenesis (alcohol, aspirin,
hyperglycaemia)
Other: GIPUT
Disorders of fat (alternative fuel): 1st and 2nd carnitine deficiency, long
medium and short- chain fatty acid oxidation defects.

8) The major problems associated with multiple births include:


A Impossibility of breast feeding (False)
B Increased incidence of APH (False)
C Increased i ncidence of preterm labour (True)
D Increased incidence of growth retardation. (True)
E Increased incidence of congenital abnormalities. (True)
Comments:
Twins occur from natural causes about 1% of the time, with triplets approximately
1:10,000, and quadruplets 1:500,000. Fertility treatment has vastly increased this rate,
putting strain on intensive care cot facilities. The major problems associated with
multiple births are: Preeclampsia, preterm labour (-2 weeks from term for each extra
child) with all the extra morbidity and mortality that goes with it, complicated
deliveries including malpresentation, growth retardation and twin/twin blood
transfusions, particularly a problems with monochorionic twins, and an increased
incidence of congenital abnormalities.

9) Concerning fetal lung development:


A Type II pneumocytes are present at 24 weeks gestation. (True)
B Cuboidal cells are capable of gas transfer in utero. (False)
C There is virtually no smooth muscle in the terminal and respiratory bronchioles at 6
months of age.(True)
D The large airways are formed at 16 weeks gestation. (True)
E The adult complement of alveoli are present at birth. (False)
Comments:
Lung development proceeds with the budding of bronchi, bronchioles, and
successively smaller divisions. By 20-24 wk, primitive alveoli have formed and
surfactant production has begun (type II pneumocytes); before that time, the absence of
alveoli renders the lung useless as an organ of gas exchange. Alveolarisation is also
influenced by physical stimuli. Both the stretch by the liquid contained in the fetal lung
and the periodic distension provided by the action of the respiratory muscles during
fetal breathing, for instance, appear to be necessary for the development of the acinus.
Their absence when the lungs or chest are compressed (as in the case of a
diaphragmatic hernia or oligohydramnios) or when fetal breathing is abolished (by
spinal cord lesions, for example) results in pulmonary hypoplasia with reduced
numbers of alveoli.
Prenatal glucocorticoid therapy decreases the severity of RDS and reduces the
incidence of other complications of prematurity, such as intraventricular
haemorrhage, patent ductus arteriosus, pneumothorax, and necrotizing
enterocolitis, without affecting neonatal growth, development, lung mechanics
or growth, or the incidence of infection. Prenatal glucocorticoids may act
synergistically with postnatal exogenous surfactant therapy. Alveoli continue to
develop up to the age of 8 or so: this is why patients with chronic lung disease
tend to survive if they get through the first two winters.

10) The following maternal conditions can cause disease in the fetus/newborn:
A Hyperparathyroidism (True)
B Immune thrombocytopaenic purpura (True)
C Myasthenia gravis (True)
D Diabetes mellitus (True)
E Thyrotoxicosis (True)
Comments:
Organ failure: Cholestasis, cyanotic heart disease, renal transplant
Immune: Rh/ ABO, Diabetes, Graves, myesthenia gravis, ITP,
isoimmune neutropaenia or thrombocytopaenia
Hormonal: endemic goitre, hyperparathyroidism, obesity
Other: pre-eclampsia, genital herpes, sickle, Drug addiction, PKU,
melanoma.

11) The fetus with intrauterine growth retardation is at risk from:


A Developing diabetes in the neonatal period. (False)
B Anaemia. (False)
C Hyponatraemia. (False)
D Necrotizing enterocolitis. (True)
E Hypomagnesaemia. (False)
Comments:
Particular risks are:
Intrauterine hypoxia and death, and birth asphyxia.
Hypothermia (relatively large surface area).
Hypoglycaemia (poor fat and glycogen stores), with an increased risk of
diabetes in adulthood.
Hypocalcaemia.
Polycythaemia (venous amount >0.65, due to fetal response to hypoxia).
Most commonly growth retardation is asymmetrical, with relative preservation of head
circumference, and this is usually due to placental insufficiency. Symmetrical growth
retardation suggests prolonged poor intrauterine growth, and there is an increased risk
of fetal chromosomal disorder or syndrome, congenital infection, or maternal smoking,
drug or alcohol abuse, chronic medical condition or malnutrition. The latter infants
tend to remain small permanently, while the former often show good catch-up growth.
Antenatal CTG and Doppler ultrasound of the uterine artery are attempts to measure
the well-being of these fetuses, though they are not terrible sensitive or specific.

12) Regarding ingested substances during pregnancy:


A Eating unpasteurised dairy products increases the risk of listeria infection. (True)
B Eating kidneys can lead to vitamin A toxicity in the fetus. (False)
C Peri-conceptual folic acid supplements reduce the risk of neural tube defects. (True)
D Toxoplasma infection can be acquired from cattle. (False)
E An amniocentesis is mandatory when the mother is over 32 years old. (False)
Comments:
Exposure to toxoplasmosis should be minimised by not handling cat litter or eating
undercooked poultry. Listeria infection can be acquired from eating unpasteurised
dairy products such as soft ripened cheeses, pates, and ready to eat poultry unless it has
been thoroughly reheated. Eating liver during pregnancy is best avoided because of
potential vitamin A toxicity. Low dose folic acid supplementation is recommended for
all women planning a pregnancy with a higher does for women with a previously
affected fetus or on anti-epileptic medication.

13) Regarding the development of the skull sutures:


A Craniosynostosis is due to dysfunctioning osteoblasts.(False)
B The skull vault develops from mesenchyme. (True)
C Scaphocephaly develops from premature fusion of the coronal suture. (False)
D Occipital plagiocephaly is usually due to infant positioning. (True)
E Scaphocephaly develops from premature fusion of the lamdoid suture. (False)
Comments:
Craniosynostosis is defined as premature closure of the sutures, and may be primary
or secondary to failure of brain growth. The majority are idiopathic, with genetic
syndromes accounting for 10%. The skull bones develop from mesenchyme, and
craniosynostosis may be due to abnormal skull base development disrupting suture
development. Osteoblasts and osteoclasts are not thought to be abnormal. Most cases
are evident from birth, and a prominent bony ridge from the affected suture (S) may be
found, confirmed by skull x-rays. Specific forms include:
Scaphocephaly A sagittal suture.
Frontal plagiocephaly A coronal/sphenofrontal suture.
Occipital plagiocephaly A infant positioning.
Trigonocephaly Ametopic suture.
Turricephaly A coronal/sphenofrontal sutures.
Single suture involvement rarely is associated with neurological problems. Surgery is
only needed for cosmetic appearance. Genetic disorders involving multiple sutures
include: Crouzon, Apert, Carpenter, Chotzen, and Pfeiffer Syndromes.

14) Regarding birth asphyxia in the newborn:


A The fetal cardiotachograph is a sensitive and specific assessment of potential asphyxia.
(False)
B Fetal blood sampling is a good predictor of long term neonatal outcome. (False)
C Low apgar scores at 5 minutes are a good predictor of long term neurological outcome.
(False)
D Alternating hypotonia and hypertonia suggests severe hypoxic ischaemic
encephalopathy. (True)
E NMR imaging is mandatory in all children with possible birth asphyxia.(False)
Comments:
The incidence of birth asphyxia is around 5 per 1000 live births, and it is an important
cause of brain damage. CTG and fetal blood sampling are both poor predictors of long
term outcome. A low apgar score in the first 5 minutes is also a poor predictor, but a
low apgar at 10-20 minutes is a much better predictor of poor outcome.
HIE is graded as:
Mild - irritability, hyperventilation, poor feeding.
Moderate - lethargy reduced spontaneous movements, fits.
Severe - no spontaneous movements, no response to pain, seizures
are prolonged and resistant to treatment. Multi-organ failure may be present.
Mild HIE usually recovers completely. Severe HIE has a 12% mortality, with 20%
suffering neurodevelopmental disabilities including CP. The incidence of these
problems is increased if cystic lesions or ventricular dilatation from cerebral atrophy
are seen on scans.

15) With regard to the normal examination of the newborn:


A Cysts can occur normally on the gums (epulis). (True)
B There may be prolapse of a ring of vaginal mucosa in a female infant. (True)
C Petechiae may be found over the head and neck following a traumatic delivery. (True)
D Swollen eyelids suggest gonococcal infection. (False)
E An umbilical hernia often requires surgical repair in the first few (False) years of life.
Comments:
Cysts of the gums (epulis) or the floor of the mouth (ranular) are normal. A white
vaginal discharge or small withdrawal bleed occurs in female infants as transferred
maternal oestrogen levels fall, and this may be accompanied by a prolapse of a ring of
vaginal mucosa. Breast enlargement may also occur in either sex and a small amount
of milk may be discharged. Traumatic cyanosis is common, and may take the form of
petechial bleeding into the skin of the head and neck area and subconjunctival
haemorrhages. Swollen eyelids and distortion of the shape of the head from delivery
are common, as are ventouse marks and forceps marks. Umbilical hernias are common
particularly in Afro-Caribbean infants and no treatment is usually required with it
resolving by the first 3 years of life.

16) Indications for chorionic villus sampling include:


A Suspected Rhesus Disease (False)
B Chromosomal analysis (True)
C Enzyme analysis for inborn error of metabolism (True)
D DNA analysis for Duchenne muscular dystrophy (True)
E Suspected congenital infection (True)
Comments:
Chorionic villus sampling has slightly greater rate of fetal wastage than amniocentesis,
but does have the advantage of being possible in the first trimester before fetal
movements have started. Indications include chromosomal analysis, enzyme analysis,
DNA analysis, and to obtain samples for PCR diagnosis of congenital infection.

17) The following are recognized features of Noonan's Syndrome:


A The palprebal fissures are mongoloid in slant. (False)
B It is occasionally associated with mild learning difficulties. (True)
C The neck is webbed with a trident hairline. (True)
D Pectus excavatum is a recognized feature. (True)
E Dilated cardiomyopathy is a potential complication. (False)
Comments:
Clinical features of Noonan's Syndrome include: Characteristic facies with
Anti-mongoloid slant of the eyes which are relatively wide-spaced, with low-set ears
and Turners-like, short webbed neck with trident hairline. The chest is commonly
deformed with pectus excavatum or asymmetry and a short sternum. The stature is
short and there may occasionally be mild learning difficulties. Congenital heart disease
is common, particularly dysplastic pulmonary valve and ASD in addition to a
hypertrophic cardiomyopathy. Many children experience great difficulty with feeding
in the first year of life.
18) In monochorionic twins with uncompensated placental arteriovenous
shunts:
A The donor twin may have olighydramnios. (True)
B The donor twin may have microcardia. (True)
C The recipient twin may have large glomeruli. (True)
D By definition the twins vary in Hb by > 5g/dl. (True)
E The surviving twin is at risk of disseminated intravascular coagulation. (True)
Comments:
Placental vascular anastomoses occur with high frequency only in monochorionic
twins. In monochorionic placentas, the fetal vasculature is usually joined, sometimes in
a very complex manner. The vascular anastomoses in monochorionic placentas may be
artery-to-artery, vein-to-vein, or artery-to-vein. They are usually well enough balanced
so that neither twin suffers. Artery-to-artery communications cross over placental
veins, and when anastomoses are present, blood can readily be stroked from one fetal
vascular bed to the other.
Vein-to-vein communications are similarly recognised and are less common. A
combination of artery-to-artery and vein-to-vein anastomoses is associated with
acardiac fetus. This rare lethal anomaly (1:35,000) is secondary to the TRAP
sequence-Twin Reversed Arterial Profusion. Neodymium: YAG laser ablation of the
anastomoses, in utero, can treat heart failure of the surviving twin. In rare cases, one
umbilical cord may arise from the other after leaving the placenta. In such cases the
twin attached to the secondary cord is usually malnourished or dies in utero. Twins of
widely discrepant size are usually monochorionic.
In the fetal transfusion syndrome, an artery from one twin delivers blood that is
drained into the vein of the other. The latter becomes plethoric and large while
the former is anaemic and small. By definition, there is a 5 g/dl haemoglobin
and 20% body weight difference in this syndrome. Maternal hydramnios in a
twin pregnancy suggests the fetal transfusion syndrome. Anticipating this
possibility by preparing to transfuse the donor twin or to bleed the recipient
twin may be lifesaving. Death of the donor twin in utero may result in
generalised fibrin thrombin in the smaller arterioles of the recipient twin,
possibly as the result of transfusion of thromboplastin-rich blood from the
macerating donor fetus. The surviving twin may develop disseminated
intravascular coagulation. Treatment of this highly lethal problem includes
maternal digoxin, selective twin termination, or Nd:YAG laser ablation of the
anastomosis. Donor twin (arterial side): Oligohydramnios Small premature,
malnourished, hypoglycaemic Pale, anaemic, hypovolaemic Small glomeruli
Thin-walled arterioles Recipient twin (venous side): Polyhydramnios Large
preterm, well-nourished Plethoric, polycythaemic, hypervolaemic, cardiac
failure, Large glomeruli, Thick-walled arterioles.
19) The following definitions are true:
A Still birth rate is the rate of fetal deaths after 28 completed weeks of pregnancy per
1000 total pregnancies. (False)
B The perinatal mortality rate is the total of still births plus deaths within the first month
per 1000 live and still births. (False)
C The neonatal mortality rate is the deaths of live born infants less than 28 days of age
per 1000 live births.(True)
D An abortion is a premature expulsion from the uterus of the products of conception
before 26 completed week’s gestation. (False)
E A miscarriage is the loss of the products of conception from the uterus before 16
completed weeks. (False)
Comments:
Still birth is defined as a fetal death after 24 completed weeks of pregnancy.
Perinatal mortality rate is the still births plus deaths within the first 6 days per
1000 live and still births.
Neonatal mortality rate is the deaths of live born infants less than 28 days of
age per 1000 live births.
Miscarriage is the loss of the products of conception before the fetus is viable,
and an Abortion is the premature expulsion from the uterus of the products of
conception either embryo or non-viable fetus.

20) The following diagnoses can be reliably made on antenatal ultrasound


performed before 20 weeks:
A Spina bifida occulta (False)
B Gastroschisis (True)
C Ventricular septal defect (False)
D Gestational age (True)
E Down's Syndrome (False)

Comments:
Gestational age can be reliably estimated if performed before 20 weeks, but after this
the margin for error increases. Multiple pregnancies can be identified. Up to 70% of
major structural abnormalities can be identified, and more detailed scans and specialist
centres arranged. Fetal growth can now be reliable measured from serial abdominal
circumference by parietal diameter and femur length. Oligohydraminos and
polyhydraminos can also be diagnosed. Although specialist centres can reliably
diagnose major cardiac malformations, VSD can be very difficult to detect. Mutual
fold thickness is being investigated as a possible means of making a diagnosis of
Down's Syndrome, but this is not generally available or accepted.

21) The following are examples of multifactorial inheritance manifest in


the neonatal period:
A Anencephaly (True)
B Pyloric stenosis (True)
C Myotonic dystrophy (False)
D Ankylosing spondylitis (False)
E Leber's optic neuropathy (False)

Comments:
Polygenical multifactorial inheritance refers to a spectrum of disorders which are
neither purely environmental in origin nor purely hereditary. They are thought to result
from the additive effect of several genes with or without the influence of
environmental or other unknown factors. Height and IQ are inherited in this way, and
these parameters show a normal distribution in the population. Relatives of an affected
person show an increased liability to the disorder so that a greater proportion of them
than in the general population will fall beyond the threshold and will manifest the
disorder. The disease may also be more severe in relatives, particularly where there is a
close relationship to the affected person and there are multiple affected family
members. In addition, a sex difference in prevalence results in an increased risk to
relatives. The phenotype may be manifest as a congenital malformation or in adult life.
Congenital malformations: Neural tube defects, congenital heart
disease, cleft lip and palate, pyloric stenosis, CDH, talipes, hypospadias.
Adult life: Atherosclerosis and coronary heart disease, diabetes mellitus,
asthma, epilepsy, hypertension, HLA associated diseases.
Leber's hereditary optic neuropathy is a mitrochondrial abnormality.

22) Regarding retinopathy of prematurity:


A All babies who have received oxygen should have their eyes xamined until a corrected
age of 44 weeks gestation. (False)
B It occurs in 50% of very low birth weight infants. (False)
C Cryosurgery or laser therapy may be indicated for grade 3 or 4 disease. (True)
D It is first detected at the equivalent of 32-38 weeks gestational age. (True)
E It may progress extremely rapidly. (True)
Comments:
Retinopathy of prematurity (ROP, retinalentral fibroplasia) affects vessels at the
junction of the vascular and non-vascularised retina. Follow-up only needs to take
place until the retina is fully vascularised. Vascular proliferation may progress to
retinal detachment, fibrosis and blindness. It was previously the commonest cause of
blindness in children, but careful monitoring has reduced its incidence to only 30% of
very low birth weight infants (more in extremely preterm infants). This is usually only
grade 1 or 2 (reversible) rather than grade 3 or 4 (requiring treatment). It is first
detected between 32 and 38 weeks of age, but may progress rapidly. Severe visual
impairment occurs in only 1% of low birth weight infants.
23) The following are useful in assessing the gestational age of an infant:
A Posture (True)
B Elbow angle (False)
C Square window test (True)
D Nipple formation (True)
E Palmar creases (False)
Comments:
Compared with the premature infant of appropriate weight, the infant with retarded
intrauterine growth has a reduced birth weight and may appear to have a
disproportionately larger head relative to body size; infants in both groups lack
subcutaneous fat. In general, neurologic maturity (e.g., nerve conduction velocity)
correlates with gestational age despite reduced fetal weight. Physical signs may be
useful in estimating gestational age at birth. Commonly used, the Dubowitz scoring
system is accurate to ±2 wk. An infant should be presumed to be at high risk of
mortality or morbidity if a discrepancy exists between the estimation of gestational age
by physical examination, the mother's estimated date of last menstrual period, and fetal
ultrasonic evaluation.
Dubowitz scoring consists of:
NEUROLOGICAL: Posture, square window, ankle dorsiflexion, arm and
leg recoil, popliteal angle, heel-to-ear, scarf sign, head lag, ventral
suspension.
CUTANEOUS: Oedema, skin texture, colour, and opacity; lanugo;
plantar creases; nipple, breast, and genitalia formation; ear formation
and firmness.

24) The following statements are true regarding smoking in pregnancy:


A Smoking assists in maturation of the fetal lung. (False)
B The reduction in birth weight is related to the number of cigarettes smoked per day.
(True)
C Maternal smoking may adversely affect testicular function in male children. (False)
D Dysmorphic facies is a recognised complication. (False)
E The newborn baby may require adjustments in drug dosages because of it. (False)
Comments:
Smoking reduces birth weight which may be of critical importance if the baby is born
pre-term. On average, the babies of smokers weigh 170g less than non-smokers, but
the reduction in birth weight is related to the number of cigarettes smoked per day.
Smoking is also associated with an increased risk of miscarriage and still birth. The
infant has a greater risk of Sudden Infant Death Syndrome. There is some evidence
that maternal smoking may adversely affect ovarian function in female children. No
dysmorphic syndrome has yet been described.
25) following are characteristic of Cri-du-Chat Syndrome:
A Hypotonia (True)
B Laryngeal abnormalities (True)
C Hypotelorism (False)
D Encephalocele (False)
E Deletion of 5p- (True)

Comments:
Cri-du-Chat Syndrome is due to a 5p- deletion. The main features are:
Hypotonia
Short stature
Characteristic cry because of laryngeal abnormalities
Microcephaly with protruding metopic suture
Moon-like face
Hypertelorism
Bilateral epicanthic folds
High-arched palate
Wide and flat nasal bridge
Mental retardation.

26) Fetal alcohol syndrome is characterized by:


A Fetal anomalies in 10% of severe alcohol exposure (False)
B Intra-uterine growth retardation (True)
C Altered dermatoglyphics (True)
D Atrial septal defect (True)
E Joint hyperextensibility (False)
Comments:
High levels of alcohol ingestion during pregnancy can be damaging to embryonic and
fetal development. A specific pattern of malformation identified as the fetal alcohol
syndrome has been documented and major and minor components of the syndrome are
expressed in 1-2 infants/1,000 live births. Both moderate and high levels of alcohol
intake during early pregnancy may result in alterations in growth and morphogenesis
of the fetus; the greater the intake, the more severe the signs. Infants born to heavy
drinkers have twice the risk of abnormality compared with those born to moderate
drinkers; 32% of infants born to heavy drinkers demonstrated congenital anomalies,
compared with 9% in the abstinent and 14% in the moderate group.
The characteristics of the fetal alcohol syndrome include:
1. Prenatal onset and persistence of growth deficiency for length, weight,
and head circumference.
2. Facial abnormalities, including short palpebral fissures, epicanthal folds,
maxillary hypoplasia, micrognathia, and thin upper lip.
3. Cardiac defects, primarily septal defects.
4. Minor joint and limb abnormalities, including some restriction of
movement and altered palmar crease patterns.
5. Delayed development and mental deficiency varying from borderline to
severe. Fetal alcohol syndrome is a common cause of mental retardation.
The severity of dysmorphogenesis may range from severely affected infants with full
manifestations of the fetal alcohol syndrome to those mildly affected with only a few
manifestations. The detrimental effects may be due to the alcohol itself or to one of its
breakdown products. Some evidence suggests that alcohol may impair placental
transfer of essential amino acids and zinc, both necessary for protein synthesis, which
accounts for the intrauterine growth retardation. The management of these infants may
be difficult, since no specific therapy exists. The infants may remain hypotonic and
tremulous despite sedation, and the prognosis is poor. Counselling with regard to
recurrence is important. Prevention is achieved by eliminating alcohol intake after
conception.

27) Neonatal convulsions can be caused by:


A Maternal hyperparathyroidism (True)
B Subdural haematoma (True)
C Birth asphyxia (True)
D Hyponatraemia (True)
E Wilson's disease (False)
Comments:
Convulsions usually point to a disorder of the central nervous system and
suggest hypoxic-ischemic encephalopathy resulting from asphyxia, intracranial
haemorrhage, cerebral anomaly, subdural effusion, meningitis, hypocalcaemia,
hypoglycaemia, infarction, and, rarely, pyridoxine dependency, hyponatraemia,
hypernatraemia, inborn errors of metabolism, drug withdrawal, or familial
seizures.
Seizures beginning in the delivery room or shortly thereafter may be due to
unintentional injection of maternal local anaesthetic into the fetus.
Convulsions may also result from administration of large amounts of hypotonic
fluids to the mother shortly before and during delivery, leading to subsequent
hyponatraemia and water intoxication in the infant.
Convulsions (epileptic seizures) should be distinguished from the jitteriness
that may be present in normal newborns, in infants of diabetic mothers, in
those who experienced birth asphyxia or drug withdrawal, and in
polycythaemic neonates.
Jitteriness resembling simple tremors may be stopped by holding the infant's
extremity; it often depends on sensory stimuli and is not associated with
abnormal eye movements. Seizures in premature infants are often subtle and
associated with abnormal eye or facial movements; the motor component is
often that of tonic extension of the limbs, neck, and trunk.
Term infants may have focal or multifocal, clonic or myoclonic movements but
may also manifest more subtle seizure activity. Apnoea may be the first
manifestation of seizure activity, particularly in a premature infant.
Maternal hyperparathyroidism can result in neonatal hypocalcaemia, as maternal
calcium crosses to the fetus and suppresses the fetal parathyroid glands.

28) Regarding hyaline membrane disease:


A It is caused by deficiency of surfactant production by type 2 respiratory cells. (True)
B The hyaline membrane seen histologically is caused by a protenacious exudate.(True)
C It may be exacerbated by hypoxia, acidosis or hypothermia. (True)
D It occurs in 45% of children born before 28 weeks of completed gestation. (False)
E Steroids given antenatally to the mother stimulate surfactant production. (True)
Comments:
Surfactant is a mixture of lipoproteins produced by type 2 respiratory cells. It is
excreted by the alveolar epithelium, and results in a lowering of surface tension so that
alveoli can remain patent. The hyaline membrane on histology is formed by a
protenacious exudate. The more preterm the infant the higher the incidence of RDS,
with the majority of those below 28 weeks gestation being affected, though the
condition can still occur rarely in the term infant. Hypoxia, acidosis or hypothermia
increase the likelihood of it, and it is also commoner in the infant of a diabetic mother.
Antenatal steroids and exogenous surfactant therapy have been major advances in its
management.

29) The following conditions can be successfully treated by surgery on the fetus:
A Diaphragmatic hernia (False)
B Hydrocephalus (False)
C Hydronephrosis (False)
D Pleural effusion (True)
E Hypoplastic left heart syndrome (False)
Comments:
Fetal surgery is being attempted in specialised centres, but the results have been
generally disappointing. Attempts to repair diaphragmatic hernia have resulted in
preterm delivery, with no definite benefit on lung development. Pleural effusions have
been successfully drained, but shunting of urinary obstructions has been disappointing.
Treatment of hydrocephalus has largely been abandoned because the survivors are
severely disabled, and the treatment of stenotic heart valves have only been successful
in case reports. Currently, routine surgery is, therefore, not an option.
30) The following suggest Patau's Syndrome rather than Edward's Syndrome:
A Overlapping digits (False)
B Holoprose encephaly (True)
C Midline cleft palate (True)
D Rocker-bottom feet (False)
E Low-set ears (False)
Comments:
Edward's Syndrome: Small chin, low-set ears, overlapping digits, rocker-bottom feet,
cardiac and renal malformations.
Patau's Syndrome: Structural defects of the brain, Midline cleft palate ,scalp lesions,
microphthalmia and other eye defects, polydactyly, cardiac and renal malformations.

31) The following conditions can be treated by giving medication to the


mother:
A Fetal supraventricular tachycardia. (True)
B Accelerating surfactant production using antenatal steroids. (True)
C Reducing the risk of kernicterous in Rhesus Disease by giving mother Phenobarbitone.
(False)
D Atropine to treat fetal congenital heart block. (False)
E Frusemide to treat fetal hydrops. (False)
Comments:
Glucocorticoid therapy given before pre-term delivery accelerates lung maturation and
surfactant production reducing the incidence in severity of RDS and IVH. For optimal
effect it needs to be given at least 48 hours before delivery, but this is often not
possible. Digoxin or flucanide can be given to the mother to treat fetal supraventricular
tachycardia. Complete heart block is unresponsive to therapy, and Rhesus
immunisation is best treated with fetal blood transfusions into the umbilical vein. This
may be required regularly from about 20 weeks of gestation.

32) The following definitions are correct:


A A preterm infant is one born before 37 completed weeks of gestation.(True)
B A low birth weight infant weighs less than 2500g. (True)
C A very low birth weight infant weighs less than 1000g. (False)
D A extremely low birth weight infant weighs less than 750g. (False)
E Small for gestational age means that the birth weight is less than the 10th centile for
gestational age.(True)
Comments:
A neonate is an infant less than or equal to 28 days of age. A preterm infant is born
before 37 completed weeks of gestation, and a post-term infant greater than or equal to
42 completed weeks. A low birth weight infant is less than 2500g, a very low birth
weight infant less than 1500g, and an extremely low birth weight infant less than
1000g. Small for gestational age means that the birth weight is less than the 10th
centile for gestational age, while large for gestational age means the birth weight is
above the 90th centile for gestational age.

33) The following drugs given in labour can cause the adverse affects indicated in
the fetus:
A Opiates - constipation (False)
B Diazepam - hypotension (True)
C Oxytocin - fetal hypoxia (True)
D IV fluids - neonatal hyponatraemia (True)
E Opiates - delayed initiation of respiration (True)
Comments:
Opiates of anaesthetic agents may suppress respiration at birth and result in a delay in
establishing normal breathing. Epidural anaesthesia can cause maternal pyrexia which
is difficult to distinguish from infection. It may also delay feeding in the infant.
Sedatives such as Diazepam may cause sedation and hypotension in the newborn.
Oxytocin may hyperstimulate the uterus causing fetal hypoxia, and is also associated
with neonatal jaundice. Excessive IV fluids may cause neonatal hyponatraemia unless
they contain an adequate concentration of sodium.

34) Prolonged jaundice in a neonate can be caused by:


A Hypothyroidism (True)
B Galactosaemia (True)
C Von Gierke's disease (False)
D Gaucher's disease (False)
E Maple syrup urine disease (False)
Comments:
Prolonged jaundice may be:
UNCONJUGATED: Hypothyroidism, Crigler-Najjar Prolonged
haemolysis (immune-mediated, RC enzyme or membrane defect)
Sepsis.
CONJUGATED:
o Infection: bacterial (UTI, septicaemia), CMV, hepatitis,
toxoplasma, rubella
o Metabolic: galactosaemia, fructosaemia, CF, alpha0-AT
o Obstructive: biliary atresia, neonatal hepatitis syndrome,
choledochal cyst.

35) Regarding the neonatal cardiac examination:


A A chest x-ray and ECG are routinely indicated. (False)
B A grade 3/6 murmur is likely to be pathological. (True)
C Cyanosis on crying is normal. (True)
D Babies with weak femoral pulses should be re-examined a few hours later.(False)
E A loud second heart sound may be normal. (False)
Comments:
Heart murmurs are often audible in the neonatal period, but resolve shortly afterwards.
Only occasionally are these caused by congenital heart disease. Innocent murmurs are
usually soft and either blowing or buzzing in character. They are usually localized to
the left sternal edge with no diastolic component and no radiation. The heart sounds
are normal and there are no accompanying thrills, cyanosis nor abnormal pulses.
Cyanosis may occur in the neonatal period when crying due to shunting through a
patent foramen ovale. A loud second heart sound or weak femoral pulses should
always be taken as significant until proven otherwise, as neonates can deteriorate
rapidly if they have significant heart disease.

36) Necrosing enterocolitis is associated with:


A Intrauterine growth retardation (True)
B Cow's milk formula feeding (True)
C Birth asphyxia (True)
D Prematurity (True)
E Oxygen therapy (False)
Comments:
This serious disease of the newborn is of unknown aetiology and is
characterised by varying degrees of mucosal or transmural necrosis of the
intestine. No particular race or sex is unduly susceptible to the disease.
Incidence ranges from 1 to 5% of admissions to neonatal intensive care units.
Since the very small, ill preterm infant is particularly susceptible to NEC, a
rising incidence in recent years may reflect improved survival of this high-risk
group of patients. The disease does rarely occur in term infants. Many factors
may contribute to the development of a necrotic segment of intestine, the gas
accumulation in the submucosa of the bowel wall (pneumatosis, intestinalis),
and progression of the necrosis leading to perforation, sepsis, and death. The
distal ileum and proximal colon are involved most frequently.
A variety of factors such as polycythaemia, hypertonic milk or medicines, or
too rapid feeding protocols may contribute to mucosal injury and subsequent
infection leading to bowel necrosis. NEC also occurs in premature infants
without stress, particularly during epidemics. The clustering of cases suggests
a primary role for an infectious agent; Clostridium perfringens, Escherichia coli,
Staphylococcus epidermidis, and rotavirus have commonly been recovered
from cultures. Nonetheless, in most situations no identifiable pathogen is
recovered. It is associated with sick infants (who may require oxygen), but not
with oxygen itself.
37) The following are characteristic features of Prader-Willi Syndrome:
A Uniparental disomy with deletion of the paternal chromosome 15 (True)
B Large testes (False)
C Hypotonia (True)
D Initial growth retardation (True)
E Large hands and feet (False)
Comments:
Prader-Willi Syndrome is a classic example of imprinting. This refers to the
observation that phenotypic expression depends on the parent of origin for
certain genes and chromosomal segments. In the case of Prader-Willi
Syndrome, this comes from the paternal chromosome 15. Interestingly, a
defect at a similar location on chromosome 15 of the mother, results in
Angelman's Syndrome.
Prader-Willi is characterised by severe hypotonia at birth, obesity after initial
failure to thrive, short stature, small hands and feet, hypogonadism, and
mental retardation. In the differential diagnosis of childhood OBESITY, causes
may be endocrine, genetic or other:
1. Endocrine: Cushing's, hypothyroidism, hyperinsulinaemia, and growth
hormone deficiency, Prader-Willi, Stein-Leventhal, and
Pseudohypoparathyroidism type 1.
2. Genetic: Turner's syndrome, Laurence-Moon-Biedl Syndrome.
3. Other: Cohen Syndrome, Carpenter Syndrome.

38) The following are recognised clinical features of neonatal sepsis:


A Hypothermia (True)
B Vomiting (True)
C Hyperglycaemia (True)
D Jitteriness (True)
E Hypocalcaemia (False)
Comments:
Clinical features of neonatal sepsis include Fever or temperature instability, poor
feeding, vomiting, jaundice; irritability, seizures, lethargy or drowsiness; apnoea and
bradycardia or respiratory distress; abdominal distension; hypo or hyperglycaemia; or
shock. Meningitis is suggested by a tense or bulging fontanelle, fits, or head retraction
(opisthotonus).

39) Hydrops fetalis is caused by intrauterine infection with:


A Syphilis (True)
B Listeria (False)
C Parvovirus (True)
D Group B streptococcus (False)
E Toxoplasma (True)
Comments:
Causes of hydrops include:
IMMUNE : Haematologic: Rh and ABO incompatibility
NON-IMMUNE:
Infectious: parvovirus, CMV, toxo, syphilis
CVS: SVT, AV malformation, heart block
Pulmonary: diaphragmatic hernia, lymphangiectasia
Tumour: neuroblastoma, haemangioma, teratoma
Hepatic: hepatitis, cirrhosis, fibrosis
Renal: nephrosis, prune belly
GI: atresia, volvulus, CF
Metabolic: IDM, storage diseases
Malformations: Arthrogryphosis, Noonan
Chromosomal: XO, trisomies
Idiopathic.

40) The following methods can be used for prenatal diagnosis:


A Amniocentesis at 16-20 weeks of gestation (True)
B Chorionic villus sampling at 12-14 wks gestation (False)
C Fetal blood sampling (True)
D Foetoscopy (True)
E Ultrasonogram (True)
Comments:
Amniocentesis is done at 16-20 weeks gestation, for measurement of
metabolites and karyotype estimation. CVS from placental tissue has to be
performed 10th -12th week. Fetal blood sampling is sometimes performed.
Foetoscopy involves endoscopic examination or surgery of the foetus through
the abdominal/uterine portal of entry. Ultrasonography is the most non invasive
of all.

41) Duchenne muscular dystrophy:


A Occurs more commonly in children of elderly mothers. (False)
B One third have an IQ less than 75. (True)
C A translocation of an autosome to the Xp21 site explains why occasionally females are
affected. (True)
D The diagnosis is commonly confirmed by detecting the gene mutation.(True)
E In a muscle biopsy immune labelled dystrophin will be visible. (True)
Comments:
Duchenne muscular dystrophy is the most common hereditary neuromuscular
disease, affecting all races and ethnic groups. Its incidence is 1:3,600 liveborn male
infants. This disease is inherited as an X-linked recessive trait. The abnormal gene is
on the X-chromosome at the Xp21 locus and is one of the largest genes yet identified.
Becker muscular dystrophy is the same fundamental disease as Duchenne dystrophy,
with a genetic defect at the same locus, but clinically follows a milder and more
protracted course. Walking is often accomplished at the normal age of about 12
months, but hip girdle weakness may be seen in subtle form as early as the 2nd year.
Toddlers may assume a lordotic posture when standing to compensate for gluteal
weakness. An early Gowers' sign is often evident by 3 years of age and is fully
expressed by 5 or 6 years of age. A Trendelenburg gait, or hip waddle, appears at this
time. Contractures most often involve the ankles, knees, hips, and elbows. Scoliosis is
common. Enlargement of the calves (pseudohypertrophy) and wasting of thigh
muscles is a classic feature. Cardiomyopathy is a constant feature of this disease.
Intellectual impairment occurs in all patients, although only 20-30% have an
intelligence quotient (IQ) less than 70.
The muscle biopsy is diagnostic and shows characteristic changes. Myopathic
changes include endomysial connective tissue proliferation, scattered
degenerating and regenerating myofibres, foci of mononuclear inflammatory
cell infiltrates as a reaction to muscle fibre necrosis, mild architectural changes
in still functional muscle fibres, and many dense fibres. A specific molecular
genetic diagnosis is now possible by demonstrating deficient or defective
dystrophin by immunohistochemical staining of sections of muscle biopsy
tissue or by DNA analysis from peripheral blood. Confirmation of the diagnosis
by one of these methods should be done in every case. Death occurs usually
at about 18 years of age. The causes of death are respiratory failure in sleep,
intractable congestive heart failure, pneumonia, or occasionally aspiration and
airway obstruction.
Copyright © 2002 Dr Colin Melville

42) Respiratory distress syndrome:


A Occurs in 30% of premature babies of 34 weeks gestation. (True)
B Occurs more frequently in infants of diabetic mothers. (True)
C The lungs have low compliance. (True)
D Is commoner in girls. (False)
E Is more common in infants of mothers addicted to narcotics.(False)
Comments:
HMD occurs primarily in premature infants; incidence is inversely proportional to the
gestational age and birth weight. It occurs in 60-80% of infants less than 28 wk of
gestational age, in 10-30% of those between 32 and 36 wk, in about 5% beyond 37 wk,
and rarely at term. An increased frequency is associated with infants of diabetic
mothers, delivery before 37 wk gestation, multifetal pregnancies, caesarean section
delivery, precipitous delivery, asphyxia, cold stress, and a history of prior affected
infants. The incidence is highest among preterm male or white infants. Earlier lung
maturation may occur when there is severe premature separation of the placenta,
premature rupture of the fetal membranes, narcotic addiction, or maternal hypertensive
and renal vascular disease. Alveolar atelectasis, hyaline membrane formation, and
interstitial oedema make the lungs less compliant, requiring greater pressure to expand
the small alveoli and airways.

42) The conditions in which Genetic Anticipation occur include:


A Cystic fibrosis (False)
B Huntington's chorea (True)
C Fragile X syndrome (True)
D Myotonic dystrophy (True)
E Marfan's syndrome (False)
Comments:
The human genome is a dynamic apparatus, and rearrangements of DNA
sequences occurring as a normal mechanism (evolved perhaps to increase the
diversity of gene expression or as divergence from and expansion of a gene
family) are susceptible to mutation, and these tracts of DNA are often referred
to as unstable. A recently recognised type of mutation involves the expansion
of tandemly repeated nucleotide triplets. These Tri-nucleotide repeat arrays
are found in normal individuals in certain genes and are capable of
occasionally being expanded in size through an increase in trinucleotide repeat
number. If the number of repeats exceeds a certain threshold, the repeat array
becomes unstable, and additional size increases are likely to occur in
succeeding generations. This type of unstable (or dynamic) mutation can result
in disease in individuals carrying the expanded repeats.
At present these Tri-nucleotide repeat expansions fall into three classes, with
corresponding classes of phenotypes. The first class is characterised by large
expansions of a CGG trinucleotide (cytosine-guanine-guanine), leading to a fragile site
in the chromosome. Such a site is so designated because it is associated with
chromosome breakage under certain in vitro growth conditions. The prototype for this
class is the fragile X syndrome (FRAXA), in which an expanded CGG repeat in the 5'
untranslated region of the FMR1 gene leads to underexpression and a clinical
phenotype of mental retardation, macro-orchidism, and other somatic changes in
affected males. The second class of disorder involves the relatively small expansion of
an in-frame CAG (cytosine-adenine-guanine) repeat in the coding region of the
respective genes, leading to a polyglutamine stretch in the resulting protein.
Interestingly, all of the known disorders exhibiting this type of expansion are
dominantly inherited, late-onset neurodegenerative diseases, the best known example
being Huntington disease. The third class of disorder involving triplet repeat expansion
is represented by the disorder myotonic dystrophy. In this case a CTG (cytosinethymine-
guanine) repeat in the 3´ {prime} untranslated region of the relevant gene is
greatly expanded in affected individuals. A commonly observed characteristic of this
dominantly inherited disease is an increase in disease severity in successive
generations, a clinical phenomenon known as anticipation. Anticipation results from
the successive increases in repeat expansion and is observed to a lesser degree in the
other classes of disorders. Facioscapulohumeral (FSH) muscular dystrophy
(Landouzy-Déjerine disease) also shows the phenomenon of anticipation. The genetic
defect in autosomal dominant FSH muscular dystrophy is at the 4q35 locus.

43) Factors contributing to the occurrence of congenital dislocation of the hip


include:
A Breech presentation (True)
B Male sex (False)
C Achondroplasia (False)
D Laxity of the joints (True)
E Spina bifida occulta (False)
Comments:
Developmental dysplasia of the hip (DDH) usually occurs in the neonatal
period. The hips at birth are rarely dislocated but rather "dislocatable."
Dislocations tend to occur after delivery and, thus, are postnatal in origin,
although the exact time when dislocations occur is controversial. Because they
are not truly congenital in origin, the term development dysplasia of the hip is
now recommended. DDH is classified into two major groups: typical, in a
neurologically normal infant, and teratologic, in which there is an underlying
neuromuscular disorder such as myelodysplasia, arthrogryposis multiplex
congenita, or a syndrome complex. Teratologic dislocations occur in utero and
are therefore truly congenital.
The cause of DDH is multifactorial, having both physiologic and mechanical factors.
The positive family history (20%) and the generalised ligamentous laxity are related
etiologic factors. The majority of children with DDH have generalised ligamentous
laxity, and this can predispose to hip instability. Maternal estrogens and other
hormones associated with pelvic relaxation result in further, although temporary,
relaxation of the newborn hip joint. There is also a 9:1 female predominance.
Approximately 60% of children with typical DDH are first-born, and 30 -50%
developed in the breech position. The frank breech position with the hips flexed and
the knees extended is the position of highest risk. The breech position results in
extreme hip flexion and limitation of hip motion. Increased hip flexion results in
stretching of the already lax capsule and ligament teres. It also produces posterior
uncoverage of the femoral head. Decreased hip motion leads to a lack of normal
development of the cartilaginous acetabulum. There is also an association of
congenital muscular torticollis (14 -20%) and metatarsus adductus (1 00%) with DDH.
The presence of either condition requires a careful examination of the hips. Postnatal
factors are also important determinants. Maintaining the hips in the position of
adduction and extension may lead to dislocation. This puts the unstable hip under
pressure because of the normal hip flexion and abduction contractures. An unstable
femoral head, as a consequence, can be displaced from the acetabulum over several
days or weeks.

44) The following are true of infants with a single umbilical artery:
A The incidence is 1:100. (False)
B Congenital anomalies are found in about 1/3 of cases. (True)
C There is an association with trisomy 18. (True)
D Associated defects may be subtle. (True)
E There is an association with Beckwith's syndrome. (False)
Comments:
A single umbilical artery is present in about 500/1,000 births; the frequency is about
35-70/1,000 twin births. Approximately one third of infants with a single umbilical
artery have congenital abnormalities, usually more than one, and many such infants are
stillborn or die shortly after birth. 18-Trisomy is one of the more frequent
abnormalities. Since many abnormalities are not apparent on gross physical
examination, it is important that at every delivery the cut cord and the maternal and
fetal surfaces of the placenta be inspected. The number of arteries present should be
recorded as an aid to the early suspicion and identification of abnormalities in such
infants.

45) A baby girl was born at 32 weeks gestation weighing 1.2kg. Primary
immunisations should be given as follows:
A DPT at a corrected age of 2, 3 and 4 months. (False)
B BCG at 6 weeks of age. (False)
C Hepatitis B vaccination at birth, 1 month and 6 months of age. (False)
D MMR at 9 months of age. (False)
E Hib at 2, 3 and 4 months of age. (True)
Comments:
Premature babies should be immunised at chronological rather than corrected
age. They should receive the normal immunisation schedule which consists of:
DPT, Hib and Polio at 2, 3, and 4 months of age.
MMR at 10 - 12 months of age.
Booster DT and Polio at 3 - 5 years of age, along with second dose of MMR.
BCG in infancy or between 10 and 14 years of age.
Booster DT and Polio at 13 - 18 years.
For children who are on the neonatal intensive care unit, oral polio vaccine
should be deferred, with the first dose being given on discharge from hospital,
and the second and third doses being given at monthly intervals thereafter.
46) Recognised consequences of ABO incompatibility include:
A Coomb's negative haemolysis (True)
B Jaundice on the first day of life (True)
C Increased severity in further pregnancies (True)
D Normal haemoglobin on the first day of life (True)
E Spherocytes in the blood (True)
Comments:
Major blood group incompatibility between mother and fetus usually results in
milder disease than does Rh incompatibility. Maternal antibody may be formed
against B cells if the mother is type A or against A cells if the mother is type B.
However, usually the mother is type O and the infant is type A or B. Although
ABO incompatibility occurs in 20{endash}-25% of pregnancies, haemolytic
disease develops in only 10% of such offspring, and usually the infants are of
type A1, which is more antigenic than A2. Most cases are mild, with jaundice
as the only clinical manifestation. The infant is not generally affected at birth;
pallor is not present and hydrops fetalis is extremely rare. Liver and spleen are
not greatly enlarged, if at all. Jaundice usually appears during the first 24 hr.
Rarely, it may become severe, and symptoms and signs of kernicterus develop
rapidly. A presumptive diagnosis is based on the presence of ABO
incompatibility, a weakly to moderately positive direct Coomb's test, and
spherocytes in the blood smear, which may at times suggest the presence of
hereditary spherocytosis. The Coomb's test is not always positive.

47) Regarding apnoea of prematurity:


A The incidence of the idiopathic form varies inversely with gestational age. (True)
B It may be caused by neonatal sepsis. (True)
C It increases in frequency during non-REM sleep. (False)
D Bag and mask ventilation is often required. (False)
E May be treated with blood transfusion. (True)
Comments:
Periodic breathing must be distinguished from prolonged apnoeic pauses,
since the latter may be associated with serious illness. The aetiology includes:
1. CNS depression (hypoglycaemia, meningitis, drugs, haemorrhage).
2. Abnormal oxygen delivery (shock, sepsis, anaemia).
3. Ventilatory defects (pneumonia, HMD, PFC, muscle weakness).
It is described as idiopathic when identifiable causes are absent. The
idiopathic form varies inversely in frequency with gestational age. It is rare on
day 1, but tends to start between day 2 and 7 of life. Sudden onset of severe
apnoeas and bradycardia deserve investigation. Apnoea is defined as
cessation of breathing for longer than 20 seconds or cessation of breathing is
associated with cyanosis and bradycardia.
Treatment includes cutaneous stimulation, with bag and mask ventilation for
recurrent or prolonged apnoea. Oxygen and Theophylline or caffeine should
be prescribed if they recur. The haemoglobin should be maintained with
transfusion or erythropoietin. Occasionally nasal CPAP is required for mixed or
obstructive apnoeas (assists in splinting the upper airway). Unless severe,
recurrent, or refractory to therapy, the prognosis is related to underlying
conditions (IVH, BPD, ROP etc.). Apnoea of prematurity usually disappears by
36 weeks post-conceptual age, and does not predict future episodes of SIDS.

48) In respiratory distress in the newborn, the following suggests a diaphragmatic


hernia:
A Delivery by caesarean section (False)
B Heart sounds on the right side of the chest (True)
C A CTG showing type 2 dips (False)
D Scaphoid abdomen (True)
E Prolonged rupture of the membrane (False)
Comments:
Diaphragmatic hernia has an incidence of 1:4000. It is often diagnosed
antenatally, or may present with failure to respond to resuscitation or increase in
tachypnoea in the neonatal period. The majority are left-sided causing
displacement of the heart into the opposite hemithorax. Over vigorous
resuscitation may cause air leaks. A nasogastric tube should be passed and
ventilation commenced at high rate, low peaks and low pressures. It is often
accompanied by pulmonary hypoplasia and persistent fetal circulation.
Oscillatory ventilation, nitric oxide or echmo may be required. Antenatal evidence
of birth asphyxia makes meconium aspiration more likely, and delivery by
caesarean section is associated with an increased incidence of transient
tachypnoea of the newborn. Prolonged rupture of membranes is associated with
an increased risk of congenital pneumonia or septicaemia.

_________________________________________________________

1. A 9-month-old infant has atopic dermatitis characterized by erythematous,


chapped-appearing patches on the cheeks and extremities. Of the following, the
MOST appropriate initial treatment is
A. administration of an oral antibiotic
B. administration of an oral antihistamine
C. administration of an oral corticosteroid
D. application of a low-potency corticosteroid
E. avoidance of cow milk protein
2. A mother calls to report that her child was exposed to chickenpox in preschool.
The girl is healthy and has no history of varicella. The parents have refused
varicella vaccine. Of the following, your BEST advice is that the child should not
attend preschool
A. for the period 10 to 28 days following the exposure
B. for the period 10 to 21 days following the exposure
C. for 2 weeks following the exposure
D. if she develops chickenpox lesions
E. until chickenpox is no longer circulating in the classroom

3. A reporter for the local newspaper is interviewing you for an article on


preschoolers. He asks you to list some normal milestones for 4-year-olds. Of the
following, the MOST typical milestone for a 4-year-old is to
A. copy a square and triangle
B. prefer solitary or parallel play
C. print his or her first name
D. speak clearly in sentences
E. tie his or her shoelaces

4. A 10-year-old boy develops severe headache, photophobia, and emesis. He is


afebrile, and his blood pressure is 180/100 mm Hg. Results of cranial nerve and
motor examination are normal. He has nuchal rigidity and extensor plantar
reflexes bilaterally (positive Babinski sign). Of the following, the MOST
appropriate study to obtain at this time is
a) cerebrospinal fluid analysis
b) computed tomography
c) electroencephalography
d) magnetic resonance imaging
e) radiography of the skull

5. After intubation, arterial blood gas measurements for a 12-hour-old term infant include a
PO2 of 18 torr and a PCO2 of 25 torr while receiving ventilation with 100% FIO2. Chest
radiography shows normal cardiac size and diminished pulmonary vascularity. The
mechanism by which intravenous prostaglandin E1 will benefit this infant is MOST
likely due to
a. decreased pulmonary vascular resistance
b. decreased systemic vascular resistance
c. increased mixing of systemic and pulmonary circulations
d. increased pulmonary blood flow
e. increased systemic blood flow
6. A female infant born to a 24-year-old woman has been diagnosed clinically as
having Down syndrome. The mother is concerned about her risk of having another child
who has a chromosomal abnormality. The statement that you are MOST likely to include
in your discussion is that her risk
a. can be estimated by determination of maternal serum alpha-fetoprotein in all future
pregnancies
b. cannot be estimated until her infant's chromosome complement has been
determined
c. is increased for Down syndrome, but not for any other chromosomal abnormality
d. is no greater than that of any other woman her age
e. is not increased until she reaches the age of 35

7. A 3-year-old boy has a 1-week history of abdominal distension, flatulence, and frequent
bowel movements (six to eight movements every 24 hours). You suspect a parasitic
infection and obtain stool samples to test for ova and parasites. Results are positive for
Entamoeba histolytica.
Of the following, the MOST appropriate initial medication to administer is
A. diloxanide
B. furazolidone
C. iodoquinol
D. metronidazole
E. paramyocin

8. A 6-month-old boy has a pattern of sleep and feeding that is very unpredictable from day
to day. He is easily startled by new sounds. He cries loudly when upset and takes several
minutes to console. He sleeps with his parents, and his growth and development have
been normal.
Among the following, the best INITIAL management of this infant would be to
A. counsel the parents about differences in infant temperament
B. hospitalize the infant for observation
C. obtain an upper gastrointestinal series
D. prescribe chloral hydrate for sedation
E. switch to a protein hydrolysate formula

9. A 10-year-old boy is brought to the emergency department after being struck by a car
while riding his bicycle. Physical examination immediately upon arrival reveals that
he is responsive to painful stimuli only by moaning. His pupils are equal, small, and
reactive.
Of the following, the sign MOST LIKELY to be associated with increasing intracranial
pressure in this patient is
A. hyperventilation
B. hypotension
C. hypothermia
D. nystagmus
E. tachycardia

10. An infant who was born with myelomeningocele has hydrocephalus, and a
ventriculoperitoneal shunt is placed. His parents are concerned about recognizing the
signs and symptoms of shunt blockage.
Of the following, the BEST information you can give the parents is that
A. behavioral changes such as decreased spontaneity and mild lethargy
may indicate shunt malfunction
B. fewer than 30% of all shunts malfunction
C. most shunt failures occur more than 5 years after the initial surgery
D. seizures are the most common manifestation of shunt malfunction
E. shunt malfunction is usually due to infection

11. A 6-year-old girl is brought to your office for evaluation of swelling of her eyes and
legs. Three weeks ago she had a fever, bloody diarrhea, and abdominal pain that
resolved spontaneously. Laboratory studies reveal anemia, mild thrombocytopenia,
normal electrolyte levels, elevated blood urea nitrogen and creatinine concentrations,
and proteinuria.
Of the following, the MOST likely cause of this child's edema is
A. acute onset of nephrotic syndrome
B. Berger disease
C. hemolytic-uremic syndrome
D. Henoch-Schönlein purpura
E. poststreptococcal glomerulonephritis

12). A male infant is born at an estimated gestational age of 34 weeks. His


measurements at birth are: weight, 1,200 g (<10th percentile); crown-heel length, 40
cm (10th percentile); and head circumference, 31.5 cm (50th percentile).
Of the following, the MOST likely explanation for the growth pattern of this infant is
A. chromosomal abnormality
B. congenital viral infection
C. gestational diabetes
D. hereditary constitution
E. pregnancy-induced hypertension

13. A 16-month-old girl is brought to the emergency department for evaluation of a


fever, irritability, and possible seizure. She has been receiving amoxicillin to treat
otitis media for the past 3 days. The child appears ill, is irritable, and has a
temperature of 39°C. Studies obtained on the cerebrospinal fluid reveal: glucose, 1.67
mmol/L (30 mg/dL); protein, 0.8/L (80 mg/dL); and white blood cell count, 500 x
106/L (500/mm3) with 85% polymorphonuclear leukocytes. Gram stain is negative,
and cultures are pending.
Of the following, the MOST likely diagnosis is
A. herpes simplex virus encephalitis
B. Lyme disease
C. partially treated bacterial meningitis
D. tuberculous meningitis
E. viral meningitis

14) You have been asked to evaluate a 7-year-old boy who has moderate persistent
asthma. He has not seen any primary care physician for more than 6 months. You
decide to formulate an appropriate treatment plan for him.
Of the following, the BEST maintenance therapy is
A. inhaled beta-agonist
B. inhaled corticosteroid daily
C. leukotriene receptor antagonist daily
D. nebulized cromolyn sodium
E. oral theophylline

15) A7-year-old boy presents with left-sided proptosis and a 1-week history of purulent
nasal discharge and fever. On physical examination, you observe downward and
outward left-sided proptosis with good visual acuity but with limitation of left lateral
gaze.
Of the following, the MOST appropriate diagnostic test is
A. a lumbar puncture
B. computed tomography with contrast of the brain
C. computed tomography with contrast of the paranasal sinuses
D. magnetic resonance imaging of sinuses
E. sinus radiography

16) A 16-month-old boy has had severe emesis and diarrhea for 3 days. On physical
examination, he appears dehydrated. You administer fluids rapidly to correct his
volume loss and serum electrolyte abnormalities. On the next day, he appears
confused and exhibits quadriparesis and dysarthria. Magnetic resonance imaging
reveals demyelination of the central basis pontis.
Of the following, the rapid correction of which condition is MOST likely responsible for
these findings?
A. hyperkalemia
B. hypermagnesemia
C. hypocalcemia
D. hyponatremia
E. hypophosphatemia

17) A 6-month-old previously healthy infant presents with a 2-day history of cough and
difficulty breathing. On physical examination, she appears lethargic, pale, and poorly
perfused. She exhibits marked tachypnea, and auscultation of the chest reveals
decreased breath sounds bilaterally and poor aeration. She has supraclavicular and
intercostal retractions. Of the following, the MOST appropriate therapy for this infant
is
A. continuous positive airway pressure via face mask
B. endotracheal administration of surfactant
C. endotracheal intubation and positive pressure ventilation
D. negative pressure ventilation
E. nitric oxide

18) The decreased incidence of enteric infections noted in breastfed infants compared with
formula-fed infants is MOST likely due to the
A. more alkaline stool pH in breastfed infants
B. nutritional benefits of human milk on the infant's immune system
C. predominance of Bacteroides and Clostridium in the gut of breastfed
infants
D. presence of protective antibodies against enteric infection in human
milk
E. sterility of human milk

19) A newborn who weighs 600 g and whose estimated gestational age is 24 weeks at birth
is admitted to the neonatal intensive care unit after successful resuscitation in the
delivery room. Arterial blood gas measurements on room air are: pH, 7.35; PCO2, 42
mm Hg; PO2, 68 mm Hg; base deficit, 2 mEq/L.
Of the following, the MOST appropriate initial management is to
A. begin intravenous vancomycin
B. begin phototherapy
C. initiate enteral feeding
D. provide bicarbonate infusion
E. provide glucose infusion

20) You have been asked to evaluate a 15-month-old toddler who was admitted to your
community hospital for observation 6 hours ago after a near-drowning episode. The nurse
caring for the toddler reports that the child has become more tachypneic and distressed.
Her oxygen requirement also has increased.
Of the following, the MOST likely diagnosis is

A. adult respiratory distress syndrome


B. upper airway oedema
C. bacterial pneumonia
D. pneumothorax
E. reactive airway disease

21) Two days ago a 5-year-old girl undergoing chemotherapy for acute lymphoblastic
leukemia spent the morning with a playmate that now has developed varicella. She has
not received varicella vaccine and has no history of having varicella.
Of the following, the MOST appropriate action is to

A. administer varicella vaccine


B. administer varicella-zoster immune globulin
C. begin oral acyclovir
D. discontinue chemotherapy for 3 weeks
E. obtain varicella titers to determine the need for therapy

22) The 23-valent pneumococcal polysaccharide vaccine is RECOMMENDED for the child
who
A. attends a large child care center
B. has frequent ear infections
C. is older than 2 years of age and has nephrotic syndrome
D. is younger than 2 years of age and has sickle cell disease
E. lives in the same household as a patient who has asplenia

23) A 9-month-old girl who attends chid care presents with fever, irritability, and pulling at
her left ear. Physical examination reveals a bulging, erythematous left tympanic
membrane and no movement with pneumatic otoscopy. Findings on the remainder of the
examination are unremarkable, and the girl has no history of ear infections.
Of the following, the MOST appropriate STEP to manage this child

A. obtain blood and CSF culture


B. amoxicillin 80 mg/kg per day for 10 days
C. referred to ENT specialist for tympanoplasty
D. covered with penicilline G 100 mg/kg
E. azithromycin 10 mg/kg on day 1 and 5 mg/kg for days 2 through 5
24) An 18-month-old infant is brought to your office for evaluation of diarrhea that occurs
every 2 or 3 days, but does not interfere with her appetite, activity level, or sleep.
Attempts at controlling her loose stools with dietary modification have been
unsuccessful. Results of a physical examination, including growth and development, are
normal.
Of the following, the MOST likely cause of this child's diarrhea is
A. chronic nonspecific diarrhea
B. cow milk allergy
C. giardiasis
D. lactose intolerance
E. postviral gastroenteritis syndrome

24) You are examining a term newborn in the nursery. His weight is 3.27 kg (50th
percentile), and his length is 50.5 cm (50th percentile). The pregnancy, labor, and
delivery were unremarkable. There are no significant findings on physical examination.
The MOST likely head circumference in this child, if it is consistent with his other growth
parameters, is
A. 31 cm
B. 33 cm
C. 35 cm
D. 37 cm
E. 39 cm

25. A healthy 8-month-old boy has had intermittent episodes of intense cyanosis of the
hands, lower arms, and feet since birth. The infant appears alert and playful during the
episodes. At other times, his arms and legs are pink but have a "mottled" appearance.
Physical examination reveals no remarkable findings other than mottling of the
extremities.
Of the following, the BEST diagnostic test for this child is
A. ambulatory electrocardiography to detect arrhythmia
B. co-oximetry to detect methemoglobinemia
C. echocardiography to detect congenital heart disease
D. electroencephalography to detect a seizure disorder
E. no test because these symptoms are benign

26. You are examining a 6-month-old boy who has infantile spasms. There is a family history
of tuberous sclerosis.
Of the following, the cutaneous finding MOST likely to be present in this infant is
A. adenoma sebaceum
B. ash-leaf macule
C. forehead plaque
D. periungual fibroma
E. shagreen patch

27. A 6-year-old girl presents for evaluation of her shot stature. Chromosome analysis
reveals a 45,X karyotype that is consistent with Turner syndrome.
Of the following, the MOST appropriate evaluation to obtain is
A. echocardiography
B. hysterosalpingography
C. magnetic resonance imaging of the brain
D. pulmonary function tests
E. slitlamp ophthalmologic examination

28. A 2-year-old girl is referred for evaluation of global developmental delay and the loss of
developmental milestones. Physical examination reveals hepatosplenomegaly and coarse
facial features. Findings on computed tomography of the head are unremarkable.
Of the following, the most appropriate evaluation to obtain NEXT is
A. chromosome analysis
B. serum thyroxine
C. TORCH titers
D. urine mucopolysaccharide levels
E. urine organic acids

29. In pediatric follow-up clinic, you are discussing various aspects of cerebral palsy
with parents.
Of the following, the MOST accurate statement is that
A. cerebral palsy is a progressive disorder of neuromotor function
B. complications of labor and delivery are the leading causes of cerebral palsy
C. most cases of cerebral palsy are diagnosed by 6 months of age
D. most children who have cerebral palsy have an associated seizure disorder
E. the prevalence of cerebral palsy has risen in the past two decades

30. An otherwise healthy 10-year-old girl is brought to your office for evaluation of
unilateral breast tenderness without galactorrhea. Physical examination reveals bilateral
breast buds that measure approximately 2 cm in diameter. She has no axillary or pubic
hair.
The Sexual Maturity Rating (Tanner) stage of this girl's tissue is stage
A. 1
B. 2
C. 3
D. 4
E. 5
31. A 4-year-old boy has a 1-month history of fevers to 39°C twice a day accompanied by a
rash. He feels well during his afebrile periods. Physical examination reveals generalized
lymphadenopathy.
Of the following, the MOST common other manifestation of this boy's illness would be
A. iridocyclitis
B. headech
C. marked leukocytosis
D. presence of antinuclear antibodies
E. presence of rheumatoid factor

32. A 2-year-old girl presents with a 2-day history of cough, congestion, and bilateral red
eyes with an associated watery discharge. She denies any photophobia or pain, and there
is no edema of the eyelid.
The MOST likely diagnosis is
A. blepharitis
B. conjunctivitis
C. corneal abrasion
D. sclerouveitis
E. uveitis

33. A 3-year-old boy presents with a 3-day history of a sore throat, temperature of 38.3°C,
nasal congestion with cloudy rhinorrhea, and conjunctivitis.
Of the following, a TRUE statement about acute pharyngitis is that

A. a negative streptococcal screening test reliably excludes streptococcal infection


and obviates the need for further assessment
B. all children who have signs and symptoms of pharyngitis should be treated with
antibiotics to prevent rheumatic fever
C. oral penicillin, 250 mg tid for 10 days, is appropriate treatment in most cases
D. streptococcal pharyngitis can be differentiated from viral pharyngitis by the
presence of tonsillar exudate and cervical adenopathy
E. viruses are the cause in at least 85% of cases

34. As part of the examination of a 3-year-old girl during a health supervision visit,
you review her medical history.
Of the following, the condition that is the STRONGEST indication for routine annual
influenza immunization is
A. all healthy children
B. asthma
C. asymptomatic heart disease
D. attendance at child care
E. frequent otitis media

35. You are evaluating an ill-appearing 9-month-old child . Findings on physical examination
include a brassy cough, conjunctivitis, and coryza. The child has a temperature of 39°C
and a red, maculopapular rash on the head, trunk, and proximal extremities.
Of the following, the complication that is MOST responsible for mortality in young children
who have this condition is
A. encephalitis
B. hemorrhagic shock
C. hepatitis
D. myocarditis
E. pneumonia

.
36. A 3-day-old term infant has cyanosis, no cardiac murmur, tachypnea, and no retractions.
Pulse oximetry reveals an oxygen saturation of 60% in the right hand and 75% in the
right foot.
Of the following, the MOST likely cause of the cyanosis is
A. group B streptococcal sepsis
B. persistent fetal circulation
C. pulmonary valve atresia
D. tetralogy of Fallot
E. transposition of the great vessels

37. Iron deficiency in a child who has unrepaired cyanotic congenital heart disease is MOST
likely to increase the risk for
A. congestive heart failure
B. digoxin toxicity
C. osteoarthropathy
D. pulmonary hemorrhage
E. stroke

38. A 12-year-old girl develops low-grade fever and a nonproductive cough following
several days of sore throat and malaise. She appears tired, and crackles are audible over
the lower right lung field on chest auscultation. Chest radiography reveals patchy alveolar
infiltrates in the right lower and middle lobes.
Of the following, the diagnostic test MOST likely to identify the etiology of this illness is
A. blood culture
B. serology for Mycoplasma
C. sputum culture
D. urine antigen detection studies
E. viral culture of respiratory secretions

39. A 12-year-old girl experiences acute onset of weakness in the lower extremities. On
physical examination, the deep tendon reflexes are absent, and results of the sensory
examination are normal.
Of the following, the MOST likely findings on examination of cerebrospinal fluid are

Glucose Protein Pressure


A. decreased normal normal
B. increased increased normal
C. normal decreased normal
D. normal increased normal
E .normal normal increased

40. An 8-year-old girl is evaluated for fever and vomiting of 2 days' duration. Physical
examination reveals a temperature of 39.9°C and blood pressure of 105/68 mm Hg; the
remainder of the findings are normal. Results of laboratory evaluation include a positive
urine nitrite test, more than 100 white blood cells per high-power field, and serum white
blood cell count of 25 x 109/L (25,000/mm3), with 4% bands and 80% neutrophils. A
midstream clean catch urine culture grows 10,000 to 100,000 CFU/mL of Escherichia
coli.
Of the following, the MOST likely diagnosis in this child is
A. acute appendicitis
B. acute cystitis
C. acute pyelonephritis
D. renal abscess
E. viral gastroenteritis

41. The parents of a 5-year-old boy who has severe factor VIII deficiency hemophilia bring
him in for evaluation after he fell off his bicycle. He was not wearing a helmet. He had no
loss of consciousness and appears uninjured except for a small hematoma over the right
side of his forehead.
Of the following, the MOST appropriate next step is to
A. admit for observation without therapy
B. discharge home
C. infuse with factor VIII
D. obtain computed tomography
E. treat with desmopressin

42. You are examining a girl at her 1-year health supervision visit. Her weight, length, and
head circumference all were at the 10th percentile at birth. There were no pregnancy,
labor, delivery, or nursery complications. Physical examination reveals her weight,
length, and head circumference are at the 5th percentile.
Of the following, this child's growth parameters MOST likely represent
A. a chromosomal abnormality
B. a malabsorptive disorder
C. an endocrine disorder
D. inadequate caloric intake
E. normal growth

43. A 1-day-old term infant develops bilious vomiting and poor feeding. You recall that the
differential diagnosis of vomiting is age-related.
Of the following, the condition that is MOST likely to cause bilious vomiting in this infant is
A. gastric stress ulcer
B. gastroesophageal reflux
C. gastrointestinal food allergy
D. intussusception >> 6 months to 2 yrs
E. midgut volvulus

44. An 8-year-old girl presents to the emergency department with a history of vomiting,
weight loss, and rapid breathing. Findings on physical examination include lethargy,
tachycardia, tachypnea, cool extremities, and pallor. Initial laboratory evaluation reveals
a venous pH of 7.10 and a blood glucose concentration of 36 mmol/L (650 mg/dL).
Urinalysis is positive for glucose and ketones.
Of the following, the MOST immediate life-threatening complication of this child's illness is
A. cerebral edema
B. hyperkalemia
C.hypocalcemia
D.hypoglycemia
E.hypovolemia

45. On the second day of hospitalization, a 5-year-old child who has pneumococcal
meningitis develops a serum sodium concentration of 120 mmol/L (120 mEq/L). Physical
examination reveals an awake and responsive child whose weight is 20 kg (an increase of
1 kg from admission), temperature is 37.8°C (100°F), blood pressure is 100/60 mm Hg,
and pulse is 100 beats/min.
The MOST appropriate management is
A.administration of demeclocycline
B.infusion of a solution containing 5% glucose and 0.3% saline at a maintenance rate
C.infusion of 80 mL 3% saline over 10 minutes
D.infusion of 0.9% saline at a maintenance rate
E.restriction of fluids
46. You are examining a 12-month-old girl whose is pale looking . Her mother is concerned
about lead. Of the following, the MOST appropriate test(s) to screen this infant for lead
poisoning is(are)
A.free erythrocyte protoporphyrin (FEP)
B.FEP and plasma lead level
C.FEP and wrist radiographs
D.plasma lead level
E.plasma lead level and wrist radiography

47. A 5-year-old girl develops fever, swelling of the parotid gland, and headache.
Of the following, the BEST diagnostic test for this child is
A. bacterial culture of parotid duct secretions
B. Epstein-Barr virus serology
C. mumps serology
D. serum amylase
E. viral culture of respiratory secretions

48. The mother of a 3-year-old reports that her son has begun to cough and have difficulty
breathing. She was keeping him home from preschool today because he had a cough and
upper respiratory tract infection without fever.
The MOST likely cause of this child's symptoms is
A.aspiration pneumonia
B. bacterial pneumonia
C. foreign body aspiration
D.reactive airway disease
E.vascular ring

49. Compared with human milk, cow milk formula is MORE likely to contain a(n)
A. greater concentration of essential fatty acids
B. higher protein concentration
C. increased lactose content
D. lower calcium-phosphorus ratio
E. lower iron concentration

50. A newborn is recognized clinically to have Down syndrome. The parents are very
concerned about the disorder and its manifestations.
Of the following, the MOST important hematologic/oncologic complication is
A. aplastic anemia
B. leukemia
C. macrocytic anemia
D. platelet dysfunction
E. thrombocytosis

51. A 10-year-old boy presents with headache. His father has renal failure. On physical
examination, the boy's blood pressure is 145/100 mm Hg and pulse is 90 beats/min.
Urinalysis reveals 10 to 20 red blood cells per high-power field, and serum creatinine
level is 61.9 mcmol/L). Renal ultrasonography shows enlarged kidneys, with three renal
cysts in each.
Of the following, the MOST likely diagnosis in this child is
A. autosomal dominant polycystic kidney disease
B. autosomal recessive polycystic kidney disease
C. juvenile nephrophthisis
D. medullary sponge kidney
E. multicystic dysplastic kidney disease

_______________________________________________________________

Q1) Among the following. Life threatening anaphylaxis in children occurs most often as
a result of exposure to :
a- Exercise
b- Foods
c- Latex
d- Preoperative drugs
e- Vaccine

Q2) which of the following clinical features is most likely to be associated with a benign
condition?
a- Bleeding 7 days after a tonsillectomy
b- Bruises over the bony prominences of the extremities, both proximal and
distal
c- Epistaxis (worse in winter)
d- Hemarthrosis
e- Menstrual bleeding that last 8 days.

Q3) which of the following findings on the newborn examination may be normal?
a- Fixed S2
b- High-pitched murmur
c- Obscured S1
d- Precordial thrill
e- Systolic ejection murmur
Q4) you identify a 3-cm nonfluctuant abscess on the shoulder of a previously well 3 years
old boy who has an allergy to both penicillin and ???.Because the incidence of MRSA is
less than 5% in your community, the cephalosporins with warm compresses is likely to
provide effective treatment. The best choice is :
a- Cefaclor
b- Cefadroxil
c- Cefixime
d- Ceftriaxone
e- Cefuroxime axetil

Q5) which of the following symptoms of GERD is more common in older children and
adolescents than in infants and young children?
a- Apparent life-threatening event
b- Asthma
c- Failure to thrive
d- Hematemesis
e- Recurrent pneumonia

Q6) A previously healthy 5- year old boy presented with hematochezia. Physical
examination ?????????. the mother reports that he has had ????????? that did not affect
his activity. The most likely :
a- Henoch-schonlein purpura
b- Infective colitis
c- Juvenile polyp
d- Meckel diverticulum
e- Superior mesenteric aneurysm

Q7) postnatal immuno-prophylaxis is more effective against hepatitis B :


a- During delivery
b- In utero
c- Through breast feeding
d- Through salivary transmission
e- Via feco-oral

Q8) HBs AB with negative HBc AB :


a- Active infection
b- High viral reproduction rate
c- Immunity after immunization
d- Immunity after recovery from infection
e- Low viral reproduction rate.
Q9) most common blood type:
a- B-
b- AB-
c- AB+
d- O-
e- O+

Q10) the percentage of blood loss during surgery that typically is used to trigger
transfusion in the absence of ???? measures is :
a- 5%
b- 10%
c- 15%
d- 20%
e- 25%

Q11) A 4 week old breastfeeding boy is jaundiced and has a total billirubin concentration
of 1.3mg/dl . the laboratory test that maximizes diagnostic efficiency is:
a- Complete blood count
b- A reticulocyte count
c- Billirubin fractionation
d- Gamma glutamyl transferase
e- Hepatic Aminotransferase

Q12) ‫( مش واضح‬B) ‫الجواب‬


Q13) ‫(برضه مش واضح‬E) ‫الجواب‬
Q14) ‫(كمان برضه مش واضح‬B) ‫الجواب‬
Q15) ‫(لسه كمان برضه مش واضح‬A) ‫الجواب‬

Q16) what is the most common underlying cause for mild to moderate neutropenia?
a- Exposure to medication such as antibiotics
b- Immune neutropenia
c- Shwachman-diamond syndrome
d- Sequestration
e- Transient marrow suppression due to viral infection

Q17) which of the following statements regarding pneumonia in children is true?


a- A specific microbial pathogen usually can be identified.
b- All children who have pneumonia should be hospitalized for observation and
treatment.
c- Pneumonia is a rare cause of child mortality worldwide
d- Radiographs of the chest always should be obtained to determine the cause
e- Viral agents are the most common cause of pneumonia in older infants and
young children.

Q18) A healthy 2 month old infant was born at 32 weeks gestation. She has grown well
since birth. On physical examination of this infant, the MOST likely finding is:
a- Ability to fixate on a face and follow ???
b- Ability to reach and grasp a ???
c- Ability to watch an object and follow it to midline
d- Absence of the motor reflex
e- Dabbling and cooing vocalization

Q19) The decreased incidence of enteric infections in breastfed infants compared with
formula-fed infants is MOST likely due to the :
a- More alkaline stool pH in breastfed infants
b- Nutritional benefits of human milk on the infant’s immune system
c- Predominance of bacteroides and ?? in the gut of breastfed infant
d- Presence of antibodies against enteric infection in human milk
e- Sterility of human milk

Q20) of the following, a TRUE statement regarding use of oral rehydration solution in
the management of acute gastroenteritis is that :
a- Feedings are initiated following 24 hours of bowel rest and intravenous fluid
therapy
b- Fermented carbohydrates are resorbed in the colon
c- Fluid and electrolytes are replaced via sodium-glucose cotransport in the small
intestine
d- High sodium concentrations allows for replacement of total body sodium
deficiency
e- Hyperosmolar solutions containing complex carbohydrates are required.

Q21) A TRUE statement about glumerular filtration rate, corrected for surface area, at 2
years of age is that it is :
a- Double that observed at birth‫وهللا اعلم‬
b- Equal to that observed at birth
c- Double that observed at 6 months
d- Triple that observed at 6 months
e- Equal to that observed in adults

Q22) iron deficiency in a child who has unrepaired cyanotic congenital heart disease is
MOST likely to increase the risk for :
a- Congestive heart failure
b- Digoxin toxicity
c- Osteoarthropathy
d- Pulmonary hemorrhage
e- Stroke

Q23) A 1 day old term baby develops bilious vomiting and poor feeding. You recall that
the deferential diagnosis of vomiting is age related. Of the following, the condition that is
MOST likely to cause bilious vomiting in this infant is :
a- Gastric stress ulcer
b- GERD
c- Gastrointestinal food allergy
d- Intussusception
e- Midgut volvulus

Q24) in a patent with rheumatic fever, of the following manifestations of acute


rheumatic fever which is not relieved by salicylate or steroid therapy?
a- Carditis
b- Abdominal pain
c- Arthritis
d- Chorea >> reserpine or haloperidol + tranqulizers
e- Fever

Q25) A new born is diagnosed with congenital heart disease. You counsel the family that
the incidence of heart disease in future children is:
a- 1 %
b- 2 to 6 %
c- 8 to 10 %
d- 15 to 20 %
e- 25 to 30 %

____________________________________________________________

Pediatrics MCQ’s by Anmar Razak

1- You are called to do a routine check up on a 7 day old Rh –ve term neonate with a
weight of 3.2 Kg who was kept in the hospital because of low apgar score at birth.
Which of the following signs requires further thorough investigation:
a. Jaundice
b. Absence of Parachuting reflex
c. Absence of Grasp reflex
d. Presence of head lag
e. Positive babinski sign
f. 300 grams weight loss

2- At 5 minutes after birth, a male neonate was noted to have blue extremities with a
pulse of 90 and a shallow, irregular breathing pattern. He was also noted to be
moving all 4 limbs. He showed no response when his foot was slapped. What
would be the appropriate Apgar score for this baby:
a. 3
b. 4
c. 5
d. 6
e. 7
f. 8

3- The most important parameters in the Apgar scoring system is(are):


a. Color
b. Reflex irritability
c. Muscle tone
d. Heart rate and respiration
e. All are equally important

4- All of the following are contraindications to breast feeding EXCEPT:


a. HIV +ve mother
b. Mother with active untreated TB
c. Mother on Chemotherapy
d. Mother taking Tetracycline
e. Mother with Hepatitis B infection
f. Active Herpes in the breast region

5- Presence of which of the following is considered abnormal in a neonate:


a. Erythema Toxicum
b. Epstein’s pearls
c. Mongolian spots
d. Capillary hemangioma (stork Bites)
e. Subconjunctival hemorrhage
f. Nevus flammeus (port wine stain)

6- On a routine exam of a 1 day old newborn, a non tender swelling of the scalp was
noted. It does not cross the suture lines. What is the most likely cause:
a. Caput succedeneum
b. Subperiosteal bleeding
c. Subdural bleeding
d. Epidural bleeding
e. Subcutaneous bleeding

7- A baby is delivered following a prolonged and difficult labour due to shoulder


dysticia. On physical examination of the new born 24 hours later, it is noted that
the Moro reflex is absent on the right. What is the most likely diagnosis:
a. Erb Duchene paralysis
b. Fracture of clavicle
c. Klumpke’s paralysis
d. Transient brachial plexus paralysis
e. Cerebral palsy

8- What is the most appropriate treatment approach in this baby:


a. Send for X-ray
b. Immobilize in a figure of 8 sling
c. Observe for spontaneous resolution
d. CT scan of head
e. Surgical intervention

9- A 1 week old female infant is being examined as part of a routine general checkup.
She is a term baby and was delivered vaginally without complications. On
examination, which of the following findings requires further diagnostic or
therapeutic intervention:
a. Unilateral breast enlargement with milk discharge
b. White vaginal discharge with a small amount of blood
c. Talipes of the left foot which can be fully dorsiflexed
d. Absent femoral pulses
e. Umbilical hernia

10-On a routine physical examination, a 1 day old male infant is found to have a
palpable abdominal mass. What is the most likely cause of this mass:
a. Polycystic kidneys
b. Posterior urethral valve
c. Hepatomegally
d. Wilms tumour
e. Neuroblastoma

11-A 4 months old male infant was brought to the clinic by his mother. She has
noticed a swelling of his left scrotum. On examintion, a non tender swelling of the
left scrotum was noted. Translumination is positive. The swelling does not resolve
on compression. He was a term infant, had an uncomplicated delivery and had a
normal routine newborn examination before discharge from the hospital. What is
the most appropriate approach in this baby boy:
a. Surgical correction now
b. Observe until 1 year of age, and repair surgically if no resolution
c. Order an U/S
d. Aspirate fluid for analysis
e. Order an abdominal CT scan

12-A male newborn was delivered vaginally at 38 weeks to a mother who developed
gestational diabetes that was poorly controlled during pregnancy. All of the
following are possible complications to occur in this baby EXCEPT:
a. Hypoglycemia
b. Hyperbillirubinemia
c. Polycythemia
d. Respiratory distress syndrome
e. Necrotizing enterocollitis

13-A 4 day old female newborn is brought to the ER with a a 1 day history of
vomiting, fever and diarrhea in adition to being irritable with a hight pitched cry.
Her mother is known to be an IV drug abuser. The baby was delivered at term by
C-section due to inadequate maternal contractions. If the baby’s symptoms, which
of the following drug effects is known to cause such presentation:
a. Cocaine Intoxication
b. Heroin intoxication
c. Methadone withdrawal
d. Heroin withdrawal
e. Cocain withdrawal

14-A 12 hour old male neonate is noticed to have rapid shallow breathing with nasal
flaring. The baby was delivered at 41 weeks gestation by C- section due to
difficult labour which was later complicated by meconuim staining of amniotic
fluid. He was suctioned properly after birth an APGAR scores were reassuring. He
has no cyanosis, and Blood pressure is normal. Temp. is normal. Xray shows fluid
in the fissures with increased vascular markings. The most appropriate approach in
this patient is:
a. Intubate with artificial ventilation
b. Observe and supplemental Oxygen if needed
c. Administer surfactants to baby, with ventilatory support
d. Order an Echocardiogram
e. Insert chest tube with an underwater seal

15-Soon after birth, a female infant developes tachypnea with nasal flaring and
grunting. Physical Examination reveals hyper-resonance on the right, with absent
air entry on the left and a displaced apex beat and heart sounds to the right. No
murmurs are heard and peripheral pulses are normal. The most likely diagnosis in
this baby is:
a. Pneumothorax
b. Pneumonia
c. Meconuim aspiration
d. Diaphragmatic hernia
e. Situs invertus

16-All of the following maybe indicated in the above patient EXCEPT:


a. Nasogastric suctioning
b. Surgical intervention
c. Chest tube insertion
d. Bag and mask ventilation
e. Intubation

17-A 2 week old breast fed infant was brought to the clinic with Jaundice. He has
normal coloured stool and urine. The baby was delivered vaginally at term. The
most common cause of his jaundice is:
a. Breast milk jaundice
b. Rh incompatibility
c. Billiary atresia
d. ABO incompatibility
e. Infection

18-A 3 day old newoborn is noted to have poor feeding, irritability and vomiting. His
pulse was 170, RR is 70, temp is normal. Tense fontanelles were noted. What is
the most appropriate approach for this baby:
a. Ampicillin and Gentamicin therapy
b. Ceftriaxone therapy
c. Surgical evacuation of a hematoma
d. Steroid therapy
e. Reassurance and monitor feeding pattern.

19-the most common congenital transmitted infection is :


a. Toxoplasmosis
b. Rubella
c. CMV
d. Syphilis
e. Herpes
20-A 12 hour old male newborn is noted to have difficulty feeding. He develops
respiratory distress with choking and becomes cyanotic soon after he starts
feeding. He is normal when not feeding and during sleep. A nurse’s effort to insert
a nasogastric tube was unsuccessful. Which of the following statements is correct:
a. His pregnancy was complicated by Polyhydramnois
b. His pregnancy was complicated by Oligohydramnois
c. This baby probably has choanal atresia
d. Xray of this baby will show bowel loops in the lung fields
e. A cleft palate will be found on oral examination

21-A 1 day old male infant of a diabetic mother is seen before discharge for a
complaint of vomiting. The baby was normal at birth and was delivered vaginally
at term. He has not yet passed any meconuim. The mother states that soon after
she finishes breast feeding the baby, he starts vomiting green coloured fluid. What
is the most accurate statement:
a. Examination will reveal and olive shaped mass in the epigastrium
b. Rectal examination will reveal an empty rectal vault with rectal biopsy
showing absence of ganglion cells
c. A nasogastric tube will be seen coiled in the esophagus on Xray
d. Abdominal Xray will show a double bubble
e. Diabetes is a major predisposing factor for this condition

22-A 10 day old female infant was brought to the emergency department after the
mother noticed that over the past 24 hours the baby started crying intensely with
poor feeding. She also noticed that the baby was lethargic and says that she thinks
the baby’s abdomen was becoming a little distended. She finally brought her to the
ER when she noticed some blood in her stool. The baby was delivered at 32
weeks gestation vaginally when the mother went into preterm labour. What is the
most accurate statement regarding this condition:
a. Maternal diabetes will probably be revealed on further questioning of the
mother
b. Abdominal Xray may show pneumatosis interstinalis >> necrotizing enterocolitis
c. The source of bleed is most probably ectopic gastric mucosa in meckels
diverticulum
d. Surgical intervention is urgently indicated
e. Hirschprung disease should be highly suspected.

23-A 2 day old premature male newborn was noted to have intermittent absence of
respiratory effort of 15 seconds duration occurring every 2 minutes. He is
acyanotic, with a pulse rate of 140 and a RR of 45. The baby was delivered by c-
section at 32 weeks gestation due to placental bleeding. What is the most
appropriate management approach in this baby:
a. Reassurance
b. Full septic workup
c. Intubate and ventilate for respiratory support
d. Measure blood electrolytes and glucose, possible EEG
e. Cranial Ultrasound

24-When assessing the need of nutritional supplements for a preterm infant, which of
the following is INCORRECT:
a. Vitamen K should be given at birth
b. Iron supplement should be started earlier at 6-8 weeks of age rather than 6
months
c. Fluoride supplement is recommended at 6 months of age if inadequate in
water supplies
d. Vitamen D is indicated if low birth weight
e. Folate supplement should not be given if goat’s milk is used for feeding.

25-In protein calorie malnutrition, all of the following is true EXCEPT:


a. Marasmus usually presents with less severe weight deficit than kwashiorkor
b. Edema is a key feature of kwashiorkor
c. Dermatitis, stomatitis, depigmented hair are seen in kwashiorkor but not in
marasmus
d. Marasmus is typically seen in polygamous societies in which children are
not weaned from breast feeding until 12 months of age
e. Kwashiorkor can be precipitated after an acute intercurrent infection

26-A father brings his 12 year old daughter to your clinic. He thinks she is too short.
Measurements show that she is below the 3rd percentile for her height and age. Her
father states that she has always looked short to him when compared with her
siblings. Both her parents are of normal height. Physical examination is normal.
The single most important parameter to use in evaluating this patient is:
a. Growth chart
b. Karyotype
c. Lateral skull X-ray
d. Wrist X-ray
e. Growth Hormone measurment
f. Thyroid hormone measurement
27-A 2 months old male infant is brought to the ER. His mother states that over the
past 2 days he developed severe diarrhea and was feeding poorly. On
Examination, the child was irritable with sunken fontanelles, had dry mucosa and
was crying but no tears were seen. Blood pressure is low. Mother said that his last
urine output was very small and concentrated about 8 hour ago. He weighs 5000
grams. The most appropriate next step in this patients management is:
a. Draw blood for complete blood count, electrolytes and glucose
b. Infuse normal saline at a rate of 20 ml/kg
c. Start IV 0.33 Normal saline in 5% dextrose at a rate of 500 mls/day
d. Rehydrate using 0.45 normal saline in 5% dextrose IV over 48 hours
e. Rehydrate using 0.45 normal saline in 5% dextrose orally over 48 hours

Answer : B (TWO TYPES OF DEHYDRATION, in high Na< extracellular fluid


deficit is not obvious, oral rehydration have different Na, with glucose added to it)

28-Concerning developmental milestones, all of the following require further


investigations EXCEPT:
a. No social smile by 8 weeks of age
b. No good eye contact by 3 months of age
c. Not reaching for objects by 5 months of age
d. Not sitting unsupported by 9 months of age
e. Not walking unaided by 12 months

Note: the following is a list of important ABNORMAL milestone abnormalities:


f. No social smile by 8 weeks of age
g. No good eye contact by 3 months of age
h. Not reaching for objects by 5 months of age
i. Not sitting unsupported by 9 months of age
j. Not walking unaided by 18 months
k. Not saying single words with meaning at 18 months of age
l. No two or three word sentences by 30 months

29-A 6 year old boy was brought to your clinic by his family who have become
concerned over the child wetting his bed at night. The family states that the child
has been dry for about two years but has recently started wetting his bed,
especially at night, although daytime wetting is also noted. The family had just
relocated to this town 2 months ago. The child has no history of mental or
neurological disorders and has had a healthy childhood so far. The father says that
his son is about to join a summer camp and wants a relief for his child before he
goes away. Physical examination is normal. The most appropriate next step in this
child’s management is:
a. Reassurance
b. Urinalysis
c. Urine culture
d. Prescribe imipramine
e. Prescribe Oxybutynin

Answer: B (D AND E ARE FOR PRIMARY, ddavp and imipra. Are for nocturnal,
oxybutinin and ephedrine for daytime)

30-A 3 year old male child was brought to the ER 6 hours after he ingested 30 tablets
of ASA pills at home. He was lethargic, tachypneic and tachycardic. He had a
seizure on his way to the ER. Arterial blood gasses on this child will most
probably reveal:
a. Metabolic acidosis with respiratory compensation
b. Respiratory Alkalosis
c. Respiratory Acidosis with renal compensation
d. Metabolic Alkalosis with respiratory compensation
e. Normal PH

31-Therapeutic measures for the above patient may include each of the following
EXCEPT:
a. Activated charcoal
b. Ipecac
c. Gastric lavage
d. Alkaline Diuresis
e. Vitamen K, glucose, and Fluid support
f. Dialysis

32-Lead poisoning can present with all of the following EXCEPT:


a. Microcytic Anemia
b. Haemolytic anemia
c. Wrist drop
d. Encephalopathy
e. Decreased erythrocyte protoporphyrins

33-A mother brings her 6 month old infant to your clinic for his routine immunization
update. He is to have his third DPTP and Hib vaccine. The mother says that after
his second immunization dose, the child developed redness and swelling at the
injection site with a fever of 38C for 2 days. Acetaminophen given to her at the
time helped relieve the complications. 4 days ago he was diagnosed with acute
otitis media and is currently on a 10 day course of Amoxicillin. What is the best
approach in this child regarding his immunization:
a. Defer immunization until after the child finishes his antibiotic course
b. Adminster both vaccines only after complete resolution of Otitis media
c. Give both vaccines now and continue antibiotic therapy
d. Substitute DT for DPTP in view of the child’s previous reaction of fever,
swelling and redness and administer with Hib now
e. Give both vaccines now but stop antibiotic therapy

34-Which of the following vaccines is contraindicated in an AIDS patient:


a. DPTP
b. BCG
c. HiB
d. Influenza
e. Hepatitis B vaccine

35-Which of the following vaccines should not be given to a 4 month old infant:
a. Influenza
b. Varicella
c. Oral polio
d. BCG
e. Cholera

36-A 3 year old male infant is brought to your clinic following a 2 week history of
persistant violent coughing paroxysms that are sometimes followed by vomitting.
The mother states that 3 weeks ago the child developed sysmptoms of a common
cold with rhinorrhea, conjunctival injection and mild cough which has resolved
except for the cough. The child on examination was noted to have facial petichiae
and conjunctival hemorrhages. Lung examination is clear. A peripheral blood film
shows absolute lymphocytosis. His mother states that he did not receive any
immunizations after the age of 1 year. The most probable diagnosis is:
a. HiB pneumonia
b. Measles
c. Pertussis
d. Diptheria
e. TB

37-management of the above patient is best done by:


a. Admission to hospital and supportive care
b. Outpatient therapy with Erythromycin for patient and his family
c. Outpatient therapy with Erythromycin for patient, and booster vaccine for
entire family
d. Outpatient supportive care only
e. Outpatient therapy with Ceftriaxone for patient and Rifampin for family
38-A 5 year old child comes to your clinic with his mother who states that the child
started developing a rash 5 days ago. She states that the child first had symptoms
of upper respiratory tract infection with cough and conjunctivits about 10 days
ago. On examination, the child was febrile, with macular rash on the head and
trunk. She was also noted to have whitish leasions on the buccal mucosa. There is
no lymphadenopathy. The most common complication of the above condition is:
Case >> measles
a. Otitis Media
b. Encephalitis
c. Subacute sclerosing panencephalitis
d. Pneumonia
e. Myocarditis
f. Arthritis

39-A mother brings her 1 year old infant to your clinic complaining of a pink
coloured slightly raised rash that developed over the past 7 days covering the face,
neck and extremities. She says that the child first developed a high fever of 40C 10
days ago. The fever subsided three days later only to be followed by the current
rash. Examination shows, in adition to the rash, Occipital lymphadenopathy. The
child is afebrile. The most probable diagnosis is:
a. Measles
b. Rubella
c. Roseola
d. Scarlet fever
e. Erythema infectiosum

40-A 9 year old boy has microscopic heamturia. A maternal uncle receives chronic
dialysis to for treatment of glomerulonephritis. Which of the following findings
would be most supportive of a diagnosis of a hereditary renal disease:
a. Cherry red spot of macula
b. Chronic hypertension
c. Duplicated renal collecting system
d. Multiple café au lait spots
e. Sensorineural deafness

41-A 3 months old female infant is suspected of having Turner’s syndrome, which of
the following findings on physical Examination is most suggestive of this
condition:
a. Head circumference of 31 cm, hepatomsplenomegally and absent uterus
b. Head circumference of 36 cm with short arms and legs
c. Hypotonia, high pitched cry and cyanosis
d. Low set ears, short neck and clubbed feet
e. Webbed neck, pedal edema and poor femoral pulses.

42-An 8 year old had a tonsillectomy and adenoidectomy 10 days ago, she has
complained of pain in the right ear for the past week. Examination of the ear is
normal. Which of the following is the most likely diagnosis:
a. Infected tooth
b. Nasopharyngitis as a complication of the surgery
c. Referred pain from pharynx
d. Subclinical otitis media due to edema of the Eustachian tube orifice
e. Temporomandibular joint pain referred to the ear.

43-A child born at 32 weeks gestation is brought to your office at 2 months of age for
a routine check up. The child has an upper respiratory tract infection but is
afebrile. Family history of seizure disorder is also present. What decision should
you make regarding the child’s first vaccination:
a. Wait until the child is two months older to administer the first vaccination
b. Wait until the child is clinically well to administer the first vaccination
c. Administer the first vaccination now
d. Consult a peadiatrician regarding the effect of the family’s seizure disorder
e. None of the above

44-A 23 month old boy has had two episodes of otitis media during the past year and
five episodes of bronchitis with wheezing during the past 6 months. The
appropriate first step in evaluating this child would be to:
a. Ask the parents about smoking in the houshold
b. Determine immunoglobulin levels
c. Place the child on prophylactic antibiotic therapy
d. Refer the child to an allergist
e. Perform an HIV Elisa test

45-a 4 year old previously healthy girl has had abdominal pain for 12 hours. There
has been no diarrhea or vomiting. She appears ill and has grunting respirations.
Vital signs are RR 50, HR 140, TEMP 39.5. She also has some neck stiffness.
Examination of the abdomen reveals tenderness with mild guarding in the right
upper quadrant. Which of the following is the next appropriate investiation:
a. A barium Enema
b. A cholecystogram
c. A chest Xray
d. An upper gastrointestinal Series
e. CT scan of the abdomen
46-A 7 year old has a persistant night time cough for 1 year. The most likely cause of
the patients cough is:
a. Foreign body aspiration
b. Immotile cilia syndrome
c. Psychogenic cough
d. Reactive airway disease
e. TB

47-A 5 year old boy has a seizure that began 45 minutes ago. Initial treatment
includes O2 and anticonvulsant. History and physical examination reveal no
precipitant cause. Which of the following tests would be most urgent to obtain:
a. Toxicology screen
b. EEG
c. CT scan of head
d. Serum Glucose
e. Serum Monia and lactic acid

48-A 4 year old girl has complained of joint pain for 6 weeks. Physical examination
reveals swelling and tenderness of the knees and right ankle. All other findings are
normal. Lab studies reveal a normal complete blood count, normal ESR, - ve RF
and + ve ANA. Patient with such findings are most likely to develop which of the
following: ( juvenile idiopathic arthritis)
a. Pericarditis
b. Iridocyclitis
c. Scaroiliitis
d. Severe destructive joint disease
e. Enthesitis

49-A 2 year old boy has had fever to 39C, irritability and drooling for 1 day. Physical
Examination reveals the uvula to be in the midline but erythematous and swollen.
The epiglottis appears normal on lateral radiograph of the neck. The most likely
causative organism in this patient is:
a. Coxackie virus
b. Heamophilus influenza type B
c. Non typeable H. flue
d. Pseudomonas aerogenosa
e. Staph. Aureus

50-A 9 year old boy had had fecal soiling for several years. He denies associated
illness, abdominal pain, or constipation. Physical examination reveals a left sided
mass. Rectal examination reveals stool present with a normal anal tone. Which of
the following is the most likely explanation for this boy’s soiling problem:
a. Chronic constipation with stool impaction
b. Hirschprung disease
c. Hypothyroidism
d. Left sided colon tumour
e. Psychiatric dysfunction

51-A case of Kawasaki disease is suspected in a 4 year old male child. Physical
examination will likely reveal each of the following EXCEPT:
a. Cervical lymphadenopathy
b. Edema of hands and feet
c. Fissured lips
d. Polymorphous rash
e. Strawberry tongue
f. Purulent conjunctivitis

52-the most important complication of Kawasaki disease is:


a. Coronary artery aneurism
b. Arthritis
c. Secondary infections
d. Strokes
e. Seizures

53-A 9 year old girl is seen in the ER because of another episode of recurring
abdominal pain that she has had for the last 4 months. Which of the following
clinical features is LEAST suggestive of an organic cause for this recurrent pain:
a. Pain that is localized to the left flank
b. She has missed three weeks of school in the past four months
c. Recurrent fever of 38 C
d. Recurrent Nausea and vomiting
e. Intermittent diarrhea

54-A 2 year old girl who is in day care program has had persistent diarrhea for the
past 6 weeks. The stools are described as watery, green, occurring 4-5 times per
say with neither blood nor mucus. She has not gained nor lost any weight. Her
height and weight are both between the 25th and 50th percentile. Physical
examintion is normal. Which of the following is the most likely diagnosis:
a. Celiac disease
b. Giardia Lamblia infection
c. Crohn’s disease
d. Milk allergy
e. Pancreatic insufficiency
55-In addition to E coli and proteus species. The organism most likely to cause
urinary tract infections in children is:
a. Candida Albicans
b. Enterococcus species
c. Klebsiella species
d. Psudomonas
e. Staph. Saprophyticus

56-A 16 year old boy presents with a 6 month history of detrioration in school
performance, fatigue and now has become jaundiced. The mother states that his
handwriting has become illegible. His liver is enlarged and hard and the spleen is
also palpable. What is the most likely diagnosis:
a. Autoimmune hepatitis
b. Hepatitis A
c. Hepatitis C
d. Wilson Disease
e. Primary sclerosing cholangitis

57-On examination, a 2 month old infant is found to have a liver 4 cm below the
costal margin with a respiratory rate of 70/ minutes and a pansystolic murmur in
the 4th intercostal space near the left sternal border with a rumbling mid-diastolic
murmur at the apex. Which of the following congenital abnormalities is most
likely to be present in this child:
a. Transposition of the great vessels
b. Coarctation of the aorta
c. VSD
d. Teralogy of fallot
e. Isolated ASD

58-A child has fever, sore throat and bilateral non purulent conjunctivitis. Which of
the following organisms is the most likely cause:
a. Adenovirus
b. CMV
c. Beta hemolytic group A strep.
d. Parainfluenza virus
e. Respiratory syncetial virus

59-A 2 day old breast fed newborn who was delivered at home presents to the ER
with oronasal bleeding. The mother has been on phenytoin during her pregnancy.
The most appropriate management in this patient is to:
a. Administer factor VIII
b. Administer 1-2 mg of vitamin K
c. Administer 20 ml/kg whole blood
d. Adminster platelets
e. Adminsiter 10ml/Kg packed cells

60-Which of the following is most important in the pathogenesis of Otitis Media:


a. Viral infection of middle ear
b. Bacterial infection of pharynx
c. Obstruction of Eustachian tube
d. Impaired cellular immunity
e. Feeding formula rather than breast milk

61-the most important goal for rubella immunization is to prevent:


a. Complications of rubella in children
b. Rubella arthritis in adults
c. Subacute sclerosing panencephalitis
d. Congenital rubella syndrome
e. Post rubella encephalitis

62-A 15 month old infant presents with the sudden onset of rectal bleeding with large
amounts of blood mixed with loose stools. Each of the following should be
considered in the differential diagnosis EXCEPT:
a. Anal fissures
b. Meckel’s diverticulum
c. Intussusception
d. Intestinal polyps
e. Salmonella enteritis

63-Complications of bacterial meningitis in children include each of the following


EXCEPT:
a. SIADH
b. Deafness
c. Cerebral palsy
d. Seizures
e. Intraventricular haemorrhage

64-A six year old girl comes to your office for a routine annual check up. On physical
examination, you find that she has a heart murmur. Each of the following findings
would support a diagnosis of a pathological murmur EXCEPT:
a. Murmur occurs in diastole
b. Murmur is associated with a thrill
c. Child has mild finger clubbing
d. Murmur is louder in supine and softer when child sits upright
e. There is a loud P2 component of the heart sounds
65-A child who has ingested a large quantity of iron tablets will demonstrate each of
the following EXCEPT:
a. Hemorrhagic vomiting
b. Hemorrhagic diarrhea
c. Status epilepticus
d. Metabolic acidosis
e. Radiopaque particles in GI tract

66-Infants have increased risk of hyperblirubinemmia with each of the following


conditions EXCEPT:
a. Infants with sepsis
b. Premature infants
c. Infants with congenital hypothyroidism
d. Infants with meconium aspiration syndrome
e. RH positive infants born to RH negative mothers

67-Prostaglandin E2 infusion can be used for the intial management of each of the
following neonatal heart leasions EXCEPT:
a. Pulmonary atresia
b. Coarctation of aorta
c. Tetralogy of fallot
d. Patent Ductus arteriosus
e. Hypoplastic left heart syndrome

68-A 6 week old infant presents with recurrent vomiting, each of the following is
consistent with a diagnosis of pyloric stenosis EXCEPT:
a. Non bilious vomiting
b. Hypochloremic metabolic alkalosis
c. Palpable epigastric mass
d. Persistent unconjugated hyperbilirubinemia
e. Abdominal distention

69-The average 4 year old has 8 to 10 respiratory tract infections per year. Factors
contributing to this rate include each of the following EXCEPT:
a. The large number of different respiratory viruses
b. Lack of long lasting immunity after many respiratory viruses
c. Frequent Exposure to other children
d. Chronic excretion of respiratory viruses for months after infection
e. High degree of contagiousness of most respiratory viruses.

70-A 2 year old boy is brought to the ER with high fever, cough and respiratory
distress. Examination confirms right lung consolidation. There is no
lymphadenopathy and no splenomegally. He is also noted to have small tonsils. his
weight is below the 3rd percentile for his age. Review of records reveals history of
repeated pneumonias, sinusitis and otitis media since the age of 9 months. His
immunization record is not available. Investigation later shows sub normal levels
of T lymphocytes and low levels of circulating B lymphocytes associated with
extremely low levels of IgG, IgM, IgE and IgA. What is the most likely diagnosis
in this patient:
a. AIDS
b. Child neglect (child abuse)
c. Incomplete immunization
d. Inherited Immune deficiency
e. Transient hypogammaglobulinemia of infancy

71-A 4 year old male child is seen by the doctor because of history of recurrent
respiratory tract and urinary tract infections associated with chronic diarrhea. His
investigation reveals low levels of IgA with normal levels of IgG, B and T cells.
Which of the following statements is correct:
a. Immunization with live attenuated vaccines should not be given to this child
b. Blood and Blood products should be carefully screened before administering
if this patient needs transfusion for any reason
c. This condition could have been prevented by careful screening of mother
during pregnancy
d. Adenosine deaminase deficiency is the defect found in most of these cases
e. NADPH Oxidase deficiency will be found on further testing

72-a 90 days old female child was brought to your clinic after her mom noticed that
the cord stump is still not separated. The child was born at term with no
complications. Examination is normal. Which of the following actions is
appropriate in this patient:
a. Reassure the mother that this is normal and it will separate on its own in 1
year
b. Surgically remove the stump
c. Investigate for immunological defects
d. Perform an HIV ELISA test
e. Send the child to the hospital for surgical closure of this umbilical hernia

73-A 3 week old baby boy was brought to the ER after experiencing a seizure. After
stabilization, the baby was noted to have wide spread eyes, low set ears, and a
narrow mouth opening with a small mandible. He had normal levels of Na, K, and
Glucose. Serum calcium was below normal. What is the most likely cause of his
seizure:
a. Fetal alcohol syndrome
b. DiGeorge syndrome
c. Wiscott Aldrich syndrome
d. McCune Albright syndrome
e. Congenital hypoparathyroidism

74-A 4 year old boy is brought to the ER 24 hours after he developed respiratory
distress. His parents state that the child has become increasingly agitated with
cough, shorness of breath and fever. Examination shows a child in moderate
respiratory distress with wheezing and stridor. There is decreased breath sounds on
the right with increased percussion note. Examination of the sputum shows small
amount of blood. Xray shows hyperlucency of the right lung field with signs of
hyperinflation especially on expiratory film. The child has no previous health
problems and his immunization record is complete. There are no pets nor smoking
around the house. What is the best next step in the management of this patient:
a. Give O2 with nebulised salbutamol and start treatment with IV
hydrocortisone
b. Send sputum for culture and start Empiric antibiotic treatment
c. Perform rigid bronchoscope
d. Intubate patient and insert chest tube on the right
e. Perform chest CT scan

75-A 2 year old female is brought to the ER with a 24 hour history of respiratory
distress consisting of cough with no sputum and fever. The parents indicate that
the child had some sneezing and rhinorrhea about 7 days ago but seems to have
progressed to the current condition. On examination, the child was lying in bed
and appears in moderate distress with interminttent stridor and a brassy barking
cough. There is no wheeze and breath sounds are equal bilaterally. A chest Xray
will most probably show which of the following characterstic signs:
a. Hyperinflation of the lung fields
b. Lateral thumb sign >> epiglottitis
c. Steeple sign >> croup
d. Supraglottic stenosis >> tracheitis
e. Diffuse oppacification of the lung fields

76-the best course of action for the above patient is:


a. Observe in the ER with O2 support
b. Send home and instruct parents to ameliorate symptoms with with vapourr
steam and keeping the child calm
c. keep in ER and give racemic epinephrine and corticosteroid IM plus O2
support
d. Intubate patient and start Ceftriaxone therapy
e. Admit patient to hospital for ventilatory support
77-A 15 months old baby boy is brought to the ER with a 24 hisotry of respiratory
distress with cough, fever and wheezing that followed a 4 day period of sneezing
and rhinorrhea. There is no history of similar episodes in the past. On examination,
the child is lying in bed with severe respiratory distress evident by tachypnea with
nasal flaring, intercostal recession and wheeze. There is no stridor. Pulse
Oxymetry shows an O2 saturation of 90% Temp is 38 C. Chest Xray shows
hyperinflation. What is the most likely diagnosis:
a. Reactive airway disease
b. Respiratory Syncytial virus infection
c. Parainfluenza virus infection
d. Foreigh body aspiration
e. Acute bronchitis

78-Management of the above diagnosis may, in general, include all of the following
EXCEPT:
a. Fluid adminstration
b. Bronchodilators
c. Ribavirin
d. RSV immune globulins
e. Steroids

79-a 4 year old girl presents to the ER with a 24 hour history of fever and respiratory
distress. She has newly relocated to this town with her parents. Her immunization
record is not available. History from her parents reveals that the child was born at
term with no complication during delivery but failed to pass meconuim in the first
48 hours of life. She also later developed rectal prolapse and required
hospitalization. Examination reveals nasal polyps and suggests left lung field
consolidation with rales. What is the most likely underlying mechanism causing
her current presentation:
a. Thick mucus secretions
b. Abnormally cilliary movements
c. Incomplete immunization
d. Immune deficiency state
e. Increased chloride absorption

80-Which of the following congenital heart disease does not cause cyanosis:
a. Tetralogy of fallot
b. Transposition of great vessels
c. Truncus arteriosis
d. Ventricular septal defect
e. Coarctation of the aorta
81-A 12 hour old infant born at 32 weeks gestation was found to have a continuous
murmur over the left sternal border. There are no other findings. What is the best
action to take in this child:
a. Give endomethacin and restrict fluid intake
b. Transfer to surgery for closure of defect
c. Transfer to surgery for ligation of PDA
d. Give PGE1
e. Observe for spontaneous resolution

82-A 12 hour baby boy born at 38 weeks gestation was found to be cyanotic 12 hours
after birth. He was tachycardic and tachypneic with no fever. Xray showed a
narrow upper mediastinum with a heart shadow having an appearance of an egg
lying on its side. What is the best course of action in this patint:
a. Immediate surgical repair of a VSD
b. surgical ligation of a PDA
c. infuse PGE1 to keep ductus arteriosus open
d. Give endomethacin to close PDA
e. Give Oxygen and observe for improvement

83-features of tetralogy of fallot may include each of the following EXCEPT:


a. VSD
b. Overriding aorta
c. Right sided aortic arch
d. Right ventricle outlet obstruction
e. Hypertrophy of left ventricle

84-A 5 year old child presents to the ER with a 24 hour history of progressive
shortness of breath, fever and fatigue, and tachypnea. Parents indicate that he
developed symptoms of common cold around six days ago. Examination reveals a
child in moderate distress with sweating and cool extremities. The child has
tachycardia, tachypnea, and hepatomegally. A pansystolic murmur is found. The
child has been previously normal with no history of respiratory or cardiac disease.
What is the most likely diagnosis:
a. Myocarditis
b. Rheumatic fever
c. Infective endocarditis
d. Coxackie A virus infection
e. Undiagnosed coarctation of the aorta

85-the most common cause of acute diahrrea in children is:


a. infection with E Coli
b. Infection with Rota virus
c. Infection with salmonella
d. Manifestation of systemic infections
e. Side effects of antibiotics

86-A 5 month old is brought to your clinic with a complaint of the child developing
cough and wheeze and sometimes apnea after feeds. The mother states that the
child has also been spitting up and vomitting since he was 1 month old. The
mother describes posturing of the child consistent with sandifer syndrome. The
child is at the 5th percentile for weight. What is the most likely cause of the
patients symptoms:
a. Tracheoesophageal fistula and atresia
b. Pyloric stenosis
c. Gastroesophageal reflux
d. Diaphragmatic hernia
e. Tracheomalatia

87-the most common cause of lower GI bleed in a child less than 1 year is:
a. Intussusception
b. Colonic polyps
c. Meckel diverticulum
d. Anal Fissure
e. Volvulus

88-an 10 months old infant is brought to the ER 12 hours after he started having
colicky abdominal pain. The mother states that the child was diagnosed with viral
gastroenteritis 7 days ago. He had since recovered well until today when he started
having severe pain with vomiting and a slight fever. There is no diarrhea and the
last bowel motion 8 hours ago was normal, although it seemed to have temporarily
relived the pain. On examination, a listless child is seen with a tender abdomen. A
sausage shaped mass is felt in the upper abdomen associated with an empty left
lower quadrant. Digital rectal exam shows normal stool which is heme negative.
What is the best next step in the management of this patient:
a. Abdominal U/S
b. Baruim enema
c. Emergency laporotomy
d. Sigmoidoscopy
e. Observation with fluid support

89-4 year old girl presents to her clinician with dysurea. A diagnosis of UTI is made.
This was her first presentation with a UTI. What is the best course of action to take
in this patient:
a. Treat her infection as outpatient, and treat recurrence as it occurs, no need to
investigate
b. Treat her infection as outpatient, reculture urine at end of treatment and
perform renal US
c. Treat her infection, fully investigate only if recurrence occurs
d. Admit to hospital for treatment and immediate VCUG
e. Treat her infection and place patient on long term prophylaxis, no need for
investigation

90-After an abnormal U/S following the first episode of a UTI in a 3 year old, a
VCUG showed grade 3 vesicoureteric reflux. No renal scarring is seen on IVP.
What is the best action to take:
a. Place child on nitrofurantoin therapy and wait for spontaneous resolution
b. Treat only if infections occurs
c. Refer for surgical correction immediatly
d. No need for intervention at this time
e. Re evaluate in 1 month, and surgically correct if no resolution

91-All of the following are poor prognostic factors associated with Acute
Lymphoblastic Leukemia EXCEPT:
a. Male
b. Black
c. WBC > 100, 000/mm3
d. CNS involvement
e. Age between 2 and 10

92-Most common solid tumor in children is:


a. Wills Tumor
b. Brain Tumor
c. Neuroblastoma
d. Leukemia
e. Lymphoma

93-a 2 year old child was brought to the ER after having a seizure. The parents who
accompanied the child stated that the child had a fever of 38 the night before
associated with earache. The parents were planning to bring the child to his
physician today but because of the seizure, they decided to take him to the ER.
The seizure lasted 5 minutes, was a generalized tonic clonic and the child
recovered rapidly after the seizure ended. There is no family history of seizure
disorder. Examination reveals bulging, immotile and hyperemic right tympanic
membrane with a temp. of 39C. What is the most appropriate next step in
management:
a. Reassurance, antipyretic and antibiotic
b. Antibiotic, Anticonvulsant and EEG
c. Lumbar puncture, Anticonvulsant and Antipyretic
d. Reassurance, anticonvulsant and EEG
e. Lumbar puncture, Antibiotics and Anticonvulsant
f. EEG, Lumbar puncture and anticonvulsants

94-A 6 months old infant is brought to the clinic by his parents. They state that over
the last 2 months, they noticed that the child was having episodes of symmetric
rhythmic contractions of the trunk and extremities that were very brief but very
frequent. Suspecting a particular cause, the doctor orders an EEG, which showed
hypsarrhythmia. What is the best treatment modality for this condition:
a. Treatment with Phenytoin
b. No treatment required as spontaneous resolution is expected
c. Treatment with thyroid hormone
d. Diagnoses of the underlying condition is required before treatment can be
decided
e. Treatment with adrenocorticotropic hormone

95-In cerebral palsy, which statement is CORRECT:


a. Progressive motor deterioration is a common feature
b. Birth asphexia is a very common cause
c. Prematurituy does not increase risk
d. Has a higher incidence of scoliosis
e. Spastic cerebral palsy occurs in less than 30% of cases

______________________________________________________________________

Nutrition
1-Good signs of dehydration
A) palpation anterior fontanel
B) tachycardia
C) delayed capillary refill
D) hypotension
E) lethargy, coma

2- Diet Management in Nephrotic syndrome include


A) protein 2 g/kg
B) Fat more than 30 % of total cal.
C) cholesterol more than 200 mg per day
D) low in polyunsaturated fatty Acid
E) Do not administer Iron unless there is clear evidence of Iron deficiency
3- All of the following true for Diet Management in Diabetes Mellitus EXCEPT?
A) Regular food pattern should be emphasized
B) Caloric intake 40% CHO
C) Encouraged low salt.
D) Encouraged low saturated fats and high fiber diet.
E) Distribute carbohydrate load evenly during the day preferably 3 meals & 2 snacks

4- All of the following Indications of Total PARENTERAL NUTRITION EXCEPT?


A) Esophageal atresia
B) Severe FTT
C) Severe burn
D)Inflammatory Bowel Disease
E) BMI<25

5- All of the following Comlication of Total PARENTERAL NUTRITION


EXCEPT ?
A) Hypoglycemia
B) Hyperglycemia
C) Hypocalcaemia
D)Metabolic alkalosis
E) Metabolic acidosis

6- All of the following associated with food allergy EXCEPT?


A) Angioedema
B) Vomiting
C) Fever
D) Laryngeal edema
E) Abdominal cramps

7- All of the following associated with Familial Hypercholesterolemia EXCEPT?


A) Autosomal co-dominant disorder
B) Elevated LDL cholesterol
C) Tendon xanthomas
D) Elevated HDL cholesterol
E) Incidence 1/5000
F)D+E

8- All of the following associated with Familial Hypertriglyceridemia EXCEPT?


A) Autosomal co-dominant disorder
B) It is characterized by elevation of plasma triglycerides
C) May be effected coronary vascular system
D) Incidence ≈1/500 individuals
E) Decreased levels of chylomicrons
9- All of the following SECONDARY CAUSES OF Hypercholesterolemia EXCEPT?
A) Hyperthyrodism
B)Nephrotic syndrome
C)Cholestasis
D)Anorexia nervosa
E)Tegretol

10- All of the following risk factor for coronary heart disease EXCEPT?
A) Elevated HDL cholesterol
B) Smoking
C) Obesity
D)Malnutrition
E)B- Blockers

11- All of the following associated with bacterial rather than viral GE EXCEPT?
A) Visible blood mixed with faeces
B) Febrile seizure
C) Diarrhoeal frequency more than 5 stool per day
D)Full blood count showing Hb 5.2 g/l,WBC14.5 PLT 102
E)Pulse rate 80/min. in a child of 2 years

12- All of the following causes the onset of persistent vomiting in 3 week old child?
A)Disaccharidase intolerance
B)Duodential atresia
C) Pyloric stenosis
D)Hiatus hernia
E)Choledochal cyst
F)C+D

13-A reduced red cell folate may be found


A)In celiac disease (gluten enteropathy)
B)After resection of the terminal ileum
C) After long term treatment with valporal
D) After 2 weeks of a low folate diet
E) Where there is deficiency of B 12
F)A+C

14- All of the following associated with infantile GE EXCEPT?


A)Is commoner in bottle-fed babies than breast-fed babies
B) In frequently due to rotavirus
C)May be complicated by subsequent Disaccharit
D)Antibiotic not indicated
E)Is associated with pathogenic E.Coli in the majority of cases

15- All of the following associated with Congenital Pyloric stenosis EXCEPT?
A)Familial tendency
B) Crumb in abdomen
C) Alkaline urine
D)Increase incidence in boys
E) Dehydration

16- All of the following are recognized causes of nonorganic failure to thrive ?
A) Maternal eating disorders
B) Maternal depression
C) Inadequate housing
D) Inadequate social support
E)Lack of extended family
F)A+B

17- All of the following are thought to be long term out comes of early nutritional deficiency?
A)Short stature
B) DM
C) Ischaemic heart disease
D) Obesity
E)Obstructive lung disease

18- All of the following are known disadvantages of milk formulae EXCEPT?
A) Approximately 25%of infants with cows milk protein intolerance with developsoy intolerance
B) Soy milk formulae tastes worse than elemental milks
C) Infants respond poorer to vaccination than elemental milks
D) Infants have lower complement levels than elemental milks
E) Infants have lower aluminum levels than elemental milks

19- All of the following causes of noninfective diarrhea EXCEPT?


A) Lead poisoning
B) Hypothyrodism
C)Hirschprungs disease
D)Gliadin sensitivity
E)Feeding difficulty

20- All of the following are true regarding the diagnosis of cows milk intolerance EXCEPT?
A)Skin test usually confirm the diagnosis
B) RAST test of >grade 2 makes the diagnosis highly likely
C) Bloodly diarrhea excludes the diagnosis
D)It is common in breast-feed infants
E)Rechallenge should be extremely caution us in the comptred with bottle-fed

21-Breast feeding is redatively protective against


A)Late haemorrhagic disease of the nowborn
B) Maternal breast cancer
C) Late onset diabetes
D)Prolonged jaundice
E)Under feeding

22-Malabsorption syndrome with primary mucosal abnormally inclue


A) Pancreatic insufficiency
B) Cron s disease
C) Abetalipoproteinemia
D)Blind-loop syndrome
E)Coeliac disease
F)B+C+E

23- Breast milk is more rich than Cow’s milk in:


A) Lactalbumin
B) Sodium.
C) Calcium.
D)Iron
E)Folic acid

24- All of the following are true Breast feeding EXCEPT?


A) Recommended food for infants both term and preterm
B) 50%of energy from proteins
C) Contains immunological benefits (Ig A ,active lymphocyte)
D)Promotes growth of lactobacillu in GI
E)Decreases incidence of allergy disease

25-Signs and symptom of 5% dehydration?


A) Oliguria , tear with crying ,loss active than usual .normal skin turgor , moist oral mucosa
B) Oliguria ,no tears with crying ,loss active than usual,sticky oral mucosa, normal or
slightly diminished skin turgor
C) Oliguria ,no tears ,sunken eyes, slightly diminished skin turgor
D) Oliguria , sunken eyes, tenting ,tachycardia , hypotension

26-Common allergy disease in school age ?


A) Atrophic dermatitis
B) Food allergy
C) Asthma
D)Allergic rhinitis
E) Drug allergy

27- Cause of Food allergy?


A)Recurrent dizziness after eating Chinese foods
B) Recurrent tingling sensation in the mouth after eating apiece of apple
C) Recurrent palpitations after drinking a cup of coffee
D) Recurrent diarrhea after drinking a glass of milk

28- Calculate fluid .,Na, K for baby has 40 kg EXCEPT?


A) Fluid 1900 ml
B) Na 120 meq
C) K 80 meq
D) A,B,C
E) Na 80 meq

29- All of the following are wrong for dehydration EXCEPT?


A) Capillary refill less than 2 second mild dehydration
B) Absent tear mild dehydration
C) Drowsy moderate- mild dehydration
D) Sunken eyes mild dehydration
E) Increased urine output sever dehydration

30- All of the following are true for hypernatramia EXCEPT?


A) The patient look not dehydration
B) Doughy skin
C) Serum Na 155 meq \L or more
D) The fluid given 1\2 of deficit with 8 h
E) Convulsion hypocalcaemia can occur

31- All of the following are nutrition delivery enteral feeding EXCEPT?
A) Oral feeding
B) NOT
C)Subcutaneous infusion
D) Gastrostomy feeding
E) Jejenostomy feeding

32. Breast milk is more rich than Cow’s milk in:


A) Carbohydrates.
B) Proteins.
C) Sodium.
D) Calcium.
E) Phosphorus.

33. All are early findings in Rickets EXCEPT:


A) Normal serum calcium.
B) High serum phosphorus.
C) Radiological changes.
D) Craniotabes.
E) Rachitic rosary.

34-Skeletal manifestations of advanced rickets include:


A-Head bossing.
B-Delayed dentition.
C-Marfan’s sign.
D-Harrison’s sulcus at thorax.
E-All of the above.

35-Causes of death in kwashiorkor include:


A-Intercurrent infection.
B-Water and electrolyte disturbances.
C-Hypoglycemia.
D-Hypothermia.
E-All of the above.

36.Recognized clinical pictures of marasmus include:


A-Growth failure.
B-Hunger.
C-Loss of subcutaneous fat.
D-Muscle wasting.
E-All of the above.

37-Adequate diet should supply:


A-Adequate amount of water.
B-Adequate calories.
C-Adequate carbohydrates, fat & proteins.
D-Adequate amount of fiber.
E-All of above

38-Contraindications of breast feeding include all of the following except:


A-Active Tuberculosis.
B-Severely undernourished mother.
C-HIV infection.
D-Premature infant.
E-Abnormalities of the mouth of the newborn.
39-Constant findings of clinical pictures of kwashiorkor include:
A-Growth failure.
B-Mental changes.
C-Edema.
D-Disturbed muscle-fat ratio.
E-All of the above.

40- Weaning means:


A-Stop breast feeding completely and start normal food.
B-Stop breast feeding completely and start formula feeding like Nan or Materna milk.
C-Introduction of foods other than milk in the infant’s diet.
D-Usually started at the age of 10-12 months.
E-None of the above.

41-The following advantage of breast milk except:


A. Is chosen in preference to a preterm formula in< 30 wk.
B. Should be avoided during episodes of GE.
C. Contains Ig A.
D. Has lower phosphate content than cow's milk.
E. Helps prevent GE.

42- A 4-month-old boy weighed 3500 g at birth. He now weighs 4.5 kg. He has been formula
fed since1 week of age.The MOST likely reason for this child's failure to thrive is:
A) absence of solids in his diet
B) exaggerated parental concern about overfeeding
C) improper feeding technique
D) omission of supplemental vitamins
E) withholding of nighttime feedings

43-You are asked to help prepare an oral rehydration solution.You recall that, in
addition to water, the MOST important components of such a solution are:
A) bicarbonate and potassium
B) chloride and potassium
C) lactate and potassium
D) sodium and fructose
E) sodium and glucose

44-Which of the following vitamins is in higher concentration in cow's milk than in human
milk?
A. A
B. C
C. E
D. K
E. B6

45- in vitamin deficiencies all are correct EXCEPT?


A. xerosis conjunctivae in vitamin A deficiency.
B. tender nerves in vitamin B1 deficiency.
C. photosensivity in niacin deficiency.
D. seizures in pyredoxin deficiency.
E. cerebellar ataxia in riboflavin deficiency.

46- A TRUE statement regarding anthropometric measurements in the assessment


of nutritional status is:
A) Acute changes in weight reflect changes in muscle mass
B) Arm circumference is the best screening tool for malnutrition
C) Single measurements are the most sensitive indicators of nutritional problems
D) Standard growth curves are equally applicable to all ethnic groups
E) Standard growth curves overestimate the early gains to be made by breastfed infants

47- Of the antibodies found in human colostrum and milk, the immunoglobulin (Ig) that is
MOST likely to prevent organisms from adhering to the infant's intestinal mucosa is:
A) IgA
B) IgD
C) IgE
D) IgG
E) IgM

48- Advantages of breast milk include all of the following except:


A-Economic and cheaper than artificial milk.
B-Sterile & free from contamination.
C-Contains antibodies.
D-Contains protein higher than artificial milk.
E-Has psychological advantages.

Neonatology

1- A 2-week-old infant is jaundiced. Findings include weight and length at the 75th
percentile for age; icterus; with hepatosplenomegaly; total bilirubin, 6.3 mg/dL;
direct bilirubin, 5.5 mg/dL; alanine aminotransferase activity, 130 U/L; aspartate
aminotransferase activity, 143 U/L; and gamma-glutamyl transpeptidase activity,
950 U/L.
Of the following, the BEST study to evaluate the excretion of bile from the liver is

A. computed tomography of the liver


B. hepatic ultrasonography
C. hepatobiliary scintigraphy
D. measurement of galactose-1-phosphate uridyltransferase activity
E. measurement of the serum alpha1-antitrypsin level

2- Most authorities encourage the early introduction of human milk in the very-
low-birthweight (VLBW) infant. However, mothers must be informed early in the
feeding process that supplementation of their milk with protein and other nutrients
may be necessary.

The MOST likely explanation for why protein supplementation of human milk
often is required in the VLBW infant is that
A. human milk contains less than half the protein of cow milk formula
B. the hepatic metabolism of protein is ineffective in most preterm infants
C. the protein in preterm human milk is of poor nutritional quality
D. VLBW infants have excessive gastrointestinal losses of ingested protein
E. VLBW infants require an increased protein intake because of their high
catabolic rate

3- Which of the following constellations of features BEST describes the fetal


alcohol syndrome?
A. Elfin facies, irritability, and supravalvular aortic stenosis
B. Growth deficiency with microcephaly, developmental delay, and short
palpebral fissures
C. Intrauterine growth retardation, triangular-shaped face, and clinodactyly of
the fifth finger
D. Short stature, webbed neck, and pulmonic stenosis
E. Weakness, club feet, immobile face, and inadequate respirations

4- Among the following, the condition that is MOST likely to predispose a


pediatric patient to the development of systemic candidiasis is
A. history of atopy
B. history of prematurity
C. immunosuppression
D. presence of an indwelling urinary catheter
E. recent tonsillectomy
5- After intubation, arterial blood gas measurements for a 12-hour-old term infant
include a PO2 of 18 torr and a PCO2 of 25 torr while receiving ventilation
with 100% FIO2. The arterial pH is 7.35, and the umbilical line arterial
blood pressure is 75/45 mm Hg. There is no cardiac murmur. Chest
radiography shows normal cardiac size and diminished pulmonary
vascularity.
The mechanism by which intravenous prostaglandin E1 will benefit this infant is
MOST likely due to
A. decreased pulmonary vascular resistance
B. decreased systemic vascular resistance
C. increased mixing of systemic and pulmonary circulations
D. increased pulmonary blood flow
E. increased systemic blood flow

6- A female infant born to a 24-year-old woman has been diagnosed clinically as


having Down syndrome. The mother is concerned about her risk of having
another child who has a chromosomal abnormality.
The statement that you are MOST likely to include in your discussion is that her
risk
A. can be estimated by determination of maternal serum alpha-fetoprotein in all
future pregnancies
B. cannot be estimated until her infant's chromosome complement has been
determined
C. is increased for Down syndrome, but not for any other chromosomal
abnormality
D. is no greater than that of any other woman her age
E. is not increased until she reaches the age of 35

7-During delivery of an infant who has an estimated gestational age of 42 weeks,


you note that the amniotic fluid looks like pea soup and contains thick
particles of meconium.
Of the following, the MOST important initial step in resuscitation of the infant is to
A. aspirate the gastric contents
B. determine the Apgar score
C. initiate tracheal intubation
D. provide positive pressure ventilation
E. suction the hypopharynx
9- In addition to irritability, sweating, and difficulty breathing with feeding, the
symptom that is MOST indicative of congestive heart failure in a 3-week-old
infant is
A. ascites
B. cough
C. cyanosis
D. diminished feeding volume
E. pretibial edema

10- An infant who was born with myelomeningocele has hydrocephalus, and a
ventriculoperitoneal shunt is placed. His parents are concerned about
recognizing the signs and symptoms of shunt blockage.
Of the following, the BEST information you can give the parents is that
A. behavioral changes such as decreased spontaneity and mild lethargy may
indicate shunt malfunction
B. fewer than 30% of all shunts malfunction
C. most shunt failures occur more than 5 years after the initial surgery
D. seizures are the most common manifestation of shunt malfunction
E. shunt malfunction is usually due to infection

11- You are evaluating a 1-month-old boy who has had inspiratory stridor since
birth. The stridor is associated with retractions when the infant becomes
agitated. Physical examination reveals a weight of 3.4 kg (10th percentile),
no expiratory stridor, and weak cry.
Of the following, the MOST likely cause of the stridor in this infant is
A. bilateral paralysis of the vocal cords
B. laryngeal cleft
C. laryngomalacia
D. subglottic cyst
E. tracheomalacia

12- A male infant is born at an estimated gestational age of 34 weeks. His


measurements at birth are: weight, 1,200 g (<10th percentile); crown-heel
length, 40 cm (10th percentile); and head circumference, 31.5 cm (50th
percentile).
Of the following, the MOST likely explanation for the growth pattern of this infant
is
A. chromosomal abnormality
B. congenital viral infection
C. gestational diabetes
D. hereditary constitution
E. pregnancy-induced hypertension

13- You are writing orders for parenteral nutrition for a 24-hour-old infant who
weighs 900 g. The infant is on a ventilator, but clinically stable.
Of the following, the MOST appropriate order is to
A. add cysteine
B. add sodium to provide 3.0 mEq/kg per day
C. provide a protein intake of 3.0 g/kg per day
D. provide nonprotein calories of at least 30 kcal/kg per day
E. use casein hydrolysate as a source of protein

14-A male infant is delivered after a pregnancy complicated by breech


presentation. Physical examination reveals internally rotated shoulders,
decreased muscle mass, extended elbows, contractures of the hands, flexion
contractures at the knees, and bilateral equinovarus deformities.
Of the following, the MOST likely outcome for this infant is
A. ambulation after initiation of early physical therapy and surgery
B. death during the first year of life due to respiratory failure
C. death in the first days of life due to renal disease
D. progressive contractures and scoliosis
E. severe psychomotor retardation and failure to attain motor milestones

15- The decreased incidence of enteric infections noted in breastfed infants


compared with formula-fed infants is MOST likely due to the
A. more alkaline stool pH in breastfed infants
B. nutritional benefits of human milk on the infant's immune system
C. predominance of Bacteroides and Clostridium in the gut of breastfed infants
D. presence of protective antibodies against enteric infection in human milk
E. sterility of human milk

16- A newborn who weighs 600 g and whose estimated gestational age is 24 weeks
at birth is admitted to the neonatal intensive care unit after successful
resuscitation in the delivery room. Arterial blood gas measurements on room
air are: pH, 7.35; PCO2, 42 mm Hg; PO2, 68 mm Hg; base deficit, 2 mEq/L.
Of the following, the MOST appropriate initial management is to
A. begin intravenous vancomycin
B. begin phototherapy
C. initiate enteral feeding
D. provide bicarbonate infusion
E. provide glucose infusion

17- A 3-day-old infant presents to the emergency department with vomiting,


lethargy, hypotonia, and jaundice. Physical examination reveals
hepatomegaly and neurologic depression. A full sepsis evaluation is
undertaken, and the Gram stain of the cerebrospinal fluid reveals gram-
negative organisms.
Of the following, the BEST additional laboratory test to obtain is
A. erythrocyte galactose-1-phosphate
B. liver glycogen content
C. plasma insulin level
D. plasma very long-chain fatty acids
E. stool porphyrins

18- You are examining a term newborn in the nursery. His weight is 3.27 kg (50th
percentile), and his length is 50.5 cm (50th percentile). The pregnancy,
labor, and delivery were unremarkable. There are no significant findings on
physical examination.
The MOST likely head circumference in this child, if it is consistent with his other
growth parameters, is
A. 31 cm
B. 33 cm
C. 35 cm
D. 37 cm
E. 39 cm

19- previously healthy 5-day-old male who was born at home develops bruising
and melena. The pregnancy, delivery, and postnatal course were
unremarkable. The infant is breastfeeding vigorously every 2 hours.
Findings on physical examination are unremarkable except for several large
bruises. Laboratory testing reveals: hemoglobin, 81 g/L (8.1 g/dL); white
blood cell count, 9.4 x 109/L (9,400/mm3); prothrombin time, 37 seconds;
partial thromboplastin time, 98 seconds; platelet count, 242 x 109/L
(242,000/mm3); and fibrinogen, 2.34 g/L (234 mg/dL).
Of the following, the MOST likely cause of the bleeding is
A. disseminated intravascular coagulation
B. factor VIII deficiency hemophilia
C. liver disease
D. vitamin K deficiency
E. von Willebrand disease

20- A newborn whose estimated gestational age is 42 weeks is stained with


meconium. Tracheal intubation reveals meconium in the hypopharynx as
well as below the vocal cords. The infant has respiratory distress. A chest
radiograph is obtained.
Of the following, the MOST likely radiographic finding is
A. coarse infiltrates
B. decreased lung volumes
C. mediastinal shift
D. pleural effusion
E. reticulogranular pattern

21- A 20-year-old primigravida at 30 weeks of gestation has a blood pressure of


160/112 mm Hg, serum total bilirubin level of 44.5 mcmol/L (2.6 mg/dL),
serum alanine aminotransferase level of 150 U/L, and platelet count of 75 x
109/L (75,000/mm3). She is hospitalized for observation and electronic fetal
heart rate monitoring.
Of the following, the MOST ominous sign of fetal distress during monitoring
would be
A. early decelerations
B. increased beat-to-beat variability
C. late decelerations
D. spontaneous accelerations
E. variable decelerations

22- A term newborn presents with bilious vomiting shortly after birth. Her
abdomen is distended slightly, and facial features are characteristic of Down
syndrome. She has passed a normal meconium stool. The pregnancy was
complicated by polyhydramnios.
Of the following, the MOST likely diagnosis is
A. duodenal atresia
B. Hirschsprung disease
C. meconium ileus
D. midgut volvulus
E. pyloric stenosis

23 - An 18-year-old primigravida at 32 weeks' gestation has a blood pressure of


148/96 mm Hg, proteinuria, oliguria, and visual disturbances. Labor is induced,
and the infant is delivered. His weight is 850 g (<10th percentile), crown-heel
length is 38 cm (10th percentile), and head circumference is 30 cm (50th
percentile).
Of the following, the MOST likely complication in this infant is
A. anemia of prematurity
B. hyaline membrane disease
C. hyperglycemia
D. meconium aspiration
E. perinatal asphyxia

24- An infant is born at 27 weeks' gestation following a pregnancy complicated by


preterm labor that progressed despite administration of a tocolytic agent.
Of the following, the most appropriate INITIAL management is to
A. measure transcutaneous oxygen saturation
B. perform endotracheal intubation
C. place an umbilical arterial catheter
D. place the infant in an open bed warmer
E. provide nasal continuous positive airway pressure

25- Of the following, the MOST important determinant of neurodevelopmental


outcome of VLBW infants is
A. antenatal obstetric management
B .infant gender
C. length of gestation
D. maternal education
E. socioeconomic status

26- A 10-day-old infant who weighed 1,750 g at birth and whose gestational age
was 34 weeks is jaundiced. His total serum bilirubin concentration is 10.0 mg/dL
and the direct fraction is 0.8 mg/dL. He is receiving intermittent orogastric feeding
of expressed human milk and supplemental parenteral nutrition.
Of the following, the MOST likely explanation for these findings is
A. Crigler-Najjar syndrome
B. jaundice due to parenteral nutrition
C. neonatal hepatitis
D. physiologic jaundice
E. pyloric stenosis
27- Early hospital discharge is defined as the discharge of a newborn earlier than
48 hours following vaginal delivery or 96 hours following cesarean delivery.
Of the following, the MOST common reason for readmission to the hospital within
7 days following an early discharge is
A. bacterial sepsis
B. congenital heart disease
C. gastrointestinal malformation
D. hyperbilirubinemia
E. metabolic disorders

28--A newborn infant is delivered by emergent cesarean section at 41 weeks'


gestation following a pregnancy complicated by a prolapsed umbilical cord and
meconium-stained amniotic fluid. At 6 hours of age, the infant has a generalized
tonic-clonic seizure.
Of the following, the MOST likely explanation for this seizure is
A. hyponatremia
B. hypoxic-ischemic encephalopathy
C. intracranial hemorrhage
D. meningitis
E. pyridoxine dependency

29- A 4-hour-old newborn has copious oral secretions and episodes of coughing,
choking, and cyanosis. The pregnancy was complicated by polyhydramnios. You
suspect esophageal atresia with tracheoesophageal fistula.
Of the following, the MOST helpful test to confirm the diagnosis is to
A. inject a contrast medium through an orogastric catheter and obtain a neck
radiograph
B. obtain computed tomography of the neck
C. perform flexible bronchoscopy
D. place an endotracheal tube and examine the endotracheal fluid
E. place an orogastric suction catheter and obtain a chest radiograph

30- A newborn is delivered by emergent cesarean section because of fetal distress


following acute abruption of the placenta. The infant is resuscitated and transferred
to the nursery. On physical examination, she appears pale, and her extremities are
cold to touch. The capillary refill is 8 seconds. Results of an arterial blood gas
analysis show a Po2 of 48 mm Hg.
Of the following, the MOST likely additional finding is
A. decreased bicarbonate concentration
B. hypercalcemia
C. hyperglycemia
D. hyponatremia
E. normal anion gap

31- Of the following, erythromycin prophylaxis is MOST likely to prevent ocular


infection due to
A. Chlamydia trachomatis
B. group B streptococci
C. Neisseria gonorrhoeae
D. Staphylococcus aureus
E. Trichomonas vaginalis

32-A 1,300 g infant who is born at 34 weeks' gestation has a head circumference of
27 cm and crown-heel length of 40 cm. At 48 hours of age, she is irritable,
tremulous, and inconsolable. Her systolic blood pressure is 65 mm Hg and heart
rate is 180 beats/min. Her face appears normal, and her cry is high-pitched. Cranial
ultrasonography reveals bilateral echo densities suggestive of periventricular
leukomalacia.
Of the following, the MOST likely explanation for the findings in this infant is
maternal exposure to
A. alcohol
B. barbiturates
C. cocaine
D. marijuana
E. opiates

33- A 4.3 kg infant is delivered to a woman whose diabetes mellitus is poorly


controlled.
Of the following, the MOST likely neonatal manifestation of maternal diabetes is
A. diabetic ketoacidosis
B.Hirschsprung disease
C.hypercalcemia
D.polycythemia
E.renal vein thrombosis

34 Of the following, the MOST helpful finding to distinguish GBS pneumonia


from RDS is
A. a normal C-reactive protein level
B. an elevated erythrocyte sedimentation rate
C. diffuse alveolar infiltrates on chest radiography
D. increased ratio of bands to segmented neutrophils
E. persistent hypoxemia on blood gas analysis

34- An 18-hour-old infant of a diabetic mother develops abdominal distension.


Physical examination reveals a protuberant, firm, but nontender abdomen; patent
anus; and no grossly visible anomalies. The infant has passed no meconium stool
since birth. A supine abdominal radiograph reveals multiple dilated loops of
intestine.
Of the following, a contrast enema would MOST likely confirm a diagnosis of
A. atresia of the colon
B. Hirschsprung disease
C. hypoplastic left colon syndrome
D. meconium ileus
E. midgut volvulus with malrotation

35- You are attending the emergency delivery by cesarean section of a primiparous
woman. The gestation was complicated by pregnancy-induced hypertension. Deep
variable fetal heart rate decelerations were noted during labor. At delivery, the
infant is acrocyanotic with poor tone; spontaneous movement and minimal
respiratory effort are present.
Of the following, your INITIAL management is to
A. ascertain the heart rate and assign a 1-minute Apgar score
B. begin tactile stimulation and provide blow-by oxygen supplementation
C. dry all skin surfaces and clear the oropharynx
D. initiate bag-mask ventilation
E. insert an umbilical catheter and administer naloxone

36- A vigorous, normal-appearing term male newborn has not voided by 18 hours
after delivery. Perinatal history is negative for maternal illness or medications.
Amniotic fluid volume was reportedly normal, and the delivery was uneventful,
with Apgar scores of 6 and 9 at 1 and 5 minutes, respectively.
Of the following, the MOST likely reason why this 18-hour-old infant has not
voided is
A. bilateral ureteropelvic junction obstruction
B .intravascular volume depletion
C. neurogenic bladder
D. posterior urethral valve
E. undocumented void in the delivery room
37-A 2-week-old neonate who was born at 32 weeks’ gestation has recovered from
respiratory distress syndrome. He has been tolerating increasing volumes of enteral
feedings via gavage. Over the past several feedings, abdominal distension, gastric
residuals, and stools that are positive for blood have been noted.
Of the following, the radiographic finding MOST supportive of the diagnosis of
necrotizing enterocolitis is
A .absence of luminal bowel gas
B. generalized bowel distension
C .intraperitoneal fluid
D. pneumatosis intestinalis
E. thickening of the bowel wall

38- A term infant is placed under a radiant warmer, the skin is dried, and the
oropharynx and nose are suctioned. After tactile stimulation, there is minimal
respiratory effort, dusky color, and a heart rate of 86 beats/min. Bag/mask
ventilation is performed for 30 seconds with 100% oxygen at a rate of 40 to 60
breaths/min. The heart rate increases to 100 beats/min.
Of the following, the NEXT best step is to:
A. administer sodium bicarbonate
B. continue bag/mask ventilation at a rate of 20 to 40 breaths/min
C .continue ventilation and begin chest compressions
D. observes for spontaneous respiration and discontinues ventilation
E. perform endotracheal intubation

39- A 900 g male infant is delivered vaginally to a woman who had no prenatal
care.
Of the following, the physical finding that is MOST consistent with prematurity
rather than intrauterine growth restriction is
A. creases over entire sole of foot
B. descended testes with deep rugae of the scrotum
C. formed and firm pinna with instant recoil
D. gelatinous translucent skin
E. raised areola and 3 mm breast buds\

40 -A 1-day-old infant develops bilious vomiting and gastric distension. She has
been afebrile and has been passing meconium-laden stools.
Of the following, the most appropriate INITIAL step in the management of this
infant is
A. abdominal radiography to look for the “double-bubble” sign
B .culture of a catheterized urine specimen
C .insertion of a rectal tube for decompression
D .placement of a nasogastric tube and initiation of intravenous fluid therapy
E. upper gastrointestinal radiographic series to look for malrotation of the small
bowel

41- While performing ultrasonography on a 31-week fetus, an obstetrician notes


that the fetal heart rate ranges from 62 to 66 beats/min. Fetal growth appears
normal, and no structural cardiac anomalies are identified. Fetal echocardiography
reveals that the fetal atria appear to be contracting at 140 beats/min, with a
ventricular rate of 65 beats/min.
Of the following, the NEXT step in the management of this infant is to
A. administer beta-agonist drug therapy to the mother
B. assess the cardiac status of the infant following labor and delivery
C. counsel the parents that intrauterine fetal death is likely
D. perform amniocentesis to confirm lung maturity and if mature, perform
immediate cesarean section
E. repeat the fetal echocardiography and fetal ultrasonography in 1 week

42- The mother of a 2-week-old infant is concerned because her baby has not had a
stool in 7 days. She has been exclusively breastfeeding him every 2 to 3 hours
since her milk came in, but the baby has only passed flatus. The infant is gaining
weight well. Results of physical examination are normal.
Of the following, the best course of INITIAL management for this infant is

……………………….
43- term infant is cyanotic and requires intubation. Findings include: heart rate,
175 beats/min; blood pressure, 60/30 mm Hg; increased right ventricular activity;
single S2; short systolic murmur; and equal arm and leg pulses; chest radiography,
normal heart size and pulmonary congestion. Arterial blood gases (right radial
artery on 100% FIO2): pH, 7.31; PO2, 43 torr; PCO2, 48 torr.
Of the following, the MOST likely diagnosis is
A. hyaline membrane disease
B. hypoplastic left heart
C. intrauterine constriction of the ductus arteriosus
D. tetralogy of Fallot
E. total anomalous pulmonary venous connection
44- A 7-day-old infant has copious purulent discharge from both eyes. The 17-
year-old mother currently complains of a yellowish vaginal discharge. The only
medications received by the infant were vitamin K and topical erythromycin
prophylaxis following delivery. Giemsa stain of a conjunctival scraping reveals
intracytoplasmic inclusions.
After obtaining appropriate diagnostic studies, the BEST management includes
treatment with
A. oral erythromycin
B. oral penicillin
C. topical erythromycin
D. topical gentamicin
E. topical sulfonamide

45- Of the following, the condition that is MOST likely to present with seizures
during the first 24 hours of life is
A .fetal alcohol syndrome
B .herpes simplex infection
C. hypoxic-ischemic encephalopathy
D. organic acidemia
E. urea cycle defect

Potrebbero piacerti anche